Fop Exams Categorized-2

You might also like

Download as pdf or txt
Download as pdf or txt
You are on page 1of 285

Fop exams

categorized
First edition—Dec 2020
Disclaimer
Dear Doctors, please Don’t take
answrs in this sheet for granted
make your best effort to verify the
answers .
also note that, a lot of questions
are incomplete ,
It will help but also may deceive you
So consider them as a guide in your
study.
Best wishes
Dr. Sh

‫نسألكم الدعاء‬
Index
1. Genetics 1
2. Cardiology 8
3. Respiratory-ENT 20
4. Nephrology-urology 38
5. Neurology 52
6. GIT-Hepatology-Nutrition 66
7. Endocrine-DM 89
8. Haematology-oncology 111
9. Infections-Immunization 129
10. Metabolic 159
11. Musculoskeletal 162
12. Dermatology 173
13. Ethics-adolecent 181
14. Patient safety 190
15. Saufgaurding 195
16. Neonatology 210
18. Develomental assessment 223
19. Emergency 243
20. Pharma-Accidents-Poisoning 252
21. Psychiatry 262
22. Palliative& pain management 272

Click on the chapter’s name to take you directly to its page


Genetics

Page |1
Genetics

Oct 2020
1. Boy with cystic fibrosis. What is the chance of his sisters to be carrier of
cystic fibrosis?
a. ¼
b. 2/3
c. ½
d. No carrier.

Feb 2020
1. Genetics:
a.Multifactorial
b.karyotype XO
c. X linked recessive
d. X linked dominant
e.Autosomal recessive
f. Autosomal dominant

A. Baby 3 weeks hungry, vomiting forceful, and history grandparent with


operation in young age multifactorial
B. Short girl, learning difficulties, webbed neck Karyotype XO
C. Baby Boy 21 months old pull to stand hold on furniture, cannot walk tone
normal, history of cousin with disability X linked recessive

2. Baby was under child protection came with smooth philtrum and learning
difficulty and thin upper lip. What is diagnosis?
a. Fetal alcohol syndrome
b. Brader willi Syndrome
c. Phynotonin syndrome

Page |2
Genetics

OCT 2019
1.EMQ:
a. 1:4
b. 1:2
c. 2:3
d. 1:100
e. 1:10
f. population risk
A. A case of sickle cell and asking of risk of having another affected child
1:4
B. father with hemophilia A what are the risks for having an affected boy
population risk
C. risk of having a carrier sibling for a family with affected cystic fibrosis child
2:3
2. Tall boy, secondary sexual characters not well developed teased at school
with mild learning difficulty.
a.46xy
b.47xxy
c.46xx

June 2019
1. Girl height 0.2% centile and weight 25% , mild learning disability ,Next?
Karyotyping
2. female student 18 years pregnant 8-wk with cystic fibrosis ,,husband not
affected with no family of cystic fibrosis she want to continue the
pregnancy
wants to know if the baby will be affected ..what advice ????
a. Wait for neonatal screening
b. Screen father for most common mutation of cystic fibrosis
c. Amniocentesis
d. Chorionic villous sampling
e. DNA after delivery

Page |3
Genetics

Feb 2019
1. Down syndrome 11-yr asymptomatic playing football short stature
without previous follow up?
a. Cervical X-ray
b. Thyroid function test
c. Blood film
d. Celiac screening
2. Boy with cystic fibrosis, what is the chance of carrier for his sibling?
2/3

Oct 2018
1.26-X liked pattern SCINARIO-what to do as mother ask about fate of her
baby? 5 month pregnant lady asks for genetic testing , maternal grand
uncle died and maternal aunts daughter had an abortion who was a
male. Advice given?
check if she is a carrier

June 2018
1. EMQ:
a. AD
b. AR
c. X-linked dominant
d. X-linked recessive
e. multifactorial
f. XO
g. XXY
a. a boy delay walking then start to walk, apart of hypotonia other
examination normal, his uncle was using wheelchair
X-linked recessive
b. short girl her height 0.4th centile mid parental height 25th centile, no
obvious physical feature XO
c. persistent vomiting at 3-6 week has grand father did gastro surgery at child
age multifactorial

Page |4
Genetics

Feb 2018
1. EMQs:
a. Genetic risk calculation
b. 1:4
c. 1:2
d. Population risk
e. 1
f. 2/3

A. Risk of having an affected baby for a well parent who have a son with
sickle cell disease?
1:4
B. Risk of having a baby boy with hemophilia to a father has hemophilia A?
population risk
C. Risk for a daughter to be a carrier of cystic fibrosis, she has an affected
sibling
2/3
2. Bay with mild dysmorphic featured, thin upper lip and smooth philtrum?
Fetal alcohol syndrome

June 2017
1. EMQ:
mode of inheritance in case of:
A. prader willi – imprinting
B. myotonia -- AD
C. rickets in child and her mother have genu valgum – Xl-dominant

2. girl with short stature and learning disability what investigation to-do?
Karyotype

Page |5
Genetics

Feb 2017
1. Case with manifestations of gynecomastia, small testicles, tall stature.
Klinefelter
2. A pregnant says her uncle died at age 1 year because of an immune
disease.Then the son of her aunt's daughter died because of immune
disease. She wants investigation for her unborn baby.
tell her she can be investigated for carrier state
3. EMQ: Mode of inheritance
A. A case with vomiting at 1 month. History of uncle went a GIT operation
at infancy.
Multifactorial pyloric stenosis
B. A case with features of turner.
XO karyotyping
C. A case with features of Duchene
XLR

Oct 2016
1. 11 years old Down syndrome patient, asymptomatic, present to your clinic
for regular check-up. O/E looks well, weight 0.4 centile. As a screening,
you should order:
Thyroid function test. ??
Coeliac screening.
Blood sugar.
X-ray spine
Blood film.
June 2016
1. neonate delivered with cleft palate
22q11 Di George

Page |6
Genetics

June 2015
1. Female with short stature examination is normal Mid parental height at
25% Her ht 0.4 centile
a. Xo. Turner
b. X linked recessive
c. X linked dominant
d. Autosomal recessive
e. Multifactorial
3. Mother has cataract her uncle has problem Her baby boy has myathy
What is inheritance?
a. Mitochondrial
b. X-linked recessive
c. X-linked dominant
d. Autosomal recessive
e. Autosomal dominant myotonic dystrophy
Oct 2013
1. A 10 years old tall boy with gynecomastia and small testes.
What is your possible diagnosis?
a. 47 XXY
b. 46 XO
c. Marfan’s syndrome
June 2011
1. Parents deaf wants to know either their child will be deaf or not,
How will u tell them?
a. Karyotyping
b. Check by music the fetal movements
c. Amniosentecis???
2004-2005-2006
1. Interrupted aortic arch, microcephaly. Possible mode??
a. Chromosomal
b gene
c.Hereditery

Page |7
Cardiology

Page |8
Cardiology

Oct 2020
1. Girl with tachycardia narrow complex more than 300 b/m which drug you
give?
a. Propranolol
b. Digoxin
c. Adenosine
d. Amiodarone
2. 12-days old baby girl with 2 days deterioration and lethargy poor feeding
lymphedema in (hands or feet) weak femoral pulsations murmur 2/6
with normal heart sounds. CRT = 3, on 100% O2 but no oxygen saturation
can be recorded. What drug to give?

a. Prostaglandin infusion
b. IV antibiotic
c. IV acyclovir
d. Dopamine
e. Lasix
3. Girl with clinical feature and recent Dx of rheumatic fever,
what investigation to roll out or in the Dx?
a. ASO titre
b. Echo
c. Blood culture
d. Throat swab
4. 12-years-old girl was previously healthy, Hx of hurting her Lt shoulder
and she did nose piercing which made infection to skin and responded to
topical antibiotic ttt. She is presented now with fever and murmur on
apex. Ask about investigation?

a. Blood culture
b. Echocardiography
c. Urea and electrolyte

Page |9
Cardiology

5. Child with small VSD needs elective tonsillectomy. The ENT doctor needs
pediatric consultation about prophylactic antibiotics?
a. No need for prophylactic antibiotics
b. cIv antibiotic
c. Chlorhexidine mouth wash
d. Give him oral antibiotic

6. Female baby 6 weeks, non-dysmorphic, presented with lethargy, poor


feeding during examination: lymphoedema on lower limbs &
systolic murmur
Co-recitation of aorta --Turner $

Feb 2020
1. Child 3 months old with mild dysmorphic features His O2 saturation 95%
with loud systolic murmur all over pericardium not cyanosed with FTT
a. VSD
b. TGA
c. ASD
d. PDA
e. TOF
2. Neonate came with heart failure What is the sure sign of Heart failure?
a. hepatomegaly
b. Cyanosis
c. Weak Peripheral pulses
d. Pedal Edema
3. Child 7 years has infrequent SVT Attacks mother is worried if he has attack
outside. What to do?
a. immerse face in ice water
b. inflate in New balloon
c. Ocular Pressure
d. propranolol

P a g e | 10
Cardiology

Oct 2019
1.EMQ:
a.VSD
b. ASD
c. Pulmonary Stenosis with normal heart sound
d. PDA
e. innocent murmur
A. 4 or 6month with ejection systolic murmur the left upper sternal
PS with normal heart sound
B. 34 weeks at 10 days with long systolic murmur BP 70/30 mmHg rt arm
and Lt leg no deference (Wide pulse pressure)
PDA
C. patient with loud systolic murmur at left lower sternal
VSD

2. patient had illness after which he had basal crepitation,HR140,RR40 ,liver


2cm what’s the diagnosis?
a.Viral myocarditis
3.boy collapsed after swimming, slowly recoverd his conscious hus cousin
with Epilepsy, What to do?
a. EEG
b. ECG
c.excersice ECG
4.down syndrome 12 year old child with VSD,developed fever,had
spleen 4 Cm Urine +++ blood What’s the diagnosis?
a. ALL
b. infective endocarditis

P a g e | 11
Cardiology

June 2019
1.EMQ
A. Classical VSD PAN SYSTOLIC murmur
VSD
B. Neonate with pounding pulse murmur continuous
PDA
C. Ejection systolic murmur in 3 months well infant in Lt intercostal space
Pulmonary stenosis
2. collapse after swimming, + family history uncle in follow up with cardio
what diagnosis?
a. long Qt
b. HOCM

Feb 2019
1. small VSD with plan for tonsillectomy no prophylaxis need:
a. chlorohexidine mouth wash
b. erythromycin
c. cardiologist opinion
d. no prophylaxis need
2. 4 weeks of fever, headache, malaise, rigors, plus spleenomegaly and
grade 2 diastolic murmur?
a. Infective endocarditis
b. Rheumatic fever
c. malaria
3. Acute rheumatic fever to rule out or in?
a. ESR
b. ASO
c. ECHO

P a g e | 12
Cardiology

4. 9month old baby with birth WT of 3.7 kg, FTT with WT below 0.4th Centile.
basal crepitations with distress
a. PCP
b.VSD

Oct 2018
1. EMQ
A. 6 months girl with BP 100/ 70 femoral pulse not delayed than brachial
and low volume---coarctation of aorta
B. 3-yr old with loud systolic murmur @mid LLSB, thrill + VSD

C. one with ESM at PA and ejection click—pulmonary stenosis


(aortic stenosis radiated to neck and pulmonary radiate to back)

2. Sign of cardiac disease


a. Cyanosis
b. peripheral edema
c. distended neck veins
d. hepatomegaly

3. Case of a girl with SVT first step


a. Valsalva maneuver
b. ice pack

4. What is duct dependent disease


pulmonary atresia

June 2018
1. child ejection systolic murmur on right sternal margin radiate to neck with
thrill
a. AS
b. PS

P a g e | 13
Cardiology

2. down with lethargy fever 1 month and tachycardia tachypnea and VSD and
fever HR = 190 with liver 4 finger below costal margin, splenomegaly,
hematuria.
a. infective endocarditis
b. viral myocarditis
3. EMQ:
a. VSD
b. ASD
c. PDA
d. pulmonary valve stenosis
e. supra pulmonary stenosis
f. COA
g. Innocent murmur.
all SPO2 > 95% and child not cyanosed
A. 1 month, 2-3/6 systolic murmur left lower sternal margin. VSD
B. neonatal on discharge ejection systolic murmur on left upper sternal
margin with thrill PS
C. neonatal on 10 days long systolic murmur left upper sternal margin both
BP in left arm and leg high normal SBP 70 PDA

Feb 2018
1. Case of AS for septic operation?
a. Operate and don’t give prophylaxis
b. Give prophylaxis in the highrisk procedures
c. No prophylaxis and come if Pt developed symptoms
2. Teenager has ear piercing after which she developed local infection, pale,
has apical murmur, night sweating, investigation?
a. Blood film
b. repeated Blood culture
c. Echocardiogram

P a g e | 14
Cardiology

3. Boy collapsed during a hockey game, then slowly regained consciousness,


his cousin has epilepsy, investigation?
a. ECG
b. EEG
c. Blood glucose

Oct 2017
1. EMQ
a. ASD
b. PS
c. PDA
d. VSD
e. TOF
f. TGA
g. AS-
h. innocent murmur

A. loud pansystolic murmur on lower sternal edge VSD


B. bounding pulses with continuous murmur PDA
C. 8-week-old ejection systolic murmur second left inter costal space. PS
2. EMQ
A. scenario baby cyanosis and no murmur when he pull his leg up TOF
B. ejection click 2nd ics to the right radiate to neck AS
C. Scenario with viral infection then 1/6 mid-systolic murmer
Innocent Murmur
3. Adolescent girl 4-week hx of fever, night sweet headache weight loss
splenomegaly heart murmur grade2/6 clubbing
a. non-Hodgkin lymphoma
b. Sarcoidosis
c. Infective endocarditis
d. Rheumatic fever
e. SLE

P a g e | 15
Cardiology

3. Child with undetectable pulse, HR 300/min and narrow complex


tachycardia and stable what is the drug of choice?
a. Adenosine
b. Amiodarone
c. Flecainide
d. Propranolol

4. one question about syncope after swimming, family hx of his uncle when
he was young and following with cardiology, Diagnosis?
a. HOCM
b. AS
c. Long QT

5. weak radial pulse, absent femoral pulse


a. coarctation of aorta
b. Aortic stenosis

June 2017
1. Collapse after playing football, Investigation?
12 leads ECG
2. test supporting diagnosis of rheumatic fever
ASOT

Feb 2017
1. A girl with attacks of palpitation and tachycardia 3 times within 6 months
after exercises. What to do?
a. 12 lead ECG
b. 24 Hour ECG
c. Exercise ECG

P a g e | 16
Cardiology

2. What is the cardiac sign that always indicate a pathological cardiac lesion?
Thrill
3. Case feverish + tachycardic + tachypneic + low pulse volume
+hepatomegaly.
myocarditis

4. EMQ:
A. PDA cardiac sign? Bounding pulse
B. TGA cardiac sign? Central cyanosis
C. WPW cardiac sign? Rate around 250

June 2015
1. RH. fever recently diagnosis: investigation??
a. ASOT
b. ESR
c. ECHO
2. boy playing football, sudden syncope, become well and neurologically
normal, what is the best investigation for diagnosis?
a. EEG
b. MRI
c. ECG- prolonged qt 12 lead
d. ECG
e. Blood sugar

Oct 2013
1. A 3 years old child with a small VSD. He has to do adenoidectomy.
What you will do for prophylaxis against infective endocarditis?
a. Give amoxicillin before the procedure
b. No prophylaxis is indicated

P a g e | 17
Cardiology

2. A 5 years old child suddenly collapsed after entering the swimming pool,
but he regains conscious few minutes after transfer to the hospital.
His uncle had a similar attack 10 years ago and had a long term follow up
with a cardiologist. What is your possible diagnosis?

a. Prolonged QT syndrome
b. Hypertrophic obstructive cardiomyopathy

2004-2005-2006
1. A girl of 3 years age previously fit and well with no health problems came
to admission unit, her BP is 136/90, the best reason to explain this?

a. Nurses taking the wrong Bp


b. BP stadiometer (automated BP machine) is used
c. the child is apprehensive
d. Coarctation of aorta
e. renal artery stenosis

2. child with following pressure and sats on cardiac catherization:

Saturation pressure
a. Coarctation of aorta SVC-78% Rt Vent 30/5
b. ASD Rt.atrium-78% PA 100/30
c. VSD
Rt.ventricle 78% Aorta 100/60.
d. PDA
PA 85%
e. tetralogy of fallot
PV 98%
Lt.Atrium 98%
Lt ventricle 98%

P a g e | 18
Cardiology

3. EMQ
A. a child of 6 years came with chest infection. Murmur can be heard on rt.
upper sternal edge. The murmur is never heard before and disappears
when child is well. he has lots of chest infection in past.
Innocent murmur

B. a 6months old child who is very restless and folding his legs on his
abdomen
and murmur on lower sternal border
TOF

P a g e | 19
Respiratory
ENT

P a g e | 20
Respiratory-ENT

Oct 2020
1. EMQ:
Bronchoscopy
FBC and film
CT chest
IgE level
IgG subsets
esophageal PH study
peak expiratory flow reading
pulmonary function test
short synacthen test
US chest
A. 2-years-old boy has been admitted to hospital 4 times in the last 6 months
with Dx of croup, on one occasion he required intubation and ventilation.
He was born at 26 weeks gestation and was oxygen dependent until the
age of 4 months.
Bronchoscopy

B. 9years o-ld girl present with a 10 days history of fever and cough. She had
responded initially to oral amoxicillin but now look ill and has stony
dullness on Rt lower chest.
US chest

C. An 8 years old boy is taking 750 microgram/day inhaled fluticasone


propionate. He has poor controlled cough and last week has become very
lethargic after short – lived episode of vomiting. Other children in his class
had similar illness but they less severity affected. He has finger clubbing, his
height on 25 th centile.
short synacthen test

P a g e | 21
Respiratory-ENT

2. A child with recurrent attacks of barking cough and mild grade fever.
Ask about diagnosis.
a. Laryngotracheobronchitis
b. Epiglottitis

3. Febrile child with muffled sounds, weak crying and torticollis. I think
there was complain of otitis media also. Best next step?
a. Oral antibiotic.
b. Nebulized adrenaline
c. Urgent refer to ENT
d. X ray lateral view
e. Neck us

4. Infant with poor feeding, lethargy, cough with intercostal and subcostal
recession with oliguria changed just one nappy per day. By examination
wheezes and crackles on auscultation, o2 sat 96%. Mother wanted him to
return home within 2 days because of Christmas.
What the 1st step in management?

a. IV fluid hydration
b. NG tube feeding
c. Nebulized salbutamol.

5. Adolescent girl came to ER with tachypnoea, shortness of breath,


tingling sensations in hands, o2 sat was 99% What to do initially?

a. Rebreathing bag
a. Benzodiazepines
b. O2
c. Iv calcium

P a g e | 22
Respiratory-ENT

6. 3-years boy with recurrent wheezy chest, cough increases with exercise
and his condition improves with inhaled salbutamol.
Which advice best given to parents?
a. Trial of ICS
b. Peak flowmetry

7.Child with 3 attacks of otitis media with effusion (glue ear) in last
4 months and he took antibiotic in each attack now come to you with
new otitis media and holding his ear in pain, mother is worried about
hearing problems associated with glue ear which she heard about.
By examination the child has red tympanic membrane and -ve light reflex
(dull). What is your next step?
a. Give a new course of analgesia on need without review ??
b. Urgent Refer to ENT doctor for grommet insertion.
c. give ibuprofen.
d. Do hearing assessment and if more 30 hz Refer.
e. Give decongestant drug and observe.

P a g e | 23
Respiratory-ENT

Feb 2020
1. 3-years old girl with night and day coughing. Mother is warred the baby
may be asthmatic What to do?
a. peak inspiratory flow and follow up
b. Spirometry after b2 agonist trial
c. try of inhaled steroid for 4 weeks and follow up
d. Spirometry
e. Fraction of inhaled FeNO

2. Child 10 years old on 2 puff preventer q 12 hours on treatment for asthma


was controlled on 2 puffs twice daily. There are new local guidelines
cheaper 15%, the old guidelines recommended to wean if controlled.
a. give 1 puff of old preventer
b. use the new preventer same dose
c. use the old preventer same dose
e. Stop Spacer

3. Child came with cyanosis life threatening asthma take nebulized


salbutamol And Oxygen, What you do next?
a. Call Anaesthesia
b. IV Mg sulphate
c. IV salbutamol
d. IV corticosteroid severe asthma
e. Oral Prednisolone
4. 3 years old Baby boy came by Ambulance from restaurant with drooling of
saliva and Stridor and difficult to swallow and refuse Juice offered with
history of asthma
a. FB inhalation
b. Anaphylaxis
c. Epiglottitis

P a g e | 24
Respiratory-ENT

5. Child 6 years old with history of repeated ear infection and otitis media in
Group activity became angry but can enjoy art lessons
What is the possible cause?
a. ADHD
b. Secretory Otitis media
c. Autism
6. Child 4-years with intermittent stridor and difficult swallowing lumpy
foods What is the diagnosis?
a. Vascular ring
b. NHL
c. Haemangioma
d. Laryngomalacia

Oct 2019
1.EMQ: Same as Sample paper
a. laryngomalacia
b. subglottic stenosis
c. subglottic haemangioma
d. viral croup
e. vascular ring
A. One week old has stridor since birth, increase with sleep and feed
Laryngomalacia

B. weeks old operated for inguinal hernia when he was 4 weeks old, suffering
from stridor, improved partially with steroid, now biphasic stridor
subglottic haemangioma

C. infant went to a party and had two courses of antibiotics but no


improvement had a chest x-ray with lesion in the right lower, after the
second course the same x-ray
Foreign body

P a g e | 25
Respiratory-ENT

2. mum went to GP for her child tonsils what’s the most important point to
put in the referral letter:
a. he has big tonsils
b. recurrent URTI
c. can’t wake him easily in the morning

3. patient had recurrent bronchiolitis in winter, went to a party last week,


then developed a cough, took two antibiotics course, x-ray with
consolidation in the right lower lobe ,the same x-ray after the antibiotics
are finished what’s diagnosis:
a. pneumonia
b. foreign body

June 2019
1. EMQ:
a. antibiotic
b. salbutamol inhaler with space
c. erythromycin
d. Oxygen
e. budesonide
f. oral dexamethasone
g. adrenaline nebulizer
A. 1- months first attack of wheeze, spo2 89% -- Oxygen
B. 2-yrs stridor---oral dexamethasone
C. normal sat 98% diffuse wheeze--- salbutamol inhaler

2. Cystic fibrosis patient, had staph pneumonia then developed, Sudden


difficulty breathing, Shifting trachea to Rt side , Decrease air entry in left
side.
a. Pneumothorax
b. Effusion
c. Empyema

P a g e | 26
Respiratory-ENT

3. 12-year girl with 4 weeks history of wheeze, shortness of breath.


No improvement after salbutamol and steroid inhaled, no history of
asthma, History of weight loss next step?
a. check inhaler technique
b. chest x ray
c. CBC with blood film
d. echo

Feb 2019
1. 3-year-old child on steroids pump 100 microgram BID per day still having
cough during sport and night cough. what will be your next action?
a. steroids 200 mcg twice a day
b. Add of Leukotriene
c. LABA
2. Girl 12-year chest clear RR 45 spo2 99 heart rate normal with tingling?
a. Hyperventilation

3. 7-year-old boy with 100mcg fluticasone Which step in asthma?


a. Leukotriene antagonist
b. increase 200mcq fluticasone
c. LABA
4. Unilateral wheeze
a. Rigid bronchoscope
5. Q about croup

6. FTT with height and weight below second centile


Recurrent chest infection

7. 12 yr female seasonal rhinitis before exam:


a. Cetirizine same day evening
b. Xylometazoline same day as needed

P a g e | 27
Respiratory-ENT

Oct 2018
1. EMQ- same as TAS sample paper investigation?
A. patient on 600 micro cortisone inhaler developed hypotension fatigue
weakness after GE
SHORT SYNCTAN TEST
B. child with stone dullness percussion on rt chest after having symptom of
pneumonia with high fever and increase breathlessness-
CHEST US

C. child with long hx on ventilator for 4 months AS he was preterm 26


weeks and became oxygen dependent – tachypnoeic and need oxygen
presented by recurrent croup??
Laryngoscopy

2. Obese child feeling tired during school hours, has already told parents
about being bullied in school. has stria in the abdomen diagnosis?
a. Depression
b. OSA
C. Cushing

3. Pt with torticollis, ear pain --- retropharyngeal abscess

4. 3y old asthmatic ask about method to give inhaled salbutamol spacer


WITH FACE Mask
5. Girl with breathing difficulty, numbness of fingers, chest clear
SpO2 normal
a. rebreathing mask
b. Iv calcium

P a g e | 28
Respiratory-ENT

June 2018
1. EMQ:
a. oral prednisolone
b. IV salbutamol
c. oral dexamethasone
d. IV fluid
e. Reassurance
f. Salbutamol puff
g. salbutamol nebulizer.
A. 5-year child known Bronchial Asthma came with acute distress, although
given salbutamol nebulizer but saturation remain 88 %
IV salbutamol no IV hydrocortisone in the option (sever BA)
B. child with history of cold presented with mild noisy breathing
oral dexamethasone (mild croup)
C. infant with history of cough and slight decrease oral intake with anxious
mother reassure (acute bronchiolitis)
2. female girl long history of chesty cough multiple previous admission due
to chest infection has a palpable mass upper right quadrant wear same
size of her brother what investigation
a . sweet test
b. Ct chest
c. X ray
3. child with recurrent RTI during past winter after attending a party he
developed sudden onset cough not resolve by 2 courses of antibiotic CXR
twice persistent consolidation finding in right side
a. FB
4. BMI obese headache, at sleep produce sound, not concentrate at school at
home
a. OSA
b. Day dream
c. absence seizure

P a g e | 29
Respiratory-ENT

Feb 2018
1. EMQ:
Sweat test
Bronchoscopy
Dairy for peak flow meter
PH studies

A. Baby with FTT, recurrent chest infections , mother said he has piles
Sweat test
B. Child on inhaled salbutamol, day symptoms improved, still wheezes after
running and has nocturnal cough
Dairy for peak flow meter
C. A toddler developed sudden shortness of breath when they were in a
party, he has a localized wheeze on the right middle lobe?
Bronchoscopy

2. Staph aurous pneumonia patient became more distressed suddenly,


diminished air entry on left side, trachea deviated to right?
a. Pleural effusion
b. Pneumothorax

3. Case of a boy 18 months had normal hearing test, what would warrant
further testing?
a. Temper tantrum
b. Not obeying mother
c. Watching TV at high sound
d. 2 acute otitis media in 3 months

4. Regarding the epidemiology of RSV?


a. Incidence of asthma will increase after RSV infection
b. 15%will require hospital admission (20-35% require hospitalization)

P a g e | 30
Respiratory-ENT

5. 4-year-old with tonsillar enlargement, snoring, what is next to do to guide


to management?
a. Polysomnography
b. Lateral X-ray
c. Over- night oxygen monitoring
6. 4-year-old girl with asthma on salbutamol with controlled her daytime
wheeze, but she still has wheezes at night and when she runs, next step?
a. Inhaled corticosteroids
b. Montelukast

Oct 2017
1. EMQ
a. oral prednisolone
b. oral dexamethasone
c. nebulized
d. adrenaline
e. salbutamol inhaler
f. oxygen
g. nebulized Ipratropium

A. mild croup scenario -- oral dexamethasone


B.18 months wheezy chest-- Salbutamol inhaler
C. long scenario but sp02 - 89%-- Oxygen

2. chronic cough investigated thoroughly normal spirometry. 14 years


Girl coughs during day but no night time symptoms. Management?
a. ENT referral
b. say to her no serious problem
c. reassure habitual cough
d. placebo

P a g e | 31
Respiratory-ENT

3. 7-year-old boy with 100mcg fluticasone Which step in asthma?


a. Leukotriene antagonist
b. increase200mcq fluticasone
c. LABA

4. scenario of 7 years resp infection not responding to antibiotic then


develop red painful lesion on his chin?
Mycoplasma pneumonia
5. 3-year-old boy with inhaled fluticasone 200mcg and salbutamol
But his asthma is not control, what you will do ?
a. LABA
b. Increase inhaled steroid
c. Leukotriene antagonist

6. EMQ
Croup
Laryngomalacia
subglottic stenosis
subglottic haemangioma
vascular rings
inhaled foreign body
A. Scenario of Infant with stridor since age of 2 days increased with
sleeping and feeding.
Laryngomalacia
B. Scenario of infant with progressive stridor since he was 2 weeks and
now stridor is biphasic, history of hernia repair at 4 weeks history of
stridor with some improvement with steroids
subglottic haemangioma
C. Scenario of child with hx of sudden cough then progressive stridor with
respiratory distress.
Inhaled foreign body

P a g e | 32
Respiratory-ENT

June 2017
1. child with wheeze with infection, family hx of atopy and asthma ,,, clear
chest ... Reassure viral induce wheeze?
2. EMQ:
Hearing tests in:
A. New-born—auto acoustic emission
B. child 6 years old – pure tune audiometry
C. child with hearing aid – auditory brain stem response
3. EMQ
A. Asthmatic child on high dose of steroid inhaler, not controlled. .
appear less than his colleague is same class and also Asthmatic.
short synacthen test
B. child have stony dullness in lt lower zone of chest
Chest us

Feb 2017
1. A spirometry readings of a patient: FVC = 95% , FEV1 = 65%, FEV (25-75) =
28% (or similar range of values). What to do next?
repeat the test after beta agonist inhaler

2. A 5-year boy. Teacher complaint of his aggressive behaviour. Boy gets calm
at drawing lessons but angry at group activities. He maintains friendships
in the playground, Scenario wanted to make sure this is not a case of ASD
or ADHD
Secretory otitis media
(hearing loss) has problem with high frequency sound
3. A case with acute asthma. Receiving nebulised salbutamol since 2 hours
now cyanosed, drowsy, can't talk.
call anaesthesia

P a g e | 33
Respiratory-ENT

4. A case with low grade fever and cough


???
5. A case with high grade fever, wheezes and crackles.
Bronchiolitis
6. A case of a girl with seasonal allergic rhinitis. She wants a rapid medication
for her nose as she has exams.
a. oral cetirizine
b. intranasal beclomethason
No options for intranasal antihistamine
7. A case with tonsillitis. A history of penicillin allergy. Best alternative?
Clarithromycin
- Options included different cephalosporins
8. A case with ear pain, fever, unilateral postauricular mass pushed
backwards
and down. Best investigation?
a. US for the swelling
b. Xray
c. mastoid CT
d. Ear swap
9. A case with inspiratory stridor started last night with a significant tracheal
tug diagnosis?
a. viral croup
b. FB

10. A case of diarrhea since birth and failure to thrive. What is the most
important reason for referral of this child to a tertiary care for
investigation?
a. recurrent chest infections
b. height and weight on 2nd centile
c. presence of undigested food

P a g e | 34
Respiratory-ENT

Oct 2016
1. Child known to have frequent viral infection associated with cough and
wheeze. He is playing with pets at home. He is on PRN Ventolin, present
with shortness of breath, cough and fever. Your advice?

a. Inhaled corticosteroids.
b. Remove pets from house.
c. Skin allergy test.
d. Antibiotics for every viral infection.

June 2016
1. neonate become cyanosed on feeding and well on crying there is single
2nd heart sound
upper airway obstruction (choanal atresia charge disease)

June 2015
1. had chest infection, on exam: stony dull chest, what investigation?
a. CT Scan
b. U/S chest

Oct 2013
1. A 2 years old child with croup. Oxygen and budesonide were given but no
improvement. What is your next step?
Nebulized adrenaline, 400 mcg/kg, of 1:1000 solution

P a g e | 35
Respiratory-ENT

June 2011
1. 3year old on 100mg inhaled fluticasone, asthma poorly controlled,
wheezing at night, Management?
a. Increase dose of fluticasone to 200
b. Add long acting bronchodilator
c. Add leukotriene receptor antagonist
d. Add ipratropium bromide
e. Oral prednisolone
2. A mother brought her child to u with recurrent tonsillitis, How will you
assess sleep aponea?
a. Overnight O2 saturation
b. overnight polysomnography
c. tonsils meeting in the midline

3. EMQ
a. Distraction test
b. Test for hearing impairment
c. School for hearing assessment
d. Refer to ENT
e. Speech and language therapist
f. Review in 6 months
g. Reassurance

A. 3yr old with no appropriate speech vocabulary of 20 words, otherwise well


loving play with age-appropriate toys and is loving and very social, but sits
very close to television
Test for hearing impairment
B. 2yr old with 6 words vocabulary, otherwise very loving boy, plays with his
friends, very social, passed neonatal screening test, has chronic otitis
media with dull tympanic membrane?
Refer to ENT

P a g e | 36
Respiratory-ENT

C. 5-months old boy born at 32 weeks, O2 dependent for 4 weeks in Nicu,


passed his screening test , mothers feels that he dosen’t listen,
what will u do?
ABR test

2004-2005-2006
1. Exercise-induced Asthma
a. can be given monoleukast
b. often present as a 1st symptom of asthma
c. cromoglycate

2. 6-year boy suffers from severe asthma and is on regular 4hrly salbutamol
inhalers and becotide inhalers at home. he comes to A&E.
what is the best first management u will give?
a. IV salbutamol
b. salbutamol inhalers
c. IV hydrocortisone
d. IV theophylline
e. oral prednisol

P a g e | 37
Nephrology
Urology

P a g e | 38
Nephrology-Urology

Oct 2020
1. 9-year-old child with primary nocturnal enuresis last year he took
desmopressin when he went for camp and he was ok on it.
Now the child is very distressed about his condition, ask about Mx?
a. Course of desmopressin
b. Enuresis alarm
c. Amitriptyline
d. Imipramine
2. day and night enuresis with drippling ask about possible US
abnormalities
a. Duplex kidney.
b. Horseshoe kidney.
c. Rt kidney mass
d. Hydronephrosis

3. 3-years old child with repeated abdominal pain and vomiting and pallor
with fever weight on 0.4 centile, no organomegaly normal abdominal
examination with hypertension. What Dx?
a. Renal artery stenosis
b. Essential HTN
c. Wilms tumor
d. Dysplastic kidney
e. Coarctation
4. Case of macroscopic haematuria RBCs +3 ,protine +2 on dipstick then
after one week dipstick test showed trace rbcs and no protein
Ask about which will be abnormal during infection.
IgA nephropathy
a. IgA leve
b. C3,C4
c. Abdominal US
d. Cystoscopy
e. Cystogram

P a g e | 39
Nephrology-Urology

5. 6-weeks infant boy with unilateral translucent scrotal mass baby was
calm, well and has good feeding, what is yours next action?
a. Scrotal US
b. Refer to surgen
c. Reassure and follow up if present in 1 year
d. No follow up needed

6. What warrant you for referral to paediatric surgery?


a. Supra umbilical hernia at 1 year
b. Undescended testis at 6 weeks
c. Umbilical hernia
d. Retracted foreskin at 3 y

7. 3 or 4 years old girl came with dysuria (case of simple UTI) with response
to antibiotics in 2,3 days. What will you do next?

a. reassures and give advice about preventive measures


b. do ultrasound

Feb 2020
1. 6 years old boy was day time controlled and night time bed wetting
2 times /Week, What to do?
a. Motivational
b. Alarm
c. Oxybutynin
d. Desmopressin

2. When to refer to Surgery old at 6 months?


a. Inguinal hernia
b. Epigastric hernia
c. Umblical hernia
d. Phymosis

P a g e | 40
Nephrology-Urology

3. Child with UTI and has renal stones by U/S urine analysis +ve leucocytes
and Nitrates what is the cause?
a. E Coli
b. Portus infection
c. Klebsiella

4. 15 years old girl has period 2 weeks ago with flank pain with fever, sweaty
And irregular cycle with pain score6/10 and distressed.
What is the initial investigation?
a. kub and xray
b. urine analysis
c. abd, us
d. blood culture

5. Child 14 years old play Rugby developed Sudden tender scrotum in left
testicular, diagnosis?
a. Lt.Testicular Torsion
b. Torsion Morgagni
c. Inguinal hernia

6. Pt with generalized Oedema with protein ++++and –ve nitrate and


leucocytes puffy eyes What is the treatment?
a. steroid
b. Lasix
c. Albumin

Oct 2019
1. 2 years old boy with polyurea, polydipsia, urine osmorality 200
Diagnosis?
a.DI nephrogenic
b.DI central
c.habitual drinking

P a g e | 41
Nephrology-Urology

2. Patient had coryzal symptoms, developed hematuria ,proteinurea what’s


next investigation:
a. urea, creatinine (IGA nephropathy)
b. FBC

3. 2-month with Klebsila and on antibiotics what to do:


a.US prior to discharge, MCUG at 6weeks, DMSA at4-6 month
b.US prior to discharge MCUG and DMSA at 6 month
c.USand MSUG piror to discharge ,DMSA at 6month

June 2019
1. Scenario facial (preorbital) swelling and edema in foot and large abdomen
(skin rash history)??
a. anaphylaxis
b. nephrotic
c. heart failure
d. preorbital cellulitis
2. day time symptoms, nocturnal bladder instability enuresis still voiding on
cough or exercise, Treatment?
a. Oxybutynin
b. desmopressin
c. amitriptyline
d. enuresis alarm and star chart

3. Umbilical hernia in infant , Mother said he is crying thinking he is in pain


Size 1*1.5 reducible easily what your best advice:
a. Wait for closure
b. Come to ER if vomit
c. Surgical opinion
d. reduced by coin
e. reduce manual

P a g e | 42
Nephrology-Urology

4. 4 weeks unilateral testis palpated at inguinal canal, And other side in


scrotum
a. Review 3m
b. Review 6m
c. Review 12m
d. refer to surgery

5. 5 month baby with typical UTI ( Ecoli respond to antibiotic within 48


hours) the next to do :
a. us after 6 weeks
b. US now
6. There was unilateral testicular pain-less firm swelling after trauma
a. heamatoma ??
b. scrotal edema
c. tumor
d. hydrocele

Feb 2019
1. 13 years GCSE exam dysuria headache nausea?
a. Trimethoprim and tack culture
b. Broad spectrum antibiotic and go to exam
c. Miss the exam and admit
2. Question about intermittent inguinal swelling increase by cough?
Inguinal hernia
3. Girl 3 years Labial fusion .. management?
a. Do 17 ohp
b. surgery referral
c. topical estrogen
d. reassurance (sop 401)
4. case describing intermittent inguinal swelling increase by cough?
Inguinal hernia

P a g e | 43
Nephrology-Urology

Oct 2018
1. EMQ:
UTI
ACUTE GN
CHRONIC RANAL FAILURE
HUS
RTA
A. baby 3 months high grade fever and weight loss- hematuria and
proteinuria at the time of infection, CBC HB 10.9, WBC=20
UTI
B. 2 weeks post skin infection ... hematuria-proteinuria
acute glomerulonephritis
C. 14- year old girl with delayed puberty and short stature Proteinuria +++
CBC WITH NORMOCYTIC ANEMIA Creatinine VERY HIGH
Chronic renal failure
2. Q from other EMQ about vomiting:
2-3 months old weight loss fever lethargic -- UTI
3. Boy 9 months, his testes in inguinal canal and can be milked down to
scrotum??so it retractile
a. reassurance
b. surgical referral
c. review after 3 months
4. Infant with fever vomiting bloody diarrhoea with increased counts
(picture of septic shock) what will u do next ?
a. stool culture
b. USG abdomen
c. Abd X-ray
d. urea and electrolyte
5.UNDESCENDED TESTES at 9 months old
refer to surgery

P a g e | 44
Nephrology-Urology

June 2018
1. Child has fever no focus with urine analysis negative for bacteria, glucose,
protein and nitrite, WBC = 20 increased
a. viral infection
b. TB infection
c. UTI
d. bacterial vaginitis

2. 2-months baby with UTI klebsiella on antibiotic not improving for 3 days:
a. Do US now and MCUG at 6wk and DMSA at 6month
b. Do US now and MCUG and DMSA at 6 weeks DMSA after 6month

3. A boy polydipsia and polyurea with urine osmolarity 200 and specific
gravity 1008, he is well
a. habitual polydipsia
b. DI nephrogenic
c. DI central
d. DM

4. child generalized oedema ascites what best investigation to do nephrotic


a. serum albumin (nephrotic syndrome)
b. urine analysis
c. ASO titre

P a g e | 45
Nephrology-Urology

Feb 2018
1.4-month old has E. coli UTI Which responded to
treatment, what to do next?
a.U/S after 6 wks
b.MCUG
c.DMSA

2. 2 or 3 year old girl with hypertension ,abdominal pain and


fever, normal abdominal examination?
a.Wilms tumor
b.Dysplastic kidney
c.Renal artery stenosis

3. 12 year old presented with acute sudden onset of vomiting ,pain in


the hemiscrotum and erythema, diagnosis?
a. Testicular torsion
b. Incarcerated Inguinal hernia
c. Torsion hydatid cyst of Morgagni

4. When to refer to pediatric surgery from the following?


a. Undescended testis at 6 month
b. Paraumblical hernia at 8 month
c. Non retractable foreskin at 3 years

P a g e | 46
Nephrology-Urology

Oct 2017
1.6- Child with dysuria, developed lately 2’ry enuresis, us done showing renal
stone. Dipstick Proteinuria+++ , Blood + , Leukocyte + ,Nitrate -
What is the Cause?
a. Hyperparathyroidism
b. Cystionuria
c. Distal Renal Tubular acidosis
d. Proteus UTI ?
e. E. Coli UTI

2. child with fever and coryza 1 day, Dipstick , Protein + , Blood+++ ,


Leukocyte + Nitrate -ve, What to do ?
a. repeat dipstick after 1 week
b. give antibiotic for UTI
c. refer to pediatric clinic
d. measure urea, electrolyte, creatinine
e. measure albumin

3. (12 or 16 year) boy presented with sudden left severe testicular pain with
swelling, he just came from football what is your diagnosis?
a. Testicular torsion
b. Haematocele
c. Torsion of hydrated cyst morgagni
d. epidydmitis

P a g e | 47
Nephrology-Urology

4. Girl 9 years with 2’ry enuresis and-ve urinalysis, what is the 1st
investigation?
a. plasma and urine osmolarity
b. specific gravity
c. u/s post voiding

June 2017
1. Protenuria & haematuria after throat infection ..
Post strept glomerulonephritis

Feb 2017
1. Most serious painless complication of incarcerated inguinal hernia
a. intestinal gangrene
b. testicular atrophy
c. intestinal stricture
d. femoral vein thrombosis.

2. A 2 months baby with UTI and culture shows E.coli. investigation?


U.S within 6 weeks
3. Unilateral flank pain + radiating to groin + urine analysis shows RBCs
above 100 & wbcs 5 + history of fever I think and few small cervical lymph
nodes
a. renal stone (Calculus)
b. HUS
c. PSGN
d. PYELONEPHRITIS
e. HSP

P a g e | 48
Nephrology-Urology

4. A case of infant , feverish, Can't remember details


UTI

5. A case 10 years with painful red scrotal


Testicular torsion

6. A case of UTI with dipstick showing +ve leukocytes and nitrites , a febrile
and good general status … best treatment ?
Trimethoprime? Revise guidelines
7. A boy fall off his bike the previous day. Now presents with asymmetrical
scrotal swelling.
a. Hydrocele
b. testicular torsion
c. scrotal hematoma
d. varicocele.

Oct 2016
1. 3 years old child not sleeping well at night because of umbilical hernia
pain. Your action:
a. Reduce and fix it.
b. Get surgical opinion. ??
c. Reassurance. ??

June 2016
1. 10 yrs old boy with eenuresis camping
Desmopressin

P a g e | 49
Nephrology-Urology

June 2015
1. severe sudden abdominal pain with vomiting, associated with pain in
left testicle diagnosis?
a. LF torsion of tests
b. torsion of Morgagni

2. nocturnal enuresis, had abnormality in renal scan, what he had?


a. duplex kidney
b. cyst kidney
c. horseshoe kidney child

Oct 2013
1. A small child with painless, soft, cystic scrotal swelling. Both testes are
palpable. What is your possible diagnosis?
a. Hydrocele
b. Torsion testes
c. Incarcerated inguinal hernia

2. A 3 years old child with fever, vomiting, and dysuria. Abdominal US


revealed renal stones. Urine routine showed leukocytes 2+, protein +,
and trace blood. What is your possible diagnosis?

a. Nephrocalcinosis
b. Renal tubular acidosis
c. Proteus UTI
3. A 2 years old child with generalized oedema and rash over the chest and
abdomen. What is the next investigation to do?
Urine dipstick

P a g e | 50
Nephrology-Urology

4. A child with bloody diarrhea, anemia, and poor urine output. What is the
next investigation to do?
a. Serum complement levels
b. Serum creatinine level
c. Serum urea level

May be creatin for triad of HUS ???

June 2011
1. A child is diagnosed to have Nephrotic syndrome and started on
prednisolone. After 2 days he presents with swollen face and testies
What is the likely explanation you will give to the mother?
a. He will have furosemide
b. He will have IV albumin
c. Its too early for prednisolone to act
d. Refer to nephrologist

2004-2005-2006
1. 12-yrs-old boy who is has normal XY karyotype and has no testis in
scrotum and us scan didn’t find any as well. He has normal length penis.
What investigation will make you get a diagnosis?
a. Laparoscopy
b. Serum testosterone
c. CT abdomen
d. B-HC

P a g e | 51
Neurology

P a g e | 52
Neurology

Oct 2020
1.EMQ:
Benign childhood epilepsy (Rolandic epilepsy)
reflex anoxic seizures
complex partial seizure
infantile spasm
febrile convolution
rigor
Night terrors
Absence seizure
A. Child have abnormal movement in one side of face with abnormal eye
movement drooling swallowing at night or during sleep and feeling tired
and sleep (post ictal).
Benign childhood epilepsy (Rolandic epilepsy)
B. Scenario about toddler got pale and loss of consciousness for seconds after
hit his head by table with muscle twitching.
reflex anoxic seizures
C. Scenario about feverish child 39c with abnormal body movement that
stopped when you touch him.
Rigor

2. 8-year-old child with 1st attack of convulsion for 5 mint now he is ok. He
had Hx of febrile convulsion when he was 3 years, mother ask if he need
anticonvulsant therapy, ask about indication to start AED?

a. Family Hx of epilepsy
b. Abnormal EEG
c. Further occurrence of convulsion
d. The AED protect against brain damage
e. Deterioration of school performance

P a g e | 53
Neurology

3. Case of unilateral facial palsy he can wrinkle his forehead, he


had mild response to steroids (mean case of upper motor
neuron lesion). Ask about next step?
a. Refer to neurology
b. CT brain
c. MRI brain

4. 8-months baby with EEG (chaotic appearance) ask about Rx?


a. Carbamazepine
b. Lamotrigine no steroids in options
c. Vigabatrin.

Feb 2020
1. Patient came with vomiting and examination has petechia received
metoclopramide then has neck stiffness and rolled eye
a. oculogyric crisis
b. Rolandic seizures
c. Myoclonic seizure

2. Boy 7 years came with Absence seizure and 3 Spike wave /Sec and
become Staring for few seconds multiple times every day. What to give?
a. lamotrigine
b. Sodium Valproate
c. Phenobarbitone
3. 8 years girl came with morning headache and poor sleep and new onset
Squint, What is the diagnosis?
a. Space occupying lesion
b. Meningitis
c. migrane

P a g e | 54
Neurology

4. Child admitted in the ward with bronchiolitis he has fever and convulsion
for 2 minutes and nurse give him O2 and RBS was 6 mmol/L and saturation
94% , what is your next action?
a. observes
b. Rectal midazolam
c. Insert Cannula
d. Buccal midazolam

Oct 2019
1.EMQ:
a. Syncopy
b. Complex partial
c. Simple partial
d. Day dreaming
e. Absence seizure

A. boy looks frightened, pale ,repetitive swollwing for 15-30sec after which
he is tired
Complex partial
B. girl at the assembly lost her consciousness, Pale and twitches after she
regain consciousness she is confused
Syncopy
C. child noticed at school by the teacher go still for 1-3 minutes
Day dreaming

2.what is the most important indication to do CT head for patient with


headache:
a. waking up with vomiting
b. increase headache frequency
c. increase headache severity

P a g e | 55
Neurology

3.patient had convulsion took rectal diazepam, IV lorazepam by paramedic


What to give:
a. phenytoin bolus
b. phenytoin infusion
c. phenobarbitone bolus

June 2019
1. migraine in child:
a. Usually unilateral
b. Family history
c. Rare under 5 years
d. Common visual aura

2. rolandic epilepsy drooling and gurgling during sleep parasia in jaw


a. need EEG
b. ca

3. case about history of meningitis with convulsion for 10 min first ttt?
a. iv lorazepam
b. iv antibiotic
c. iv saline
d. antibiotic +acyclovir

4. infant with URTI feverish 38.5, with history of febrile convulsion but more
than 5 minutes( 7 minutes ) what best advise to tell her, mother if it occurs
again?
a. Buccal midazolam
5. another case of febrile convlesion,,recurent ,,known case
a. oral brufen
b. regular paracetamol
c. plenty of drink

P a g e | 56
Neurology

Feb 2019
1. Patient complain of body and leg pain, on exam he has swelling on lumber
region? Spinal tumor ??
2. weakness on L side of face and was able to wrinkle that side of the face.
Improved slightly with steroids?
a. Discharge with GP follow up
b. CT
c. MRI
d. Neurologist referral

3. Headache neck rigidity?


Subarachnoid haemorrage

4. 13 years old girl moaning and closing her eyes throughout?


Pseudo-seizure

5. 9-year girl have family issues (step father come to home), argue with her
mother and doesn’t want to get out of bed, then when she got out turned
pale fall down with twitching of hand, then wake up after 2 minutes and
she don’t remember except she was dizzy?
a. reflex anoxic
b. complex partial
c. pseudo seizure
d. syncope

6. EMQ
A. Baby with seizure stop by touch --- Rigor
B. Crying+ apnea--- Reflex anoxic

P a g e | 57
Neurology

Oct 2018
1. EMQ
A. ABSENCE SIEZURE—ETHUXAMIDE
B. REFLEX ANOXIC SIESURES-REASSURANCE and education
C. FEBRILE CONVULSIONS- reassurance and explanation
2. A 15-years old boy suffered injury during rugby ball Match, unconscious
for 5-min, amnesia for 5 min only regain conscious. What to do next?
a. go for the match
b. play if CT normal
c. play after full recovery
3. Girl with epilepsy what is your plan in school??
June 2018
1. EMQ
a. brain tumor
b. cluster headache
c. tension headache
d. migraine

A. child with bilateral headache banding in nature not interfere with activity
tension headache
B. child with long history of headache now become more severe not relief by
paracetamol and ibuprofen and associated with morning vomiting and
ataxia --- brain tumor
C. child long history of headache some time associated with abdominal pain
and vomiting--- migraine
3. Toddler went with her mother to shopping she want to buy sweet but the
mother refused then she cry too much cyanosed then seizing
a. vagal attack
b. expiratory anxiety apnea breath holding spell
c. partial complex
d. pseudo-seizure

P a g e | 58
Neurology

Feb 2018
1.EMQ
a. Generalized tonic clinic epilepsy
b. Hypocalcemia
c. Hypoglycemia
d. Meningitis
e. Tuberous sclerosis
f. Sturge Weber

A. Baby flexes his legs and cries and becomes pale, his mother has
hypertension--- Tuberous sclerosis
B. Girl developed fits, she was treated by carbamazepine, had temp 38.7 &
vomiting since-last night, has blotchy rapid spreading rash.Meningitis
C. Asian boy with fits, his mother has a limp and previous fracture
Hypocalcemia

2. 7-year-old boy with 3 spike and wave ttt ?


a. Lamotigine
b. Sodium valproate
*no ethuxamide in the options
3. An obese girl with headache. Examination show bilateral papilledema, she
can’t abduct her eye, What’s the next step?
a. Cranial MRI
b. LP with manometry

4. Case of febrile convulsions he was 2 years I think turning blue mother is


nurse father teacher, advise in next attack?
a. Buccal midazolam
b. Ambulance
c. Rt position

P a g e | 59
Neurology

5. case of headache and 6th nerve palsy asking about diagnosis?


a. Space occupying lesion

6. one patient having convulsion for 20 min, at home receive midazolam at


home then lorazepam in ambulance. what is your next step?
a. phenytoin bullous
b. phenytoin infusion
c. Lorazepam

Oct 2017
1. patient with history of headache which of the following is significant
alarm?
a. Increase in severity
b. Increase in frequency
c. Wake from sleep with vomiting

2. scenario about eye lid ptosis, when asked to raise eye brow he did it given
steroid for 2 days and improve in 1st week , presented to you?
a. discharge and if urgent symptom comeback
b. refer to neurologist
c. do CT
d. refer to family doctor

3. 9-years old girl have family issues (step father come to home), argue with
her mother and doesn’t want to get out of bed, then when she got out
turned pale fall down with twitching of hand, then wake up after
2 minutes and she don’t remember except she was dizzy?
a. reflex anoxic
b. complex partial
c. pseudo-seizure
d. syncope

P a g e | 60
Neurology

4. EMQ
a. IV lorazepam
b. IV phenobarbitone
c. Rectal ibuprofen
d. PR paraldehyde
e. oral phenobarbitone
f. Vigabatrin
g. Carbamazepine
h. oral Na Valproate

A. scenario of a neonate and develop frequent fits -- Phenobarbitone


B. scenario with temp of 39’c and develop seizure half an-hour , given 5 mg
diazepam PR by paramedics, still seizing? IV lorazepam
C. 2 years-old with generalized tonic-clonic fits. 3 episodes in the last
1 month- Oral Valproate

5. scenario with hx of febrile convulsion, what will increase risk of


recurrence?
a. prolong seizure
b. family hx of epilepsy
c. high fever

June 2017
1. case about treatment of the EEG chaotic appearance
infantile spasm
2. child has fever 39 and his limbs shake stop when you hold them
Rigors
3. child 4 months with abnormal head shape prominent RT parietal and LT
occipital
Plagiocephaly

P a g e | 61
Neurology

4. Ramsay hunt syndrome...type of lesion


Lower motor neuron lesion of 7th cranial nerve with herpes simplex
infection
5. Case of absence seizures and ask about EEG finding?
3 spikes & waves per second

6. tension headache and take paracetamol and morphine no improvement


what to do next?
Stop medications (Medication overuse headache)

Feb 2017
1. Boy with history of measles before 10 days, convulsed today for more than
10 mins, relieved after buccal midazolam but he is still unconscious
2-hours post ictal with no focal neurological signs. What to do ?
CT or MRI

2. A boy at ER, always goes to school alone, presented with GCS 12/15 and
neck stiffness.
a. subarachnoid hge
b. extradural hge
c. cerebral artery

3. A male adolescent presented to ER with coma, Alcohol smell from his


mouth, facial ecchymosis and bruising, unequal pupils... he was alone in
the street. (Seems to be severely hit by thieves or something).
What is most probable cranial hemorrhage ?
a. subarachnoid
b. extradural
c. intra ventricular

P a g e | 62
Neurology

4. Adolescent girl. Presented with her friends who were in a party.They say
she suddenly began to shout and make abnormal movements or
something then collapsed on the bed. She is now non-oriented, closing
her eyes and moaning.
a. Postictal
b. MDMA toxicity
c. Pusodosizure
5. A case inpatient, convulsing since 2 mins, feverish. Best next step?
a. Reassurance
b. insert cannula ?
c. buccal midazolam
d. antipyretic
6. EMQ:
A. A case with diffuse headache without vomiting Tension headache
B. A case with headache, vomiting, abdominal pain Migraine
C. A case with headache mainly at the morning and vomiting Brain tumor
June 2016
1. EMQ: Sample paper
A. day dreaming
B. benign vertigo ( pale, tearing blinking)
C. complex partial ( aura)

2. protrusion of the eye and no kidney problem


neuroblastoma
3. petit mal epilepsy ECG
3 spikes and waves per second

4. scenario about headache = tension

P a g e | 63
Neurology

June 2015
1. 8yrs came with ataxia and convulsion Best invistigation?
a. CT
b. MRI
c. EEG

Oct 2013
1. A child with ptosis, meiosis, and anhydrosis. What is your possible
diagnosis?
Horner’s syndrome

Feb 2011
1. Which of the following diagnoses is the most likely in a 3-year-old boy who
has epilepsy and, on examination, has numerous depigmented macules
and two café-au-lait spots?
a. Tuberous sclerosis
b. Neurofibromatosis 1
c. Ataxia telangiectasia
d. Incontinentia pigmenti
e. Sturge–Weber syndrome

P a g e | 64
Neurology

2004-2005-2006
1. a 14year old girl who is upset with her mum as mum's boyfriend has
moved into their house. she doesn't come down for breakfast, so mum
goes to call her. mum finds her sleeping on her bed. girl starts shouting in
her mum and suddenly gets up, she goes pale and falls down
unresponsive, mum notices that fingers of one hand r moving, girl gets up
after few seconds and doesn't remember anything other than feeling
dizzy. the one best reason to explain this

a. reflex anoxic seizures


b. centerotemporal epilepsy
c. psuedo seizure
d. hypoglycaemia

2. A girl has midline sacral cystic lesion. And investigation necessary is?
a. cranial ultrasound - searching for intracranial vascular malformation
b. renal ultrasound karyotype
c. excision with skin cover

3. 12-yr-old with h/o freighting attacks in school,mother had similar


problem and didn’t have it after adolescence, maternal uncle died early
a. epilepsy
b. benign vertigo
c. sick sinus

P a g e | 65
GIT
Hepatology
Nutrition

P a g e | 66
GIT -Hepatology-Nutrition

Oct 2020
1. EMQ: GIT
a. Colonoscopy with biopsy
b. upper GIT endoscopy with biopsy
c. anti tissue transglutaminase
d. abdominal US
e. barium swallow
f. abdominal CT

All scenarios have same family history of father has peptic ulcer and other
member with crohn disease (I think aunt?).

A. 12-years child with weight loss and diarrhea and RIF pain and
tenderness. Colonoscopy with biopsy

B. 2-years child with pallor and faltering growth.


anti tissue transglutaminase

C. 14 years girls with epigastric pain which awaken her from sleep at
night.
upper GIT endoscopy with biopsy

2. How to Dx overweight?
a. Weight 99.6%.
b. BMI = 99.6%
c. BMI = 91%
d. Waist circumference > 95% centile.
e. BMI= 27

3. Child with constipation with abdominal pain and faecal impaction


up to umbilicus. ask about management?

a. Admit and do dis-impaction in hospital


b. Prescribe laxative and send him home.
c. Oral dis-impaction at home with close review.

P a g e | 67
GIT -Hepatology-Nutrition

4. 6-weeks-old boy from Nigerian parents brought by his mother with


poor feeding and lethargy with jaundice and dark colour urine the
mother has changed his feeding to formula milk due to poor weight
gain. What initial investigation for Dx?

a. Urine culture.
b. Conjugated bilirubin.
c. Electrophoresis.
d. G6PD level.

5. Baby on breast milk, mother changed to formula milk after starting


work,after then the child developed skin allergy and not gaining
weight (case of cow milk protein allergy) what to do?

a. Continue same formula


b. Extensive hydrolysed formula milk
c. Alginate
d. Emollients

6. 11-years boy with obesity BMI = 99% centile with small penis
(surrounding fat pad) no other abnormalities, normal puberty and
the mother said he has normal appetite. The family is concerned
about him.
What to do to assess his condition?

a. DNA analysis
b. Diet and activity assessment
c. Abdominal or scrotal US
d. Glucose tolerance test
e. Thyroid profile

P a g e | 68
GIT -Hepatology-Nutrition

7. Vegetarian mother with exclusive breast feeding of her baby for


1- year the child has poor appetite. Which nutrient is deficient?

a. Iron NO B12 in options


b. Calcium
c. Zinc
d. Folic acid.

8. 3-years-old boy came with yellow coloured skin and white sclera,
mother is vegetarian and she said that he likes to drink too much
squash juice,
what the Dx?

a. G6pd
b. Thalassemia
c. Carotenemia

9. Scenario about child with jaundice and hepatitis A ask about best
test to assess the severity of hepatitis?

a. High liver enzymes


b. ALP
c. HBe Ag +ve
d. Prolonged PT

10. 18-months child came with acute (24h) deterioration, vomiting and
bloody diarrhea and pallor, investigations: PH 7.2, BE -10 , HB 12
(within normal range given) , Wbc 20 , Plat 145
What to do initially?

a. Abd us
b. Abd x-ray
c. Urea & E
d. Stool c&s

P a g e | 69
GIT -Hepatology-Nutrition

Feb 2020
1. EMQ-GIT:
a. gastroenteritis
b. UTI
c. malrotation
d. pyloric stenosis
e. overfeeding
f. constipation

all takes 150 ml per kg

A. 7 weeks increasing non bilious vomiting, lethargy fever 38.5 C


not well, not feeding well
UTI

B. 7 weeks well baby 48hrs intermitted bilious vomiting, baby


Tachycardia, capillary refill time 4 Sec
malrotation
C. 7 weeks old baby boy with non-bilious vomiting, allows hungry
pyloric stenosis

2. Child 6 years in the school reduce fluid intake with soiling and
teased at school because his odor What is the initial management?

a. Increase fluid intake


b. laxative
c. refer to CAMHS
3. 10 years old Tall Boy with advance bone 1year his height 90% and
wt. 98% blood pressure 91 %and his sister and mother were normal
weight No other abnormality and not started puberty after
2 months weight and height still increasing

a. simple obesity
b. Cushing
c. Adrenal tumor

P a g e | 70
GIT -Hepatology-Nutrition

4. Baby 5 months old started formula feeding then developed Ecezema


and Diarrhea --What to do?

a. Extensive hydrolyzed formula


b. Soya milk formula
c. Lactose free formula
d. Amino acid formula
5. Female Child came with recurrent abdominal pain high ESR and
arthralgia, Diarrhea with mucous and mother with vitiligo What is
the most diagnostic investigation?

a. colonoscopy
b. Celiac screen (TTGA)
c. Barium enema
6. 11 years boy Obese with pigmentation in the neck and glycosuria
and RBS 7mmo/L and with mild elevated liver enzymes
What is the diagnosis?

a.Fatty nonalcoholic liver disease


b. Hepatitis
c.Type 2 DM
d.Cushing disease
7. 3 years Child came diarrhea and undigested food particles and his
wt. on 9th centile

a. refer to dietitian
b. Reassurance
c. Antispasmodic

P a g e | 71
GIT -Hepatology-Nutrition

OCT 2019
1.EMQ:
a. Giardiasis
b. Rota virus
c. cow milk
d. Celiac
A. 3year old child well apart from having diarrhea with food particles
toddler diarrhea

B. baby on breast feeding, mum shifted to formula and he is well and


thriving has eczema, developed watery diarrhea 24 hours ago
Rota virus

C. baby ,mum changed to formula under pressure of her family


diagnosed with colic at 5 weeks of age,has nappy rash,eczema and
red scalp
CMPA

2.somali mother want to breast fed her baby what vitamin deficiency
will be the most :
a.vit D
b.vitA.
c.vit K.
d.vit C

3. baby with jaundice, coombs negative,total bilrubin 120


Conjucated 90 what’s the diagnosis:

a.bliary atresia
b.galactosemia

4.baby with vomiting,plyoric stenosis what’s the most clinical sign:


a.lost weight
b.varcious hungery
c.non bilious vomiting

P a g e | 72
GIT -Hepatology-Nutrition

5.female 13 years old,down Syndrome had abdominal


bloating,diarrhoea,had her thyroid function checked last year was
normal ,her mother abandoned her:
a. celiac
b. chron’s
c. Hypothyroidism

6. Girl 10 year with all males in the family diseased, and a dead uncle
a. Check her cholesterol level
b. Wait until she is competent

7.edemtous child malnourished What to give:


a.cow formula

JUNE 2019
1.EMQ
3 scenarios about bloody diarrhea (same as TAS sample paper)

A. low Hb and platelets


HUS
B. Painless bloody diarrhea + well child
colonic polyp
C. 8 years old + 6 months history of bloody diarrhea + lethargy
weight loss
chrons

2. 22monthChild healthy eating well on 25 centile I think , diarrheal


3-6 time per day episodes with food particles:

a. Giardiasis
b. Toddlers diarrhea
c. Celiac

3. inspiratory stridor increase during feed and sleep and apnea


(SINCE BIRTH)
a. laryngomalacia
b. GOR

P a g e | 73
GIT -Hepatology-Nutrition

4. Constipation with soiling in 5 years child, since start school also at


Home
a. fear of school toilet
b. Constipation
c. sepration anaxity
d. Child abuse

5. Scratch around anus at night /red margin around anus painful


Defecation
a. theadworms
b. Sexual abuse
c. groupA strep
d. staph

6. 11-years-old girl, breast stage 3 ,obesity screening, weight and height


(>98%) centile BMI 91% BMI ( 30 ) What is the best management?

a. Weight management and exercise & life style


b. Statins
c. Lipid profile and liver function tests

FEB 2019
1. 3 weeks birth weight 3.4 current 3.7 feeding 125 ml 6 time
per day with vomiting Overfeeding =TI 202ML/KG/DAY
a. Overfeeding
b. GOR

2. 3rd day baby bilious vomiting


a. Admit and investigate

3. Worse sign of hepatitis?


PT prolonged sop p.412

P a g e | 74
GIT -Hepatology-Nutrition

4. severity of dehydration?
a. Dry mucous membrane
b. weight loss wither it is 5% or 10%
c. prolonged capillary refills time
d. skin turgor
e. hypotension

5. A mother is having difficulty feeding her child and believes that you
cannot help her?
Breast feeding team
6. Obese with micro-penis
Normal obesity

7. EMQ Choose suitable investigation


a. CRP
b. ESR
c. Amylase
d. fecal elastase
e. Anti-tissue transglutaminase
f. H pylori stool
g. H pylori serology

All diabetic patient and complain of

A. scenario with peri-umbilical pain and abdominal distention,


constipation .
Anti-tissue transglutaminase
B. scenario with intermittent epigastric pain, poor apetite, wake at
Night, weight loss
H pylori stool
C. scenario with right iliac fossa pain and mass, diarhea for 6 weeks
fecal calprotectin ?

P a g e | 75
GIT -Hepatology-Nutrition

OCT 2018
1. EMQ:
vomiting no diarrhea
A. weight loss 6 weeks hungrey
pyloric stenosis
B. 3-months old weight loss fever lethargic
UTI
C. recurrent bilious vomiting
malrotation

2. Child with obesity what is the cause to be treatable :

a. height 25centile
b. MICROPENIS
c. bradypnea
d. FRAGILE X
e. poor school performance

3. Newborn discharged with no resuscitation needed. Chest x-ray


showed gas in abdomen. At 6 hrs developed choking while feeding,
at present maintaining saturation with o2, crackles in chest.
mother was polyhydrominous
a. H type TEF
b. esophageal atresia with distal fistula
c. diaphragmatic hernia

4. Neonatal conjugated hyperbilirubinemia but enzymes normal

a. breast milk jaundice.


b. neonatal hepatitis
c. biliary atresia

P a g e | 76
GIT -Hepatology-Nutrition

June 2018
1.EMQ:
a. duodenal atresia
b. pyloric stenosis
c. Hirschsprung Disease
d. Volvulus
e. Intussusception

A. baby delivered at home, mother told green vomitus within 18 hr,


baby was hypotonia
duodenal atresia; hypotonic child = Down syndrome

B. baby around 4 week persists vomit and look hungry her vomit
coming after milk feed suddenly traverse across their room.
pyloric stenosis; with projectile vomiting & hungry

C. an infant with vomiting, on and off abdominal pain loose bowel


motion (not sure)
intussusception (not sure)

2. baby with tachypnea persisted vomiting at 12-hour old CXR no air


bubble in the abdomen

a. trachea-esophageal fistula,
b. proximal esophageal atresia without fistula

3. A case of child with chronic constipation for 6-months fecal mass


reach till abdomen

a. Admit for oral dis impaction


b. Oral disimpaction at home with review
c. review within one week
d. start laxative

P a g e | 77
GIT -Hepatology-Nutrition

4. infant FTT with generalized edema and underweight look


dehydrated, miserable

a. milk-based formula
b. cow milk formula Kwashiorkor
c. NS bolus
d. soya formula
e. albumin infusion

5. preterm baby now 4-month, history of 10-day diarrhea mother


switch breast to formula due to her work baby has eczema

a. hydrolysate formula
b. preterm formula
c. soya formula
d. lactose free

6. child eating variety of food but can't gain weight, he has low Hb,
abdomen is protuberant.

a. coeliac
b. rickets

7. how to diagnose the sure persistent vomiting sign pyloric stenosis


a. Hypochloremia
b. hypokalemia

8. A girl 14- year delay puberty has of abdominal pain loose motion 3-7
time the mothers has vitiligo, what investigation to do?

a. colonoscopy
b. marker of celiac disease

P a g e | 78
GIT -Hepatology-Nutrition

9. a child 8 month brought to the hospital for investigation due to


jaundice found to have yellow hand chest, and abdomen and white
eyes mother like to drink vegetable purred juice pale face

a. carotenemia
b. Gilbert
c. breast milk jaundice

Feb 2018
1.EMQ
a. Upper GIT endoscopy with biopsy
b. Colonoscopy with biopsy
c. IgA anti tissue transglutaminase
d. U/S abd
e. Serum amylase

All has a Father with duodenal ulcer, aunt with croh's

A. 14-year-old with diarrhea and blood and weight loss RIF


Colonoscopy with biopsy

B. 3-year-old no weight gain miserable with vomiting and abdominal


distension.
IgA anti tissue transglutaminase

C. 14-year-old with upper abdominal pain and wake in night.


Upper GIT endoscopy with biopsy

2. One boy with liver enzymes high and pigmentation of skin


glucosuria obese, gynecomastia, RBS normal diagnosis?

a. Type 2 DM
b. Fatty non-alcoholic liver
c. Cushing

P a g e | 79
GIT -Hepatology-Nutrition

3. 3-month-old baby girl developed blood in stools after her mother


started work and gave her bottle feeds, diagnosis?
Cow milk allergy

4. A boy passing stool without feeling, teased at school because of this,


history of neglect, ttt?

a. CAMHS
b. Laxative
c. Star chart

2.EMQ
a. Hirschsprung
b. Intussusception
c. Pyloric stenosis
d. Gastroenteritis
e. Ulcerative colitis

1. Baby on amoxicillin, has fever and vomiting 6 times crying, which


contained blood in the last two episodes
Gastroenteritis
2. Baby can’t gain weight, has non bilious vomiting every
time he feeds, he is always hungry.
Pyloric stenosis
3. Down with bilious vomiting.
Hirschsprung

P a g e | 80
GIT -Hepatology-Nutrition

Oct 2017
1. EMQ
List of Causes of Gastroentestinal bleeding (Same as Sample paper TAS)
a. Anal fissure
b. Crohns
c. Angiodysplasia
d. Colonic Polyp
e. Cow’ s milk Allergy
f. Diverticulitis
g. HUS
h. HSP
i. Sexual abuse
j. Yersenia

A. Scenario 2 day hx of bloody diarrhea and fever - non blanching Rash


on his trunk, HGb 7 , Plt 20 , WBC15 , albumin 38 .
HUS

B. Scenario 8-yrs boy- 6 month hx of bloody diarrhea ass. Wight


Loss and lethargy. HGB9 , Plt 524 ( highesr , crp )
Crohns

C. Scenario 12 yr - intermittent painless bleed with normal stool, no


diagnostic clinical feature, normal labs.
Conlonic Polyp

2. 7-months old with coryza and cough given amoxicillin next day
developed vomiting and profuse diarrhoea and managed with oral
rehydration what is the Cause?
a. Rotavirus
b. RSV
c. Antibiotic induced enteritis

P a g e | 81
GIT -Hepatology-Nutrition

3. scenario of 5-months vomiting, blood in stool, breast feeding


what to do?

a. lactose free formula


b. extensive hydrolysed formula
c. soya formula
d. amino acid formula

4. EMQ
Choose suitable investigation
a. CRP and ESR
b. Amylase
c. fecal elastase
d. Anti-tissue transglutaminase
e. H pylori stool
f. H pylori serology

All diabetic patient and complain of


A. scenario with peri-umbilical pain and abdominal distention,
Constipation
Anti-tissue transglutaminase

B. scenario with intermittent epigastric pain, poor appetite, wake at


Night, weight loss
H pylori stool

C. scenario with right iliac fossa pain and mass, diarrhea for 6 weeks
CRP& ESR

5. 18-months breast feeding for 6-month then mixed diet. passing


stool with blood on surface, stool is small and hard for 6 month,
what to do?

a. dietary management
b. rectal myometry

P a g e | 82
GIT -Hepatology-Nutrition

June 2017
1. child is over weight?
a. weight ~99th centile
b. BMI ~91th centile
c. weight ~45kg
2. child with macrocytic anaemia and rickets and bruises and
dry skin what is the operation lead to this?
Ileal resection /Short bowel syndrome
3. First investigation in celiac disease
IgA-antitissue transglutaminase

4. child vomiting And hypokalmic hypo chlormic alkalosis


pyloric stenosis
5. Case of neonatal jaundice & pale stool .. Most likely biliary atresia ..
What is first investigation?
US or HIDA scan
6. case with foul smelling stool undigested vegetable will child and
open her bowel 3 times daily grow on 9th centile ...management?
Reassurance
7. Definition of obesity .. option obesity diagnosis
a. Wt > 99th centile
b. BMI > 91st centile

8. Child presented with pallor and abdominal distention what


investigation to do? Trans tissue glutaminase ig-A

9. 6 weeks infant with jaundice and clay stool INR 5 what to do?
a. IV vit K
b. oral vit K
c. IM vit K
d. fresh frozen plasma
e. cryoprecipitate

P a g e | 83
GIT -Hepatology-Nutrition

10. case with soiling and since he starts school and start to refuse also
to do at home.
afraid of school toilet

Feb 2017
1. Baby changed from breast to formula then developed severe
eczema and diarrhea and lost weight.
Cow milk Protein Allergy
2. Case with history of mild eczema but good feeding, then developed
diarrhea in last 24 hours.
rotavirus
3. 2.5 years with loose stools with undigested particles.
Toddler diarrhea

4. A case with pale, loose stools


Biliary atresia

5. A baby changed from breast to formula the lost weight significantly.


Next?
start hydrolysed formula

Oct 2016
1. EMQ1: Nutritional deficiency
a. Vitamin B6
b. Vitamin B12
c. Vitamin K
d. Vitamin D
e. Vitamin C
f. Zink

P a g e | 84
GIT -Hepatology-Nutrition

1. Neonate with seizure. B6


2. Neonate with gum bleeding. Vit K
3. Neonate with broadening of ulna. Vit D
June 2016
1. EMQ:
Nutrient supplement for patient with:
A. perioral dermatitis = zinc
B. vegetarian diet = b12
C. obese lazy dietary management not improved they give him drug
( i cant remmeber the name trade name ) Vit D

2. diarrhea polyhydramnios >> bicarbonate 28, ph 7.5, cl 89 low Na


low,
cl loosing enteropathy

3. management in kwashiorkor

4. EMQ investigations:
A. 18month constipation failure to thrive,
sweat test (cystic fibrosis)
B. 3-month vomiting metabolic alkalosis
US Pyloric stenosis

June 2015
1. child had obesity, dyslipidaemia, hyperglycaemia, what the risk
factor for her when she was born:
a. preterm 2.5 kg @ 1 yr old 7.5 kg
b. preterm 2.5 kg @ 1 yr old 10 kg
c. term 2.5 kg @ 1yr old 10 kg
d. term 2.5 kg @ 1 yr old 7.5 kg
e. term 4.5 kg @ 1 yr old 10 kg

P a g e | 85
GIT -Hepatology-Nutrition

Oct 2013
1. Which of the following is most likely to confirm diagnosis of Pyloric
stenosis;
a. Hypochloremia ??
b. Hypokalemia
c. Non bilious vomiting
d. Metabolic acidosis
e. Increased urea

2. A child with constipation and impacted feces up to the umbilicus.


What you will do?

a. Start laxative for disimpaction and review


b. Enema to relieve impaction
c. Admit to the ward for disimpassion

3. A 10 years old child with oral painful mouth ulcers and anal fissures.
What is your diagnosis?

a. Crohn’s disease
b. Ulcerative colitis
c. Celiac disease

4. A 6 weeks old infant with jaundice and pale stool. ALT, AST, and
conjugated bilirubin levels are elevated.
What is your immediate
action?

a. Intramuscular vitamin K
b. Intravenous vitamin K
c. Oral vitamin K ??
d. Vitamin A
e. Surgery

P a g e | 86
GIT -Hepatology-Nutrition

Feb 2011
1. 2-years old boy seen by his GP and founded to be malnourished
and oedematous. his weight is >3SD weight for height.
concious but miserable. he sends him to emergency where he got
admitted. urine dipstick was negative, What will u do?

a. Feed orally with cow’s milk formula


b. Feed with soya milk formula
c. Give 1ml/kg furosemide
d. Give human albumin 4.5% 20ml/kg E. 5ml/kg 10% dextrose

2004-2005-2006
1. A child admitted with pyloric stenosis is on IV fluids for 24 hrs. his
bloods results now Na=136, K=3.2. what is next step u will do?

a. inform anaesthetist and urgent surgery


b. continue IV fluids and defer surgery
c. repeat IV fluids and repeat U&E in 4 hrs, if stable then inform
anaesthetist

2. EMQ:
Tell the one best investigation in each case

A. 14-year-old girl complaining of pain in her upper abdomen for


1 month.She avoids eating food. Her father has pancreatitis, her
grandmother has crohn's, her mother has hypothyroidism.
upper GI endoscopy-- peptic ulcer?

B. a child of 6 years diarrhoea for 1 month and has lost 1 kg since 1


month. Her father has pancreatitis, her grandmother has crohn's,her
mother has hypothyroidism
stool examination – Giardiasis?

P a g e | 87
GIT -Hepatology-Nutrition

C. a child of 10 years has diarrhoea for 4 months and has lost weight.
Her father has pancreatitis, her grandmother has crohn's,her
mother has hypothyroidism
Colonoscopy—chron’s

3. A mother has mastitis with sore discharging nipple and is treated


with antibiotics meanwhile the baby who is breast fed

a. Should not be breast fed.


b. should be fed as usual
c. Should express milk and fed later
d. Should wait till infection resolves and baby fed infant formula

4. EMQ
A. A school boy gets diarrhoea and vomiting, no blood after eating
something With some abdominal pain. No fever
E.coli ?
B. Same boy gets appendicitis like pain with blood in stools
Campylobacter
C. Same boy gets diarrhoea and vomiting with outbreak in school and
acute onset
Norwalk virus

5. A boy who has been given elemental diet and steroids for crohns
comes back with a relapse.
What is the main symptom that suggests relapse?

a. Abdominal pain
b. weight loss
c. General ill health

P a g e | 88
Endocrinology
DM

P a g e | 89
Endocrinology-DM

Oct 2020
1. 13-years-old girl with breast pad and scanty pubic hair. asking bout
tanner stage?
a. Breast stage 1 pubic stage 1
b. Breast stage 1 pubic stage 2
c. Breast stage 2 pubic stage 2

2. Boy 12-years-old obese small testis size 2.5 ml, ask about 1st sign of
puberty?
a. Testicular enlargement
b. Axillary hair
c. Penis enlargement

3. Child with ambiguous genitalia with hyponatremia and hyperkalaemia,


Presented with shock, ask about Dx test?
a. 17 hydroxyprogesterone
b. Aldosterone
c. Cortisol.

4. Girl with type 1 DM on basal and bolus insulin regimen with HBA1C 7.9%.
mother found her in in her room with convulsion and sweating
what the cause?
a. Epilepsy
b. Hyperglycaemia
c. Hypoglycaemia

5. 14 years old girl with hypothyroidism due to hashimoto thyroiditis on


levothyroxine 125mcg. She came to you with high TSH and normal T4,
she said she feels good, what to do?
a. Check T3,T4
b. TRH
c.Check thyroid AB
d. Check compliance with her GP

P a g e | 90
Endocrinology-DM

6. 14-years-old girl deteriorated in school performance with weight gain


and cannot do well in gymnastic session and felt tired during day.
she had scanty period. Mother thought she has been bullied at school.
a. Hypothyroidism
b. Cushing
c. OSA
d. Anorexia nervosa

Feb 2020
1. Teenager Female was active and playing Gymnastic she gain wt. with
decrease school performance
a. hypothyroidism
b. Polycystic ovary
c. Obesity
2. Newborn with palpable gonads and hypospadias, What investigation
leads to diagnosis?
a. 17 Hydroxyprogesterone
b. karyotype
c. Abd us
d. Serum Electrolytes

3. 14 years Obese Girl with weight loss and polyurea with family history of
DM, What to do next?
a. Random Capillary blood sugar
b. HA1c
c. Fasting Blood sugar
d. C Peptide
e. Glucose tolerance test

P a g e | 91
Endocrinology-DM

4. 11 years child diabetic was visiting sister in the hospital mother found him
shaking, she measured RBS it was 2.5 mmol/L What best action?
a. give 100 ml sugary fluid
b. piece of Biscuit
c. bar of chocolate
d. IM Glucagon
e. Toast
5. DKA Scenario 14 years old pulse 120/min
What is the initial maintenance?
a. 0.9 sodium 500 ml 20 mmol KCL
b. 0.45 NS 500 ml with 20 mmol KCL

6. Diabetes table: Pt takes Take Long acting in the Night 20 IU and 1 unit for
8 gm carbohydrate before breakfast and 1 unit insulin for each 10 gm
carbs before lunch and 1 unit for 10 gm carbs in the evening.
(The readings are high before bed and evening meal)

a. Increase short acting Lunch and dinner


b. Increase long acting alone
c. increase short acting of lunch and dinner and long acting
d. Increase short acting of dinner
e. Increase short acting of lunch

P a g e | 92
Endocrinology-DM

7. Child came with low Ca ,Phosphate and High Alkaline phosphate and
High PTH, diagnosis?
a. vit d deficiency
b. Hyperparathyroidism
c. CRF
8. Girl 14 years her length 147 cm and her mother 162 cm she ask if she will
be tall as her mom
a. 8 cm increase in the last year
b. Tanner stage B2
c. Tanner A2
d. she has Menarche

9. Baby born with ambiguous genitalia family want to give baby name
What you will till the parents?
a. wait for chromosomal results
b. Give name that can be fit for male or female
c. give male name it is most probable male.
d. Do not assign name and wait for the full investigation results

Oct 2019
1.EMQ:
a. decrease long acting insulin
b. decrease short acting insulin
c. continue same dose insulin with oral rapid acting glucose fluid
d. continue same dose with oral fluids
e. extra short acting insulin

P a g e | 93
Endocrinology-DM

A. pt. diagnosed with DM has voimiting and nauseated RBS 4.6(normal)


continue same dose with oral fluids
B. 14 years girl diagnosed with DM came with morning hyperglycemia
RBS at 2 am 2.9 (low), RBS at 10 am 10-15
decrease long acting insulin

C. child with DM vomited once ,has abdominal pain ,RBS21,ketone ++


extra short acting insulin

2.DM patient developed hypoglycemia after playing match the school did the
RBS and called you to tell it’s 2mmol what advice to give them
a. give him sugary drink in a Cup
b. inject with Glucagon IM
c .call ambulance

3. girl with palpitaion and tremors for 6 weeks TSH 0.02, T4 and T3 are high
what to do:
a. send her to endocrine clinic in 2 weeks
b. Give her carbmizaole
c.give her propyiothiouracil

4. girl had irregular cycle , hirsutism and dark pigmentation what’s the
diagnosis:
a. PCOS
b. cushing

P a g e | 94
Endocrinology-DM

5.patient was diagnosed withDM and his sugar level were controlled for five
years now, his father has HTN what’s the most important thing to tell his
family:
a. he will need insulin for life
b. he will need low carb diet

6. 13 years.pt on the 50th centile for weight and on the 75th centile for
height she is not growing for the last 2years only 4 cm /year
(mid parenteral centile was not on it) her school performance was
acceptable tanner staging B3 P3 what to do:
a. TFT
b. anti-tissue trans glutaminase Ig-A antibodies
c. IGF
d. LH/FSH
e. chromosomal

June 2019
1. senario descrip. bekwith wedman ... large tongue, ear crease
decrease weight 12%, weight now 4.1kg, Next investigation ?
a. Glucose
2. polyurea polydepsia +weight loss
a. RBS
b. OSMOLARITY
c. GTT

3. Boy 12 hour with very small phallus and bilateral un palpable gonad no
dysmorphic features, NORMAL feed. No hypotonia, what diagnosis?
a. CAH
b. Prader willi
c. Klinfelter syndrome

P a g e | 95
Endocrinology-DM

4. Diabetic conscious hypoglycemic 2.5, Sweaty twitches, Next ?


a. Oral glucose tablets
b. Orange juice and slice toast
c. IM glucagon
d. Iv glucose 10%

5. high TSH ,no uptake by scan


a. thyroid aplasia
b. Lingual thyroid
c. dyshormonogenisis

6. case with investigations Ca low, Ph low normal, Pth low, Alp high, 25vit d
very low ,ttt?
a. One alfa
b. Vitamin D
c. Calcium
d. Calcium and phosphorus
7. Girl 6 y with breast enlargement breast 2 pubic 1 axillary 1 obese child
thelarche
8. GE vomiting 2times at night and one in the morning before giving insulin
dose Glucose 6 normal value given (5-9), Ketone +
a. Continue same dose, Give sugary drinks and monitor glucose and
ketones
b. Reduce dose and give sugary drinks
c. shift immediately to hospital
d. continue same dose and encourage food and drink

P a g e | 96
Endocrinology-DM

Feb 2019
1. case of DKA CR 2 sec heart rate 120 PH 7.18, what to give immediately?
a. NS .9%
b. 10ml/kg bolus
c. 5% maintenance
2. Diabetic child on insulin pump presented with DkA Not shocked But severe
Acidosis Unwell, abdominal pain and vomiting, What is the next step ?
a. shot saline 10 ml/kg.
b. maintenance fluid with deficit 5%
c. maintenance fluid with deficit 10%
d. start insulin infusion ,05 u/k/h.
e. increase dose of insulin pump

3. Girl with type 1 dm. On basal bolus regimen with poor control. Hba1c
was 7.9% .there was stressful condition at home Parents heard her
screaming and saw her fitting and sweating what is the cause?
a. Night terror
b. Hyperglycemia
c. Hypoglycemia

6. Obese micro-penis
Normal obesity
7. Hyperthyroid girl on carbimazole tremor, palpitation
Propranolol
8. 11 year girl breast bud developrd 2 year ago
Normal
9. Case about premature adrenarche 8 years with breast and hair
Central (tumer) ??

P a g e | 97
Endocrinology-DM

Oct 2018
1. Child with obesity what is the cause to be treatable
a. ht 25centile
b. MICROPENIS
c. bradypnea
d. FRAGILE X
e. poor school performance
2. 12 years old well controlled type 1 DM, RBS 0.2 after evening snack.
a. Islet cell tumour
b. EXCESSIVE INSULIN INTAKE
3.ask about puberty SEQUANCE in males-
Testis,pubic hair,height
4. Premature thelarche

6. Young Boy with acne , pubic and axilary hair with HTN
a. opremature adrenarch
b. adrenal tumour
c. CAH

7. There is one for DkA maintenance fluids

8. hypoglycemia after short acting insulin BY MISTAKE FROM THE MOTHER


a. glucogel oral
9. Diagnosis of overweight
a. BMI 91
b. WT MORE THAN 75
c. WAIST MORE THAN75

P a g e | 98
Endocrinology-DM

June 2018
1. EMQ:
a. extra dose short acting insulin
b. IVF with insulin infusion
c. continuous same dose with oral fluid
d. call emergency for admission
e. decrease long acting insulin.
all child known to have DM type 1 on long acting insulin and short acting
insulin each meal

A. came with RBS 21 ketone +, whole the day, look well. with history of fever
38, cough and sore throat
extra dose short acting insulin
B. came with RBS 4.6 ketone +, unwell with history of nauseating, vomiting
continuous same dose with oral fluid
C. came with low glucose 2 mmol at 2 am and in the morning hyperglycemia
decrease long acting insulin (Somogy phenomenon)

2. An obese girl BMI 99 centile:


a. exercise and decrease calorie
b. dietitian referral
c. orlistat
d. exercise alone
e. weight management program
3. A 10-year girl on mother DM type 2 Asthma management her father had
heart attack, and uncle of father died young
a. lipid profile
b. no worry she is girl
c. Measure lipid and follow up until puberty

P a g e | 99
Endocrinology-DM

4. Girl with type 1DM take 20-unit basal insulin and bolus insulin
(can’t remember the dose) each meal with so many readings given in a
table something like below
a. increase short acting lunch and evening
b. increase long acting insulin

5. A girl Ca low phosphate low ALP high


a. hypophosphatemic rickets
b. Vitamin D deficiency

6. new born with ambiguous genitalia you assure parent investigation in


process.. what to do to aid diagnosis?
a. chromosomal analysis
b. 17 hydroxycholesterol
c. FISH
d. urea & electrolyte

7. DKA child with hyponatremia child HR high, capillary refill > 3 second
a. 20ml/ kg 0.9% NS NEW Guidelines 2020
b. 10ml/ kg 0.9% NS
c. dextrose
d. insulin IV

P a g e | 100
Endocrinology-DM

8. 12 year DM on insulin .. Poorly controlled with frequent admissions due to


DKA .. The parents are very meticulous about her treatment .. What is the
best for compliance of child ?
a. Psychology referral
b. insulin pump
c. involve her in weekend diabetic camp
d. let’s the child to take the insulin

Feb 2018
1. Sudani mother live in UK want to breast feed her baby for long time for
6-months mother BMI < 18, which deficiency baby will develop?
a. Vitamin D deficiency
b. iron deficiency
c. folic acid
d. protein
2. Long scenario of acute case of DKA, then at the end of the scenario they
asked what is the best maintenance fluid in DKA?
a. 0.9% saline with 20 mmol KCl in 500 ml
b. 0.9% saline

3. 14 years old female 147 and her mother is 167, she is concerned that she is
Short, which of the following will indicate that she will be at least as tall as
her mother?
a. Breast development stage 2
b. Pubic hair 2
c. Menarche
d. 8 cm increase in height last year

P a g e | 101
Endocrinology-DM

4. A child has hypothyroidism, came for follow up, has normal T4 but raised
TSH, best next step?
a. Measure free T3
b. Call the GP to ask about Pt compliance.

5. Baby with ambiguous genitalia and absent gonads, what test will lead to
the diagnosis?
a. Karyotyping
b. 17 OH progesterone
c. U/S

6. Child with short stature, Low ca, Ph and 25 hydroxycholecalciferols, ttt?


a. Vit D
b. 1 hydroxy-choleciferol

7. Diabetic with vomiting and diarrhea for 24 hrs looks miserable.


RBS 6 mmol, urine ketone + what is your management?
a. Give oral sugary fluids
b. Refer to emergency department
c. Give his insulin and encourage oral fluids
d. Stop insulin

8. 13 year old girl has Down syndrome with weight Loss 5 kg in the last
month, pale, proximal ms atrophy , didn’t start puberty , abandoned by
her mother when she was a baby, HR 120, agitated, diagnosis ?
a .Hyperthyroidism
b. Anorexia nervosa
c. Psycholgical deprivation

P a g e | 102
Endocrinology-DM

Oct 2017
1. Girl with type 1 DM. On basal bolus regimen with poor control.
Hba1c given was7.9%. There was stressful condition at home
Parents heard her screaming and saw her fitting and sweating
what is the cause?
a. Night terror
b. Hyperglycaemia
c. Hypoglycaemia

2. Tanner Staging for a girl presented to You with Breast Bud and sparse
pubic hair ?
a. Breast Stage1 pubic stage 1
b. Breast Stage1 pubic stage 2
c. Breast Stage2 pubic stage 2
d. Breast Stage2 pubic stage
e. breast stage 2 pubic stage3

3. year old girl deterioration in her school performance, with weight


gain and can't do well in gymnastic session. scanty periods
a. hypothyroidism
b. cushing
c. bullying
d. anorexia n.

4. A 4 yr old boy newly diagnosed with diabetes after DKA, weight loss.
family hx of diabetes, Most important thing to tell to the family?
a. the diabetic could be due to his genetic background
b. he will require lifelong insulin treatment
c. there is associated autoimmune

P a g e | 103
Endocrinology-DM

5. 15-year-old girl irregular menses, brown pigmentation, over weight,


excessive facial hair. Diagnosis?
a. Cushing
b. PCOS

6. 6-yr girl height 97th centile, with tanner score2 of axillary and pubic hair
and1 for breast cause?
a. premature adrenarche
b. premature thelarche
c. adrenal tumor
d. precocious central puberty

June 2017
1. DKA you give saline bolus what to give next?
‫حسب صيغة السؤال‬

2. 7 days old boy with glucose 1 and insulin 15


Hyperinsulinism
3. short stature, what is initial step?
mid-parental height
4. About the 12 years old child with hyperglycemia .. His father &
grandfather had DM when they were 22-25years... Mechanism of this?
MODY
5. 14yrs girl has obesity and tall for her peers, with breast development
stage 3 what to do?
a. MRI Brain
b. Do lft.lipd profile.
c. life style and weight management
d. refer for assessment by dietician

P a g e | 104
Endocrinology-DM

6. case of polyuria and polydispa weight loss ... investigation ??


random Blood glucose

7. 9 years old child grow in 9th centile for the last 3 years ..what is the
growth velocity expected next

Feb 2017
1. Bilateral firm breast masses in 18-month girl with normal development ..
what investigation to do?
Reassurance
2. They provided a table of blood sugar level during the day.Readings
showed that levels are normal (6-9 mmol) at the morning and before
launch meal, then they get much higher (~20 mmol) before and after
evening meal. Asked how to manage?
increase the launch and evening doses of short acting insulin

3. Case with Hyperglycaemia + Met acidosis + tachypnea +tachycardia + CRT


3 + Sodium 129 . First step?
a. Normal Saline 10 mL/kg
b. 20 mL/kg saline

4. Small child with hypoglycaemia in ER , received IV dextrose and now active


and playing. Ketones are negative. Mother is a nurse and father a lawyer
and
diabetic patient. The mother says he got 2 episodes of apnea Last year
Fabricated illness.
- same question in sample paper answer was reactive hypoglycemia which
occur after meal

P a g e | 105
Endocrinology-DM

5. Case with manifestations of gynecomastia, small testicles, tall stature.


Klinefelter
6. 11-year-old girl showing signs of 2ry sex characters started 2 yearsago.
Cause of the early appearance?
Normal puberty
7. A girl with Acne + few pubic hair + normal clitoris ...
CAH

8. Another case with signs of puberty


Mostly was central precocious puberty
9. A tall boy + obese + advanced bone age.
simple obesity

10. A case with Ambiguous genitalia. What to tell the parents about
his gender?
postponing naming the infant till some investigations.

11. A case with low phosphorus, low normal calcium, low vitamin D and high
PTH. Best ttt ?
a. Vitamin D
b. 25 OH Cholecalciferol
c. Oral phosphorus
d. oral calcium
12. A girl with antithyroid antibodies. Best combined ttt with carbimazole?
Propranolol

P a g e | 106
Endocrinology-DM

13. A case I think with precocious puberty, normal random and postprandial
glucose level and also cholesterol level. Best diagnosis?
a. brain tumors
b. Polycystic ovary

OCT 2016
1. Neonate with TSH 180 and no uptake in thyroid region:
a. Thyroid aplasia.
b. Dyshormonesis

2. EMQ3: DM management
A. 14 Y/O female diagnosed 6 months back with DM and she is on long acting
insulin at night and short acting insulin before meal. RBS at 2 am= 2.9
mmol/l and at morning before breakfast 10 mmol/l.
Decrease LA insulin

B. Teenager well controlled diabetic has infection now RBS=21,


urine ++ ketone
Extra-dose insulin

C. 4 years-old children with fever 39 and vomiting after supper as well as


abdominal pain and nausea, RBS= 6, urine no ketone
Same dose insulin and encourage surgery fluid

P a g e | 107
Endocrinology-DM

June 2016
1. 6 yrs old boy with headache and enuresis, NA 124, k 5.6, cortisol low,
a. adrenal insufficiency
b. SIADH
C. RTA
2. headache, osmolarity given
a. cranial DI
b. increase dose of desmopressin

3. neonate hypoglycaemia replacement 8 mg per Kg not resolved: liver 2 cm


a. hyperinsulinemia
b. GSD
4. CAH enzyme the commonest:
21-hydroxylase

June 2015
1. A newborn on examination found to have ambiguous genitalia with
hypospadias. Parents want to name their baby what is your action plan:
a. Tell them the sex of baby is uncertain and advise them to wait
b. Tell them to choose a name match for both girl and boy
c. Tell them to ascertain sex after karyotyping

Oct 2013
1. Baby with ambiguous genitalia, you told parents that you have to consult.
what is the next step?
a. Karyotyping
b. Send blood for U &Es

P a g e | 108
Endocrinology-DM

June 2011
1. Complication of Diabetes Mellitus in 16-yrs old
a. Hypertension
b. Hypoglycemia ???
c. Chronic renal failure
2. An 11yrs old boy comes to u that his penil length is only 2.5cm and not
like his friend What will u tell him about the first sign of onset of
puberty?

a. enlargment of testis

2004-2005-2006
1. A child with pubic hair, no breast development
a. brain tumour
b. adrenarche
c. adrenal tumour
d. precocious puberty

2. What is the cause of neonatal hyperthyroidism sec to maternal


hyperthyroidism?
a. TSH
b. Thyroid stimulating immunoglobulin
c. T4

3. A child got admitted with DKA with severe dehydration. After rapid
correction of the dehydration, after 6 hrs he appears drowsy, confused.
what is the next best thing u want to do?

a. IV Mannitol
b. check potassium
c. CT scan
d. LP

P a g e | 109
Endocrinology-DM

4. EMQ
A. 8-year-old girl has pubic hair development but no breast development.
mum says her arm pits smells like adult sweat. She is known asthmatic.
She is on regular becotide and had 4 admission in past requiring oral
prednisolone on each admission.
premature adnarche

B. 10-year-old girl has hair development on face and centripetal obesity.


Mum says her arm pits smells like adult sweat. She is known asthmatic.
she is on regular becotide and had 6-7 admission in past requiring oral and
IV prednisolone on each admission. -
cushings disease
5. EMQ
A. 14-yr-old girl with tanner stage 1, not growing and had no periods,TSH
normal LH ,FSH normal..Estrogen and testerone normal
constitutional delay of growth

B.14-yr-old girl has had periods and now no periods with slightly raised FSH
and not growing
Pituitary adenoma

C. 14-yr-old girl with had periods earlier and now no periods. With LH RAISED
as compared to fsh and Slightly raised estrogens
Polycystic ovary syndrome

P a g e | 110
Haematology
Oncology

P a g e | 111
Hematology-Oncology

Oct 2020
1. EMQ:
Hereditary spherocytosis
G6PD
thalassemia major
thalassemia trait
hemophilia A
ABO
A. Turkish 1-year old girl, failure to thrive, breastfeeding , pallor, with
hepatosplenomegaly with Hb = 4, microcytic hypochromic anaemia.
thalassemia major
B. Malazian boy with pallor and dark color urine without
hepatosplenomegaly took herbal ttt.
G6PD
C. North Europe 3 years boy, with mild jaundice during neonatal
period needed phototherapy for 1 day, he had two sisters are normal.
Now he presents with pallor, splenomegaly, Hb=6.5
Hereditary spherocytosis
2. Child returned from his country to London and developed weight loss,
pallor and limping for 3 weeks. One knee and ankle joints. bone x- ray
showed osteolytic lesions ask about Dx?
a. Leukemia.
b. Chronic osteomyelitis
c. Bone tumor.
d. Osteosarcoma

3. Scenario about child with Lt eye swelling and protrusion with


hypertension, ask about Dx?

a. Orbital cellulitis
b. Neuroblastoma
c. Nephrotic syndrome

P a g e | 112
Hematology-Oncology

4. Female patient with generalized purpuric rash eruption. Rt knee pain


without redness nor swelling, had also pain in ankle joints in one-week
history. Asking about dx?
a. HSP
b. systemic onset JIA
c. JIA
d. septic arthritis
e. SLE

Feb 2020
1.EMQ
a. G6PD assay
b. Osmotic fragility test
c. HB electrophoresis
d. sweat test
e. Abdominal U/S
f. Direct Bilirubin

A. 3 weeks with prolonged Jaundice with pale stool


Abdominal U/S
B. 3 weeks with consanguinity parents, anemia, UTI infection
g6pd assay
C. 4 months consanguinity no ethnicity with pallor parent with spleen +++
osmotic f test
2. Girl 6 years Carrabin with 2 weeks fever and lethargic now has limping
with Low Hg 7 and platelet 22000 and WBC 6000 and Splenomegaly with
fatigue. Diagnosis?
a. ALL
b. ITP
c. NHL
d. Sickle cell anemia

P a g e | 113
Hematology-Oncology

3. Patient came after upper Respiratory tract infection with ecchymosis and
purpuric rash and low platelet no HSM and small cervical LN
a. ITP
b. HSP
c. ALL

4. FBC with target cell anemia consanguinity, Microcetic hypchromic anemia


Hb electrophoresis

OCT 2019
1. Indian pt. came to UK, complain of tiredness, pale, spleen 2cm,
hepatomegaly 2cm, no facial features with/out Jaundice what to do:
a. CBC+ blood film
b. Electrophoresis

2. 18-month-old with vomiting and diarrhea the last one was bloody HB 10
CRP 20 ,WBC 14
a. Intussception??
b. bacillary dysentery
c. Ecoli

June 2019
1. baby 4 mouth with bleeding from nose and Bruises and petechia already in
under child protection Came with these findings,
a. Child abuse
b. ITP
2. Scenario about Tumor lysis syndrome, Leukemia ,Low hb 68 ,High wbcs
and High k , oncology team advice to start allopurinol What to give?
a. Blood transfusion
b. Iv saline

P a g e | 114
Hematology-Oncology

3.EMQ:
a. spherocytosis
b. G6PD
c. physiological
d. ABO incompatibility
e. CMV
f. RH incompatibility
g. prolonged physiological jaundice
h. galactosemia
all unconjugated hyper bill, asking about diagnosis

A. 6 hrs Jaundice with HSM 7cm spleen liver 2cm,,with peatecheal rash
comb -ve
CMV
B. Jaundice in 18 hour neonate mother o+ baby A+ coomb +ve
ABO incompatibility
C. Jaundice in 38 h female neonate baby well ,,FH of neonatal jaundice need
phototherapy
spherocytosis

4. EMQ
3 scenarios all Indian has microcytic hypochromic anemia:
(asking about diagnostic investigation)
A. Healthy child + vegetarian + was breast fed up to 6 months cow milk
waddling gate and bow legs, Delayed…..
Ferritin v/s vit D
B. Indian with HSM
Electrophoresis
C. years eating healthy mixed food + irritability and WT loss 0.4 Centile
Anti-tissue transglutaminase

P a g e | 115
Hematology-Oncology

Feb 2019
1. Patient with leg pain since few weeks but now parents notice having
ecchymosis on back and on exam he looks pale and having
hepatosplenomegaly
ALL
2. Periorbital oedema after 1 week of diahrrea hemoloytic anemia and
thrombocytopenia pale. He want investigation?
a. IgA
b. Antestreptolysin o titre
c. Peripheral smear

3. maculopapular purpuric rash and arthritis


a. JIA
b. HSP

4. 4 or 5 month not sure of age , breast fed , well baby , found to have
HGB of 5.8 what to do ?
a. blood transfusion
b. oral iron
c. change milk

5. scenario eye swelling with protrusion, fever, bp 140/85 Diagnosis?


a. hyperthryroid
b. neuroblastoma
c. langerhans histocytosis
d. orbital cellulitis

P a g e | 116
Hematology-Oncology

OCT 2018
1. EMQ-hemolytic anemia
A. Turkish 1 YEAR OLD FTT hepatosplenomegaly –HB 6
micocytohypochrominic anemia
thalassemia major
B. malazia dark coulored urine without HSM take herpal tt
G6PD
C. NORTH EUROPE 3years with mild jaundice for 1 day during neonatal
period-2 sister are normal-presented by spleen ++only
conginital spherocytosis

2. 2 yr Child of parents from Pakistan, FTT, breast feeding till 1 year


now on solid foods and unmodified cows milk , pale , HSM + , investigation
to diagnose ?
hb electrophoresis

3. Child with cough x 2 days ... had taken herbal medicine , developed
dark urine and microscopy normal
a.G6PD def
b. lead poisoning

4. ASian 6HB with palor and cousin parents and HSM diagnosis?
a. Sickle cell disease
b. THALESMIA

P a g e | 117
Hematology-Oncology

5. Pakistani boy 2 ys pale breastfeed until 1 year on solid food


unmodified cow milk-no HSM
IRON DEF ANEMIA

June 2018
1. EMQ
a. aplastic anemia
b. ALL
c. G6PD
d. SCA
e. Thalassemia
f. HS
A. a child with history of viral infection before then he becomes pale Hb
low,WBC normal
aplastic anaemia (no parvovirus infection in option)
B. feature of SCA play football cold weather leg pain look pale
SCA
C. a child with pallor and splenomegaly??
HS ??

2. female with swelling raised edge fluctuate in the thigh blue colour bleed
after mother scratch accidentally .. what you will do ?
a. observation
b. check platelet level
c. laser
d. steroid
(Hemangioma with Kassaback merit syndrome)

P a g e | 118
Hematology-Oncology

3. 2 months child with SCD what you will give?


a. iron
b. folic acid
c. pneumococcal vaccine
d. Antibiotic prophylaxis
4. baby of twins by mistake he receive wrong blood transfusion then develop
blood reaction how to avoid blood group mismatching
a. check the identification of the patient bed side
b. check the name
5. Child know case of congenital neutropenia has fever was measure at home
38. 3
a. ask to go to hospital if another spike
b. go to hospital as soon as possible
c. wait till fever subside
d. measure temperature after one hour if still high go to hospital

6. child has bone pain in knee for long time feature of pancytopenia, X ray
osteolytic lesion many site of bone
a. Osteosarcoma
b. leukaemia

Feb 2018
1.EMQ
a. Nurtitonal iron def anaemia
b. hereditary shpetocytosis
c. transient erythrocytopenis of childhood
d. Thalssimea minor

A. 4 month old, breast fed baby, Hb low , Mcv low ,reticulocytes normal?
Nurtitonal iron def anaemia

P a g e | 119
Hematology-Oncology

B. Child required phototherapy in the neonatal period , now has jaundice ,


splenomegaly: Hb low Retics: high
hereditary spherocytosis
C. 3 months old had viral infection few days before: retics 0.1 , MCV normal
HB lower limit of normal
transient erythrocytopenis of childhood
2. EMQ
a. Cephalohaematoma
b. G6PD
c. Failure to establish enough breast feeding
d. ABO incompatibility

A. Baby born term weighing 3.4 kg, at 5 day presented with drowsiness and
jaundice, mother blood group A+, Child O + ,sodium 150, urea10
Failure to establish enough breast feeding
B. Large swelling on head after pentose extraction mother , blood group O+,
baby O+ , high un conjugated bilirubin
Cephalohaematoma
C. Turkish boy has viral infection and developed jaundice,positive FH
G6PD

3. Child in social care register, presented with extensive bruises in his shin
epistaxis and purpura in the trunk, he had URTI, diagnosis?
a. ITP
b. HSP
c. Child abuse

P a g e | 120
Hematology-Oncology

4. A boy from Bangladesh came to the UK a year ago presented with liver
and spleen, jaundiced and tired in the school, no symptoms in his country?
a. Monteux
b. CBC with film
c. Stool for parasites
d. HB electrophoresis
Answer :CBC with film(if intial test,,if want diagnostic then will do hb
electrophorisis)
5. A boy with limb pains, fatigue,URTI and pallor, I think there were
investigation showing low Hb and thrombocytopenia, diagnosis ?
-ALL
6. Child has abd pain and rash in his legs and buttocks, history of viral
infection?
HSP
7. Baby has jaundice, Coombs rest weakly positive ?
ABO

OCT 2017
1. EMQ
a. Reactive
b. Atypical mycobacterium
c.TB
d. Rubella
e. EBV
f. ALL

P a g e | 121
Hematology-Oncology

A. scenario generalized lymph nodes, splenomegaly


ALL vs EBV
B. 4 year non tender with color change above it 1 node cxr clear long
Period
Atypical Mycobacterium
C. tonsillitis one month back , 0.5 nodes( multiple ) in the occipital area
and posterior triangle
Reactive Lymphadinitis

2. EMQ:
a.CMV
b. Alloimmune
c. GroupB strep
d. ParvoVirus
e. ITP

A. 15 year old pregnant lady give birth to IUGR baby by c/s due totransverse
lie at 35 weeks presented with fits, brusies and spleenomegaly of 3 cm.
CMV
B. 15 year old pregnant lady give birth to baby by c/s dueto transverselie at
35 weeks presented with fits, on examination mottled, tachypenic, plt
120 , wbc2
Group B Strep
C. 15 year old pregnant lady give birthto baby by C/s due to transverse lie at
35 weeks presented with fits. Antenatal scan at 20
weeks showed ascites, HGB low , plt low .
Parvovirus

P a g e | 122
Hematology-Oncology

3. Baby 48 hour his billurinin 444 conjugated 17 mother is scotch blood group
O+ve Father is Nigerian baby group A-ve Family history of exaggerated
jaundiced , combs' test -ve What’s your diagnosis?

a. Extensive physiological jaundice


b.G6pd
c. ABO incomp.
d. Rhesus isoimunization
e. Galactosemia

4. 4 or 5 months not sure of age, breast fed, well baby, found to have HGB of
58 what to do?
a. blood transfusion
b. oral iron
c. change milk

5. Caucasian, splenomegaly, mild jaundice .


- hereditary spherocytosis

6. scenario eye swelling with protrusion, fever , bp140/85, Diagnosis?


a. Hyperthyroid
b. Neuroblastoma
c. larghenhan histocytosis
d. orbital cellulitis

P a g e | 123
Hematology-Oncology

June 2017
1. child need urgent blood transfusion his parent told u that his blood group
is A +ve ...what type of blood group u will use
O -ve
2. case with heamolytic ureamic syndrome what u will find in blood film
fragmented RBS schistocyte

3. Child with UTI prophylactic drug & developed jaundice


G6PD

4. Mechanism of anemia in parvovirus


RBS progenitor in bone marrow(red cell aplasia)

5. baby taking cow milk .. serum ferritin or iron low and TIBC high
Iron deficiency anemia
6. baby and his sister have pica
a.lead level
b. abd X-ray

7. baby have testicular enlargement and inguinal lymph node


a.ALL
b. testicular malignancy

Feb 2017
1. Infant with poor feeding and fever. Culture urine showed UTI. Treated
for UTI and feeding improved but developed jaundice. Best
explanation?
G6PD
2. 7 month baby pale with splenomegaly. Diagnosis ?
H.Spherocytosis.
3. A case of ITP after viral illness
4. A case of Aplastic a. (Bleeding and anemia)

P a g e | 124
Hematology-Oncology

5. A case of 12 month baby with improper weaning and looks pale.


Most possible type of anemia.
Iron Deficiency

Oct 2016
1. A known case of ALL on maintenance chemotherapy, following with you
and receiving the chemotherapy in a tertiary centre. The specialist nurse
call you from the tertiary centre telling the neutrophils count is abnormal
and I think adjustment of the dose is required. Your action:

a. Call your consultant.


b. Recalculate the dose and make sure of it and agree with her.
c. Ask her to send a fax about her concern.

June 2016
1. anaemia 6-month, mother think he is pale, infrequent diarrhea,
low MCV PLT 495 Retic low, formula milk started

a. cow milk allergy


b. iron deficiency
c. folate B12
d. physiological

2. PT normal APTT prolonged boy with gum bleeding,


mother has the same condition
von Willebrand

P a g e | 125
Hematology-Oncology

June 2015
1. grandmother has given antipyretic child developed bleeding, increase
APTT (53 sec), fibrin 0.5, what the diagnosis?
a. DIC
b. aspirin toxicity

Oct 2013
1. An infant with congenital neutropenia with his family in a camp, the infant
developed fever 38.5 and the mother called you
What is the best advice to the mother?
a. Go to the hospital as soon as possible
b. Give paracetamol and observe the baby

2. Child presented with haemorrhagic manifestations. There is family history


of 2 relatives died from intracranial haemorrhage. PT and APTT are
normal. What is the most likely diagnosis?

a. Check factor XIII


b. Haemophilia
c. Vw

3. A mother of a 5yr-old who has eczema noticed a swelling beneath her


ear while combing his hair?

a. Refer to dermatologist
b. Reassure and send home
c. Refer to otolaryngologist
d. Review after 6 weeks

P a g e | 126
Hematology-Oncology

4. A 5 yr old girl is brought by mother, she has lost 5 kg weight for past few
months, agitated and can't sleep in night? Prescribe some sleep remedy

a. Malignancy
b. Hyperthyroid
c. Abuse

5. A 6 months old infant with facial eczema, found to have a palpable LN


over the neck, which is about 3 cm in size, smooth, and non-tender.
What you will do?

a. Refer to a surgeon
b. Reassurance & review
c. Start antibiotic therapy

6. A small child with left testicular mass, prominent pallor and bruises.
What is your possible diagnosis?
a. ALL
b. Testicular tumor

7. A 2 years old boy with severe pain during night and was crying
excessively. The parents can calm the baby with milk. He was pale with
bruises over the chin, but an active child. What is your possible diagnosis?

a. Trauma
b. ALL
c. Chronic hemolytic anemia
d. Iron deficiency anemia

P a g e | 127
Hematology-Oncology

2004-2005-2006
1. You are called to review a 2-day old new-born who is pale and jaundiced.
He is of Afro-Caribbean ethnicity. Antenatal, mum had an uneventful
pregnancy. What next best test?

a. G6pD assay
b. Stool reducing substance
c. Thin film
d. Hb electrophoresis
e. Osmotic fragility test

2. Child presents Unwell with shoulder and chest pain looks pale,
investigations are as follows, Platelets 171, WCC 2.5 N 0.5 , Hb 9.5 ESR 90
What is the most likely diagnosis?

a. Pauciarticular arthritis
b. ALL
c. Chronic fatigue syndrome
d. TB

P a g e | 128
Infection
Immunization

P a g e | 129
Infection-Immunization

Oct 2020
1.EMQ: Infections
maternal infection leads to baby congenital defect
a. CMV
b. HIV
c. HBV
d. Rubella
e. Varicella
f. Parvovirus B19
A. Congenital heart disease.
Rubella
B. Dermal scaring and limbs hypoplasia.
Varicella
C. Q about hydrops fetalis.
Parvovirus B19

2. Child with meningococcal infection what prophylaxis drug do you give to


his sister?
a. Ciprofloxacin.
b. Azithromycin.
c. Amoxicillin.
d. No prophylaxis needed
3. 14-days old baby with purulent eyes discharge not respond to
chloramphenicol, what the causative organism?
a. Chlamydia trachomatis
b. Streptococcus
c. Neisseria gonorrhea

4. 2- months-old infant came with cough and post tussive vomiting.


Asking about ttt.
a. azithro

P a g e | 130
Infection-Immunization

5. Mother had varicella 5 days after delivery of full-term baby, the baby well
and good feeding and has no sign of varicella what to do?
a. Check maternal and baby IgG and IgM and tack action according the
result.
b. Give IVIG to baby and give acyclovir if develop rash.
c. Give oral acyclovir.
d. No need for any action.

6. Child with fever for 6 days and red tongue and eyes with cervical lymph
nodes with swelling of the hands (signs of Kawasaki).
What to give which will affect the outcome complications?

a. Oral Aspirin
b. IVIG
c. IV antibiotics
d. Ibuprofen

7. 18-hours-old boy with lethargy and poor feeding with inflamed umbilical
skin with 3 blisters on the buttock. Ask about causative organism?
a. Staph epidermis
b. Group A strep
c. Staph aureus

8. Junior doctor came from Thailand developed diarrhea and vomiting and
he had a duty shift on weekend. what he should do?

a. Stay home till 48 hours free of symptoms


b. Do work and keep good hand hygiene (extra precautions)
c. Use PPE (wearing mask and gowns when dealing with patients)
d. Do investigation and take loperamide
e. Go work when he feels very well.

P a g e | 131
Infection-Immunization

Feb 2020
1.EMQ-Infection:
a. Acyclovir
b. Fluocxilline
c. co-amoxiclave
d. benzyl penicillin
e. trimethoprime
f. cefotrixone

A. 7 days ill Baby with vesicle in chest


acyclovir
B. 7 weeks baby Bullous skin infection and umbilicus redness and
secretions with fever
fluoxcilline
C. 3 years old well girl with urine +ve for nitrate and leucocytes
oral co-amoxiclave

2. 2 months old Baby with 2 weeks cough history not immunized and no
isolation room.
a. Admit to SCBU in Isolate
b. Refer to another hospital
c. Admit to adult ward
d. Discharge
e. Admit and inform senior
f. Admit and inform Infection Control

P a g e | 132
Infection-Immunization

3. Child coughing for 2 weeks CBC show lymphocytosis WBCs 17.000


a. Pertussis
b. TB
c. Measles

4. Patient with tonsillitis and history penicillin anaphylaxis


a. Clindamycin
b. Ciprofloxacin
c. Cephalexin
d. Clarithromycin
5. Child 10 months with fever and cough with conjunctivitis, fever ,start rash
from face and spread to all body. His brother newly admitted to Nursery
(Same as Sample Paper)
a. Measles
b. Roseola infantum
c. Scarlet fever
d. Kawasaki

6. 2 months baby came to clinic with fever 38.5 Continues crying and poor
feeding and take paracetamol, become well What to do?
a. full septic screen and start IV Cefotaxime
b. Admit to the ward and observe.
c. Send home and if sick came again
d. CBC, urine culture and observation all over the day.
e. Oral Antibiotics

P a g e | 133
Infection-Immunization

7.Child with swelling in the eye lead and ophthalmoplegia (no Ceftriaxone)

a. IV Cefotaxime
b. Clindamycin
c. Oral Co Amoxiclav
d. Metronidazole

8.Term Baby born and developed Jaundice on day 3 of life and received
Phototherapy for 2 days then discharged home on day 4 of life came on
day 6 with fever 38.3 and metabolic acidosis PH 7.3 and hepatomegaly
2 CM below the costal margin and hypoglycemia 2.2 mmo/L
a. sepsis
b. Renal tubular acidosis
c. Collagen storage disease
d. Cong.Heart disease
e. Kernicterus

9. Child in a Camp with congenital neutropenia then developed fever 38.7 C


the mother call you asking for Advice What to tell parents?
a. Advise her to give paracetamol and observe
b. Come to hospital if he became unwell.
c. Go to Hospital as soon as possible

10. 1-month boy his brother has meningitis what to do for prophylaxis?
a. single dose ciprofloxacin
b. Rifampicin
c. Ceftriaxone
d. Cefotax

P a g e | 134
Infection-Immunization

11. Child 13 month old with nephrotic Syndrome on Oral Corticosteroid dose
what is contraindicated vaccine?
a. MMR
b. IPV
c. DTP
d. Hepatitis B Vaccine

12. Pt 6 years has recurrent ear infection by examination has mass behind
ear and increase upward and laterally and pushing ear downward,
What to do?
a. CT head
b. Swab ear discharge
c. U/S
d. Blood Culture
e. X ray mastoid bone

13. Child came with swelling since 3 months in the Lt Anterior triangle with
discoloration and U/S shows clear fluid ,Born in UK fully immunized

a. Atypical mycobacteria adenitis


b. Infected branchial cyst
c. TB

P a g e | 135
Infection-Immunization

Oct 2019
1. EMQ:
a. EBV
b. ALL
c. Reactive lymphadenitis
d. Atypical mycobacterium
e. Kawasaki disease

A. child with fever, then lymph node periauricular soft, mobile non tender
1-1.5 cm has facial oedema and well child
reactive lymphadenitis
B. child with fever for one week, red tongue, cervical LN, erythrmtous rash
Kawasaki disease
C. child with fever, hepatosplenomegaly, cervical LN
EBV

2. vaccination for 2 months old baby (Know vaccination by Heart)

a. DTaP+RV+MenB+pcv
b. DTaP/IPV/Hib/Hepb +RV+ MenB
c. Dtap+RV+Menc+ppcv

3. 14-year old boy with chlamydia infection what’s the treatment:

a. doxycycline for 7 days


b. amoxicillin
c. ciprofloxacin

P a g e | 136
Infection-Immunization

4. 15-year-old girl with regular periods of which the last one was 2 weeks ago
came with Diarrhea, vaginal discharge, BP90/50 shaking and feverish
what’s the diagnosis:
a. gonorrhea
b. toxic shock syndrome

5. child had measles then afterwards developed seizure what’s the


investigation:
a. cranio imaging Ct or MRI
b. LP

6. 6-week-old baby breast fed, jaundiced, had meningitis what to give:


a. cetrixone
b. ceruxime
c. gentamicin
d. Cefotaxime

7. 6years old with sibling had meningitis what’s the prophylaxis:


a. ciprofloxacin
b. ceftriaxone

8. baby brought by his mum had fever 38C mum afraid he will get meningitis
what to tell her:
a.when he develops rash go immediately to the ER
b.when he becomes pale seek emergency help
c.try to elicit neck stiffness,if it’s there go to the hospital

P a g e | 137
Infection-Immunization

9. patient brought by his mum had vomiting and cough, becomes blue,
What to do:
a. prenasal swab
b.FBC

10. patient had fever 40, confluent rash, conjunctivitis, red tympanic
membrane what to do?
a. give Antibiotics
b. wet wrap
c. oral parcetamol

11. patient with illness took antibiotics then developed painful nodules over
Shin what’s the cause
a. mycoplasma pneumonia
b. staph

12. patient got MRSA infection what’s the important way to prevent the
spread:

a. treat the staff and eradicate


b. isolation
c. wash hands

13. Mother with HIV what to give the baby:


a. ziduviodine

P a g e | 138
Infection-Immunization

14. patient came with swelling in the eyelid, painful eye movement what’s
treatment:
a. amoxcillin oral
b. iv ceftraxione

15. A child was treated one week ago for Acute upper respiratory tract
infection mother noticed mass in neck and occipital region splenomegaly
what could be dx?
a. Infectious mononucleosis
b. Non Hodgin Lymphoma
c. Lymphoma

June 2019
1. conjunctivitis received chloramphenicol drop not improvs, came with
cough
a. clarithromycin oral
b. iv ceftriaxone
c. amoxycillin oral
2. patient On chemotherapy, his Brother got chicken pox
a. Give vzig and acyclovir when rash
b. ivig
c. acyclovire
3. Boy going to India took anti-malarial prophylactic, after coming back, pain
in Throat, enlarged cervical lymph node 3cm, HSM, constipation, jaundice.
a. IMN
b. Malaria
c. hepatitis A

P a g e | 139
Infection-Immunization

4. amber 3, signs in 5 years old child, tachypnea, tachycardia fever, decrease


conscious What investigation to do
a. cbc,urine c/c .,,,
b. cbc ,,crp,,urine c/s,,xray

5. BCG vaccine site of injection:


a. Intradermal Lt arm at insertion deltoid
b. Intradermal lt between shoulder and insertion of deltoid
c. Intradermal Rt arm at insertion of deltoid

6. Peeling flexures ...crust around eye and mouth,on mucosal membrane


affection what diagnosis:
a. SSSS
b. Erythema multiform
c. Toxic shock syndrome
d. epidermolysis bolusa
e. steven jonson

7. female need prophylaxis for meningitis, on ocp ?


Cipro

Feb 2019
1. 15 y old migrant Fever+ petechiae+ palatal petechia splenomegaly+ fever?
EBV
2. Facial Eczema with LN 1.5 cm ,,soft ,,mobile
a. review after 2w
b. excisional biopsy

P a g e | 140
Infection-Immunization

3. patient 3 year old from Asian country received, BCG vaccination came for
he has mass in neck for last few week with discharge and discoloration of
skin otherwise patient is vitally stable?
a. Active TB
b. Atypical mycobacterium TB

4. A refugee child from Syria with night sweats,thin, malnutrition, respiratory


signs .. diagnosis??
a. TB
b. PCP

5. 3 years girl Exposed to meningococci


Cipro

6. 1.5 month dry cough ,,vomitting afer cough(clear cenario of pertusis)


a. Erythromycin
b. Amoxicillin

7. eye complaints not improving after chloramphenicol


a. chlamydial Infection
b. Neisseria

8. 15 year Fever night sweat hepatosplenomegly lymphadentopathy?


a. brucelosis
b. NHL
c. TB

P a g e | 141
Infection-Immunization

9. Hx of tonsillitis a month ago with palpable cervical swellings 0.5mm?


a. reactive lymphadenitis

10. blister on buttock and redness around umbilicus?


a. staph aureus
b. group A streptococcus
c. staph aureus
d. E choli
e. haematuria

11. unvaccinated child who developed a fever, recovered, and then


developed a rash across his back?
a. Roseola infantum

12. EMQ:
a. Atypical mycobacterium
b. TB
c. Rubella
d. EBV
e. ALL
A. generalized lymphnodes, splenomegaly
EBV
B. 4 year non tender LN with color change above it 1 node cxr clear- long
period atypical mycobacteria
C. tonsillitis one month back, 0.5 nodes (multiple) in the occipital area and
posterior triangle
reactive lymph node

P a g e | 142
Infection-Immunization

13. And also small erythematous patch in the forearm Doctor thought it is
viral origin and reassure the child After 1 week Pain and fever still the
same & the patch increases in size?
a. borrelia
b. Mycopoasma pneumonia.

Oct 2018
1. Pt develop rash after penicillin and had sore throat
IMN
2. Rash after taking amoxicillin for tonsillitis
Infectious mononucleosis

3. 2 months old unimmunized child with cough requiring urgent admission,


no cabin available in ward. What to do?

a. Transfer to other facility


b. Admit in SCBU isolation cabin
c. Keep in main ward n inform infection control people
d. Keep in main ward till cabin available
e. Push hard to vacate cabin ....???special care unit?

4. 34-Child has cough x 2 weeks , clinically chest normal FBC … lymphocytosis


a. TB
b. pertussis

5. which are contraindicated in nephrotic syndrome


MMR

P a g e | 143
Infection-Immunization

6. Prophylaxis of meningiococcal meningitis?


a. rifamicin
b. Cipro

7. Child 5 y with allergy to penicillin and tonsillitis what to give?


a. clindamycin
b. cipro
c. azithromycin
8. FEVER FOR 7 DAYS AND CRACKED LIPS AND RED EYE diagnosis
Kawasaki
9. 3 y old with fever then followed by rash-diagnosis
Roseola infantum

June 2018
1. EMQ: exactly same in sample paper
a. Measles
b. Kawasaki
c. cow milk allergy
d. scarlet fever.

A. 10-months baby presented with fever 38.5 conjunctivitis, cough.


confluent blanching rashes, started on his face then cover entire all his
body. he has history of coryza with fever few days back, his sister 3 year
brother has recently started nursery
Measles
B. 18-months girl with fever for 1 week, conjunctivitis, red tongue, swelling
hands, spread erythematous rashes, palpable cervical LN fever
Kawasaki
C. 18-months girl 3 days unwell, feverish on & off red cheeks pale around
her mouth, pus in the tonsil, erythematous rashes all over body. her fever
recorded up to 38.5 HR 150.
scarlet fever

P a g e | 144
Infection-Immunization

2. child with inflamed eye lid painful limitation of movement


a. oral amoxicillin
b. IV ceftriaxone

3. New born 5 days with vesicle on trunk and lesion on rt toe redness after
2 days, with, baby febrile high grade and vomiting
a. herpes infection
b. epirdetmolysis bulosa
c. staph skin syndrome
d. varicella

4. a young child 3-4 year with bell palsy and skin erythema
a. IV doxycycline
b. amoxicillin
c. penicillin
d. ceftriaxone

5. child healthy within the last two weeks. Maternal history of varicella
infection, developed rash 2 days before labour what you will give
a. IV acyclovir
b. VZIG

6. 7-year traveler chronic cough for 6-months CXR reticule nodular shadow
Investigation?
a. tuberculin test
b. CT chest high resolution
c. HIV
d. sweet test

P a g e | 145
Infection-Immunization

7. baby 2-month continuous crying for 2 hours baby after immunization of


a. continues same schedule immunization
b. stop to give one of immunization

8. mother want to give breast feeding which contraindication


a. HIV
b. hepatitis B

9. 7-year traveler chronic cough for 6-month CXR reticule nodular shadow
investigation
a. tuberculin test
b. CT chest high resolution
c. HIV
d. sweet test

Feb 2018
1. 14-year-old with chlamydia urarthritis
a. Doxycycline for 7 days
b. Amoxicillin
c. Trimethoprim
d. Cephalosporin

2. A girl with viral infection and rash temp 40 conjunctivitis and pink
Tympanic Membrane, what to give?
a. Erythromycin
b. Antihistaminic
c. Paracetamol

P a g e | 146
Infection-Immunization

3. oral prophylaxis of 3-year-old whose brother has meningitis?


a. Ciprofloxacin
meningococal -cipro
h.influenzae-rifampicin
pneumococcal no prophylaxis

4. The site of BCG vaccination?

a. Intradermal in the left arm between the shoulder and deltoid


b. Intradermal in the right arm
c. Intradermal in the left arm at the insertion of deltoid muscle
d. Subcutaneous in the left arm between shoulder and deltoid

5. Discolored swelling in anterior triangle of neck since birth, not hot nor
tender, Pt born and lived in the UK, fully vaccinated, U/S showed clear
fluid?
a. Infected branchial cyst
b. Atypical mycobacterium
c. Tuberculous adenitis

6. Case of ALL on chemotherapy has contact with varicella patient?

a. VZIG and come back if rash appears


b. VZIG and Acyclovir if rash appears
c. VZIG and varicella vaccine

P a g e | 147
Infection-Immunization

7. A girl with coughing for 3 weeks, vaccinated Petechia over face, shoulder
and chest, diagnosis?
a. Pertussis
b. ITP
8. Child has 3 Amber scores in the traffic light system for emergency
assessment, what to do?
a. Blood and urine cultures, LP,Cxr
b. Blood and urine cultures,LP
c. Other options…..

Oct 2017
1. 7- An 18-months asylum seeker migrates from Africa to UK. No details
of his vaccination available.What are you going to Do?
a. Normal UK schedule with rotavirus
b. Normal UK schedule without rotavirus
c. BCG followed by UK schedule
d. Vaccinate according to country-of-origin schedule

2. scenario about fever for 3-days given antibiotic then fever


subsided but there was wide spread rash defect MMR?
a. rosella infantum
b. EBV
c. Measles
d. Allergy
e. Rubella

3. A child with eyelid swelling started on iv antibiotics. After 2 day


No improvement. painful and can’t gaze upward .where to refer?
a. Ophthalmologist
b. Neurosurgeon
c. Radiology
d. ENT

P a g e | 148
Infection-Immunization

4. 2-years-old girl with 6 perianal warts and mother was


treated for warts what is best advice to give?
a. not play with other children till warts gone
b. mostly warts are attributed to sexual abuse
c. will spontaneously disappear
d. chlamydia screen
e. HPV vaccination not needed

5. scenario with one boy develop bouts of cyanosis and cough, ttt?
a. Erythromycin
b. Amoxicillin

6. scenario of 7 years resp infection not responding to antibiotic then


develop red painful lesion on his chin?
Mycoplasma pneumonia
7. scenario with non blanching rash CRT 4 sec, fully immunized Diagnosis?
a. meningeococcal sepsis
b. nisseria meningitis

June 2017
1. child with eczema his mother noticed lymph node in posterior cervical
what to do?
a. reassure and discharge (should be reassure & review)
b. reassure and follow with investigation if persist after 3 months

2. EMQ:
A. CSF with 30 neutrophil and 30 lymphocytes ...protein 1.6 gluc 1.4 and
3 weeks history of irritability
Partially treated meningitis
B. CSF done 3 times and RBC 9000
Subarachnoid haemorrhage

P a g e | 149
Infection-Immunization

3. chid 7 days fever with cough and red eye


Pertussis
4. EMQ:
meningococcal prophylaxis in: All Ciprofloxacin
A. 1-month old child
B. girl taking COP
C. pregnant female
5. Whole class having sever watery diarrhoea with presence of ova in stool.
What is the organism?
Giardia

Feb 2017
1. A case with soft palatal petechiae, cervical lymph nodes -
IMN
2. A girl with fever, increasing headache for 4 days. Develops
upper limb fits. HTN, Bradycardia. Best management before
investigation?
a. Iv mannitol
b. Iv antibiotics
c. Iv acyclovir and iv antibiotics

3. mother developed chickenpox 5 days after delivery. what to do to the


infant
VZIG + come back if rash develops
4. Feverish, Bullous rash, widespread, hyperaemic surfaces when exfoliated.
Mucosal surfaces not affected. Sterile culture.
a. epidermolysis bullosa
b. TEN
c. SJS
d. SSS

P a g e | 150
Infection-Immunization

5. Varicella case diagnosis , Neonatal + varicella rash … ttt?


acyclovir IV 7 to 10 days
6. A case showing signs of Kawasaki ... most important ttt ?
a. Oral Aspirin
b. IVIG ?
7. A case with soft palatal petechiae, cervical lymph nodes -
IMN
8. A case with meningitis. Best prophylaxis for contact brother?
Rifampicin No options for cipro
9. A case of face eczema + small postauricular mass. Best investigation?
Reassure and follow up after 2 weeks

Oct 2016
1. Child present with painful left knee, paracetamol doesn’t help but
ibuprofen help. H/O travelling to France 5 weeks ago. O/E he is afebrile.
Diagnosis?
a. Rheumatoid arthritis.
b. Juvenile Idiopathic Arthritis.
c. Lyme disease
d. Leukemia.
e. Septic arthritis

2. Preterm, 32 weeks, baby with RDS, post M.V. CSF: E. coli meningitis.
Long-term complication:
a. ROP.
b. Chronic lung disease.
c. Hydrocephalus.
d. Deafness

P a g e | 151
Infection-Immunization

3. Best drug if given intrapartum reduce the risk of HIV transmission:

a. Zidovudine.
b. Nevirapine.
c. Lamivudine

4. Neonate found to have bilateral conjunctivitis and hepatosplenomegaly:


a. HSV
b. CMV
c. Rubella

June 2016
1. EMQ infection:

A. previously well child refuse feeding developed heart failure


viral myocarditis -coxakie b virus
B. periventricular calcification
CMV
C. registrar can’t illicit red reflex
rubella

2. immunocompromised with his brother 2-months, vaccine table

3. 14 yrs, know the table by heart

4. mother with hep B immunoglobulin to the baby:


HBs ag+ antiHbc ag, anti HBSag + Hbeag,
Immunoglobulin + vaccine

P a g e | 152
Infection-Immunization

5. EMQ:
Luekemia
dengue fever
malairia
typhoid

A. fever+ bradycardia--- typhoid


B. fever, from Kenya, has fatigue, malaise, vomiting, took antimalarial
drug ---- malaria
C. fever and bone pain--- Leukemia

6. prophylaxis of meningitis: -
a. ciprofloxacin
b. ceftriaxone inj once

June 2015
1. baby has GE developed heart failure organism:
a. strept
b. coxsacki
*coxsacki A --hand foot and mouth disease
*coxsacki B -- viral myocarditis

2. baby born after 6 days lethargic, after 9 days developed vesicles:


a. HSV
b. Varicella
c. staph
d. listeria

P a g e | 153
Infection-Immunization

3.EMQ:
Ceftriaxone
Cefotaxime
Amoxicillin
Penicillin
Vancomycin
gentamycin

A. treatment of salmonella
ceftriaxone
B. Rx of listeria
amoxicillin plus gentamicin in n. guidelines
C. Rx of necrotizing fasciitis swab growth of MRSA
Vancomycine

Oct 2013
1. EMQ:
Best antibiotic for:
A. Newborn with listeria monocytogenes ampicillin
B. Salmonella infection cephalosporin
C. A cystic fibrosis patient with acute pseudomonas infection
ceftazidine and tobramycine

2. Patient with atopic eczema and itchy skin lesion on top of eczema then
manifestation of cerebellar ataxia started to appear:
a. post infectious encephalomyelitis
b. chicken box
c. herpes

P a g e | 154
Infection-Immunization

3. year old presented with annular scaly lesion on back parents were
applying ketoconazole for two weeks. Now presented with fever
headche.history of travelling 3 weeks before.
What is caustive organism:

a. Borrelia burgdorferi ??
b. Leishmania
c. Ricktesia

June 2011
1. HIV +ive mom on HAART treatment what to give during delivery i/v that
she should not transfer virus to baby
a. Zidovudine
b. Interferon gamma
2. A child on maintenance of ALL…her sister has vericella…what will u do for
the ALL kid??
a. VZ immunoglobulin and vaccine
b. VZ immunoglobulin and acyclovir
Review new guidelines
3. A child who missed his pertussis immunization and now 2 yrs old.
There is outbreak of pertussis now and the parents are worried. What
will you tell them when is pertussis vaccine not safe??

a. He had 2 convulsions previous week


b. he has 39-degree fever
c. family history of epilepsy

P a g e | 155
Infection-Immunization

4. EMQ
a. BCG 1
b. Influenza
c. MMR 2 3
d. DTaP,IPV,MeN C
e. IPV
f. PCV
g. MENC
Which vaccination to avoid
A. A baby born to HIV +ive mother
BCG1 --- until screen the baby
B. Afghan boy with –ive mantoux but +ive smear, bcg vaccination given for
1 week back
MMR
C. 4-years boy with nephritic syndrome finished prednisolone treatment
recently
MMR

Feb 2011
1. 7-year-old with faltering growth bloated abdomen, Anorexic. Small bowel
biopsy confirms corns disease What is first treatment of choice?

a. polymeric diet
b. prednisolone
c. infliximab
d. none of the above

P a g e | 156
Infection-Immunization

2004-2005-2006
1. Mother deliver a baby with microcephaly, snuffling nose. Told it can be
detected in routing preg. Test and treatable. Which infection?

a. Treponema
b. CMV
c. Rubella

2. A 5 weeks old child came with complain of sticky eyes since birth and he
was on chloramphenicol since birth and now? RUL collapse on x ray, but
otherwise happy in himself. What is best treatment?
a. IV Antibiotics
b. oral erythromycin and topical eye ointment
c. just supportive treatment.

chlamydia pneumonia and conjectivitis - topical tetracycline for 2 months


and oral erythromycin

3. Influenza vaccine given to


a. premature otherwise healthy
b. pt. receiving chemo
c. chronic lung diseases
d. HIV pt.
e. Diabetics

4. Somalin boy of 10yr residing in UK has grade 1 heaf test...{Mild} the


family is going back to settle there...not symptomatic
a. he should have BCG and then sent
b. he should be treated with inh. and rifampicin
c. he should have his heaf retested in one month
d. Should not have BCG.

P a g e | 157
Infection-Immunization

5. EMQ
A. A 10 yrs. old. With no purulent bilateral conjunctivitis, fever for 6 days.
Cervical lymphadenopathy with peeling of hands and feet. And rash all
Over body
Kawasaki
B. A9 m old with same fever b/l nonparent conjunctivitis. Sub occipital
lymphadenopathy and rash spreading from face to trunks. Fever for 5 days
measles
C. a 2 yrs. old with rash from face to trunk with sparing around mouth and
lymphadenopathy and b/l conjunctivitis
scarlet fever

P a g e | 158
Metabolic

P a g e | 159
Metabolic

Feb 2020
1. Female with learning difficulty and lens dislocation

a. Homocystinuria
b. Marfan Syndrome
c. Prader- willi Syndrome
d. Soto Syndrome

Oct 2018
1. Infant with irritability and vomiting and hx of SIDS WHICH of the
Following lead to diagnosis?

a. glucose
b. ammonia ??

Feb 2018
1. A boy developed seizures after routine hernial repair, had hypoglycemia in
the past during a gastroenteritis episode, forgot the rest of the scenario
MCAD

Oct 2017
1. Mental retarded girl with lens dislocation what is the cause?

a. Marfan syndrome
b. Homocystinuria

P a g e | 160
Metabolic

June 2017
1. case with hyperammonia & normal glucose
UCD

Feb 2017
1. A case with positive screening test for PKU, what next ?
Measure phenylalanine level in blood.

Oct 2016
1. 2 years old, previously healthy apart from febrile convulsion at age of
9-month, present for hernia repair and has convulsion before the repair.
Diagnosis?

a. Idiopathic epilepsy.
b. Hyperinsulinism.
c. Hypoparathyroidism.
d. Medium-Chain acyl-CoA dehydrogenase deficiency.
e. QT syndrome

P a g e | 161
Musculo-
skeletal

P a g e | 162
Musculoskeletal

Oct 2020
1. 7-years-old boy complains of easy bruising, clumsy walking. The child
doesn’t want to participate in sports. He is presented with bruise over his
knee, he didn’t remember any trauma and by examination there were
multiple bruises over and below his knees. History of delay walking and
father also had history of delayed walking. Diagnosis?

a. Hemophilia A.
b. Ehler Danlos syndrome
c. Marfan syndrome
d. Restless syndrome
e. ALL

2. A child with 6 weeks history of waking at night, crying in leg pain. The
child has mild ligament laxity. Examination was normal. Diagnosis?
Growing pain

Feb 2020
1.EMQ:
a.SUFE
b.Perthus
c.reactive arthritis
d.Oligoarthritis JIA
e. Septic arthritis
A. 3 years Baby with history of diarrhea and Gastroenteritis in Nursery
infection and limp reactive arthritis
B. 6 years Boy with history limp for 3 months after exercise perthes
C. 14 years old Africo-cribian boy limp after exercise with sever hip pain
trying to reduce weight. SUFE

P a g e | 163
Musculoskeletal

2. 2 years old came with 6 weeks history swollen Lt Knee Join and
1 metatarsal Joint with no rash or fever , and no history of HLAB 27 +ve
and maternal aunt Psoriatic arthritis ,What is the diagnosis?
a. systemic onset juvenile arthritis
b. Oligoarticular JIA
c. Polyartecalura arthritis
d. Psoriatic juvenile arthritis

3. Patient has hot and tender Knee Joint and has fever(Septic arthritis)ESR 40
What is best next step?
a. Joint aspiration
b. Knee U/S
c. X ray
d. IV antibiotics

4. In DDH screening, which test indicates dislocatable rather than


dislocated hip?
a. Positive Ortolani test
b. Positive Barlow test
c. Hip Click
d. Affected limb is shorter (length asymmetry)
e. Failure of abduction of the affected side.

P a g e | 164
Musculoskeletal

Oct 2019
1.12 year old girl with depigmented spot 1mm she had 4 Patches ,good at
sports,mother says her back is deviated,when she puts her hands on the
floor,won’t be corrected what’s the cause:
a. NF
b. tubers sclerosis
c. idiopathic
2.Asian12 year old with rash on her face and joint swelling
what investigation:
a. DsDNA
b. ASO titer

June 2019
1.EMQ:
A. 3yr with limited external rotation and limping after recent URTI On
antibiotic prophylaxis there also history of VUR with recurrent UTI
transient synovitis
B. 12yr well growing with hip limited painful movement after exercise
BMI 91 SUFE
C. swollen red hot joint, after chicken pox difficult to pear weight
Septic arthritis

P a g e | 165
Musculoskeletal

Feb 2019
1.EMQ
a. Discitis
b. transient synovitis
c. reactive
d. Perth's

A. 8-year old girl previously fit and well suddenly unable to weight bear.
Examination of lower limb neurology is NAD. h/o upper respiratory tract
infection few days ago. O/E tenderness over lumbar spine region
remaining all examination was normal
Discitis??
B. leg pain at night Growing pain

2.There was also a Q about child coming with calf mass. Bruising after
playing football with brother, History of clumsy walking. And delayed
walking with avoidance of sports in school, father also I think had similar
condition normal apgar score
a. cerebral palsy
b. hemophilia A
c. Ehler Danlos

Oct 2018
1. EMQ:
A. fever tender warm joint-SEPTIC ARTHRITIS
B. fever 4 week ago and rash in the leg and buttocks-HSP
C. CASE HUS

P a g e | 166
Musculoskeletal

2. Rash..Anti dsDNA +ve what should we do next—slit lamp

3. Child 6 y with limping. No fever. Hip cannot be abducted or internally


rotated
a. Perthes
b. Slipped capital femoral epiphysis

June 2018
1. EMQ:
a. CBC, CRP, ESR
b. X-ray hip
c. X-ray hip & knee
d. X-ray frog position
e. Aspiration
f. US knee
g. CT scan hip
h. blood film

A. child with pain in knee, nor redness no swelling, fever 38. hip limited
internal rotated what the initial investigation.
CBC, CRP, ESR
as long as not redness no swelling so no sign of osteomyelitis so the
initial

B. child limping, limiting internal rotation, BMI 27


X-ray frog SUFE
C. child with history of frequent infection before, now came with bone ache
look pale and unwell
blood film exclude ALL

2. child with tip toe at age of 4 year and genu varus


a. reassurance??

P a g e | 167
Musculoskeletal

3. a female age walk at 24 month she born preterm she can stand easily from
sit position
a. diplegic cp
b. DMD
c. DDH
4. 14-year obese limping:
a. slipped upper femoral epiphyseal
b. Perth’s disease

Feb 2018
1. child pain below knee with swelling
a. Osgood shelter disease
b. Perth’s
c. SUFE
d. tibial tubercle
e. osteosarcoma
2. 7-year-old has knee pain and a limp especially after sports?
a. Perth’s disease
b. Slipped femoral epiphysis
c. JIA

3. 8-yr old previous viral infection...now pain in ankle joint...calf muscle


tenderness present on passive dorsiflexion. all are normal a part from
CK >1000?
a. Achilles tendonitis
b. Benign myositis

P a g e | 168
Musculoskeletal

4. A girl 14 years with brown pigmentation 3 in number 1 mm each in her


back and scoliosis, when she bends with both her hands on the ground the
Rt hips still raised more than the LT, Diagnosis ?
a. NF (more than 6 and more 15mm)
b. Bone tumor
c. Idiopathic

Oct 2017
1. EMQ
a. septic
b. SLE
c. Juvenile idiopathic arthritis
d. Scleroderma
e. Dermatomycotic
f. Lyme

A. joint pain, lymph nodes, HTN


SLE
B. Rash in the face, eyelid and back of fingers
Dermatomyositis
C. A child with swelling of knee joint. I think there was raised skin lesion
of ankle with central pallor.
Lyme

2. Most serious complication in a SLE mother


a. complete heart block
b. Thrombocytopenia

P a g e | 169
Musculoskeletal

3. scenario with 4 weeks baby with DDH (unstable left hip ) for
paediatric surgery tomorrow and mother asks you what he will do?
a. Open reduction
b. Close reduction
c. Abduction traction
d. Abduction splint
e. Double nappies

4. There was also a Q about child coming with calf mass. Bruising after
playing football with brother History of clumsy walking. And delayed
walking with avoidance of sports in school, father also I think had similar
condition normal apgar score
a. cerebral palsy
b. hemophila A
c. Ehler Danlos

June 2017
1. baby with doubtful dislocation of hip what to do?
Barlow test
2. Child present with limping and failure to abduction of hip following sport
perthe’s disease

Feb 2017
1. EMQ:
A. Joint pain with history of a school epidemic of gastroenteritis.
Reactive arthritis
B. A 8 year old with limping and knee pain after karate lessons. Knee
examination was free. Best diagnosis?
Perth disease
C. Obese + 12 years + limping
Slipped femoral capital epiphysis

P a g e | 170
Musculoskeletal

2. Case with bone aches in joints and shins specially at night. ESR 90, normal
WBCs and Hb. A brother with psoriasis.
a. psoriatic arthritis
b. SLE
c. ALL

3. Child with joint pain, one week later became hot, swollen tender and
painful on passive and active examination. X-ray was free. Baby was
looking well but I think feverish. Best next step?
a. Ultrasound
b. fluid aspiration

4. A case of JIA .. parents are discomfort after being told that their child will
be transferred to another department. What is mostly that department ?

Ophthalmology

June 2016
1. hip dislocation management:
a. binternal reduction
b. external reduction
c. double diapers

2.EMQ-Investigations:
1. neonate delivered by CS due to bradycardia which persist after delivery
anti Ro
2. teenager malar rash arthritis red urine
anti double stranded DNA

P a g e | 171
Musculoskeletal

Oct 2013
1. A child with swelling of both knee joints and one elbow joint. To which
doctor you should refer the patient?
a. Orthopedic
b. ENT
c. Physiotherapy
d. Ophthalmologist
e. Surgery
(i.e. a case of oligoarticular arthritis, for fear of anterior uveitis)

June 2011
1. A baby 2-yrs-old developmentally normal, born at full term, no past
history of note, presented with tip toe walking?

a. Idiopathic
b. cerebral palsy
c. DDH
d. Osteomyelitis

2. A child who is recently diagnosed as case of juvenile rheumatoid arthritis,


and parents are very tensed when the news is broken on them, they were
told that he will be refered to some department which they forgot, what
might the refral?

a. ophthalmology
b. physiotherapy
c. anesthesia
d. hematology

P a g e | 172
Dermatology

P a g e | 173
Dermatology

Oct 2020
1. Child with eczema ask about the most potent topical steroid?
a. Clobetasol
b. Betamethasone
c. Clobetasone
d. Hydrocortisone

2. Child complains of recurrent attacks of macular erythematous rash


appear suddenly and disappear in 24 h which white areas and associated
with recurrent RTIs, with history of facial eczema in infancy and soya-
based formula. Mother has hx of urticaria. Diagnosis?

a. chronic nonspecific urticaria.


b. Erythema multiforme
c. Hereditary Angioedema
d. Ig E syndrome / hypersensitivity

Feb 2020
1. child 3 years has multiple mollascum contagosum not annoying him
What to do?
a. reassurance
b. Cryotherapy
c. Sclerotherapy

P a g e | 174
Dermatology

Oct 2020:
1. EMQ:
a. reassurance
b. oral Griseofulvin
c. topical steroid

A. girl with pneumonia had hair loss with unequal hair loss reassurance
B. Boy with batch of hair loss (Alopecia) topical steroid
C. girl with scaly lesion in the scalp (Tenia) oral Griseofulvin

June 2019
1.EMQ:
1. baby with molusim contagusum teased at school
Silver nitrate cauterization
2.haemangioma on eye difficult to open eye
propranolol
3. eczema skin dry his brother has extreme
Emollient

2.Picture of child with rash (vesicles and crusted lesions on the trunk, face
and lower limbs, history for eczema and mother has painful mouth lesions
recently:(picture)
a. eczema herpeticum
b. Impetigo
c. Chicken pox

P a g e | 175
Dermatology

3. Large planter wart?


a. Not swim
b. refer to dermatologist
c. Put water proof plastic
d. not swim until releved spontinously
e. not sleep until response to treatment

Feb 2019
1.extensive eczema on the face, on hydrocortisone mother worried about
corticosteroid use on the face

a. tacrolimus ointment (1%)


b. oral steroids
c. oral criprofloxacin for 3-6 month
d. oral antihistamine
tacrolimus ointment .3%(Over 2yrs age)

Oct 2018
1.Pt with features of molliscum contaguism and teased by his friends ttt
cryotherapy

P a g e | 176
Dermatology

June 2018
1. 3 year child severe eczema using local hydrocortisne and there was
thickening of skin, what the next ?
a. topical steroids belch
b. topical steroids closet
c. oral steroid
d. oral tacrolimmus
e. increase frequency of hydrocortisone

Feb 2018
1.EMQ
a. Emollients
b. Propranolol
c. Cryotherapy
d. Silver nitrate
e. Reassurance
f. Steroids
g. In tras-lesion scklerotherapy

A. Baby with dry skin on his trunk, sister has eczema-- Emollient
B. Baby has a large hemangioma which prevent his eye opening?
propranolol
C. Child has mulloscum contagiosum and is teased by his friends because of
it--- Cryotherapy

2. Most potent steroid ?


a. Clobatisone
b. dermovate (clobitasol)
c. Betnovate

P a g e | 177
Dermatology

Oct 2017
1. EMQ
a. Reassure
b. topical steroid
c. oral steroid
d. injectable steroids
e. oral Griseofulvin

A. girl with pneumonia had hair loss with unequal hair loss reassurance
B. Boy with batch of hair loss (Alopecia) topical steroid
C. girl with scaly lesion in the scalp (Tenia) oral Griseofulvin

2. extensive eczema on the face, on hydrocortisone mother worried about


corticosteroid
a. tacrolimus ointment (1%)
b. oral steroids
c. oral ciprofloxacin for 3-6 month
d. oral antihistamine

June 2017
1. Child with severe eczema interfere with his daily life mother keep dairy
milk what to-do next?
a. 1% hydrocoticosone
b. topical tacrolimus

2. child with itchy rash and swelling around eye completely disappear next
day his mother has swelling around her eyes when exposed to pollens
Allergy

P a g e | 178
Dermatology

Feb 2017
1. Case with rash, papular with umbilication/depression on top.
Moescum Contagiousum

2. Case of Eczema with 2ry bullous impetigo bacterial infection and unwell,
ttt ?
IV Flucloxacillin

Feb 2011
1. A 7 yr old girl has umbilicated papules on face and hand ,her
friends tease her and don’t let her play with her her Management?
a. Reassure
b. Refer to cryotherapy
c. Topical silver nitrate

2. A 15-year-old girl presents complaining of an odd patch of skin


that she noticed on her left thigh and which has developed over
the past couple of weeks. On examination there is a very firm
and slightly indurated pale area of skin on her upper thigh, which
is a few centimetres across, and the lesion has an erythematous?
border. The pale area of skin appears to have a rather atrophic,
glazed appearance.
What diagnosis fits best with this clinical picture?

a. Lichen sclerosus et atrophicus


b. Pityriasis vesicular
c. Dermatomyositis
d. Morphoea

P a g e | 179
Dermatology

2004-2005-2006
1. Pityriasis rosea all true except?
a. usually resolves within 8 weeks
b. herald patch is small
c. itchy
d. resolves spontaneously

2. to reduce the skin induration in child requiring IM injection


a. Orange hub needle
b. Use leg rather than arm
c. rub the site after injection

P a g e | 180
Ethics

P a g e | 181
Ethics

Oct 2020
1. 10-years-old girl came with mother partner with acute abdominal pain
which Dx as acute appendicitis need operation within 4 hours, mother
outside the city and aware about her condition.
Whom to take the consent?

a. Mother partner.
b. Daughter.
c. Surgeon doctor in charge.
d. Mother by phone.

2. 13-years-old girl want to have HPV vaccine but her mother refuses, the
girl read about the vaccine and understand (competent) what to do?

a. Wait until she 18 years


b. Follow mother’s wish
c. She can have the vaccine if she is Gillick (Frazer) competent

Feb 2020
1. Child live with grandmother under protection program came with
ruptured
appendicitis came with grandmother, who can consent?

a. Grandmother
b. Social service
c. surgeon who do the Operation
d. Doctor of child protection program

P a g e | 182
Ethics

2. Mother found OCPs in her 14 years old daughter room, she ask you not to
Write OCP What to do?
a. Advise her to discuss with her daughter
b. Stop prescribing the medications

3. Father of patient find a file of pt in the restaurant and give to you and you
find it is the hand writing of 1 of your colleges
What is the most appropriate action?
a. Tell the father not to till any one
b. talk to your colleague
c. Inform the GMC
d. Report to the government
e. informs your consultant

Oct 2019
1. girl came to ER with her mum’s partner with pain, has to go to emergency
appendectomy, the mother is away who is going to consent:
a. the partner
b. the girl
c. mother by phone
2.when to transfer a cystic fibrosis patient to an adult service:
a. when he is familiar with the staff in adults clinic
b. when he is aware of the dose and the way to take his medicine
c. When he is aware about his disease
d. Aware to take decisions.
e. Understand his role and decision making

P a g e | 183
Ethics

3. 14-year-old girl, has a 15 year old boyfriend, they had unprotected sex
once and came to ask for emergency contraception, she understands the
benefits and uses of contraception, she doesn’t want her parents to know
what to do:
a. tell her you can’t issue the pills unless her parents are notified
b. prescribe the emergency pills

June 2019
1. 12yrs Boy follow up for crohns, told nurse he had new anal ulcers and
don’t want his parents to know?
a. encourages to tell parents and keep confidentiality

2. 15 yrs came in septic shock.... For emergency...... the nurse tells you that
she didn't take consent for canula insertion.. What you will tell her?

a. inserts it without consent


b. take parents’ consent
c. check if she competent for consent
d. Proceed without consent

Feb 2019
1. Q ask about consent:
a. 13 year and over if competent
b. 16 year and over if competent
c. Any age if competent

P a g e | 184
Ethics

Oct 2018
1. GP role in parent meeting
2. Mother want to do circumcision to her boy and divorced I think
what to do
a. GP must refer for surgery
b. parent can discuss together

June 2018
1. child has brought by the teacher with intestinal obstruction need urgent
gastro surgery, parent not around, the home was called grandmother was
there .. the child reregister in protection plan; who give the consent

a. grandmother
b. surgeon who will do the operation
c. doctor on charge of care
d. the teacher
e. doctor on call in child protection

2. A man found a medical sheet of admitted patient in the restaurant of the


hospital . he inform you as a doctor then you found same hand writing of
your colleague, what you will do ?

a. to talk to the colleague


b. inform local medical team
c. inform medical governcy
d. call medical council

P a g e | 185
Ethics

3. a mother of 13 year old girl , found OCP in her bed room, and went to GP
for advise. what the GP will do?

a. advise mother talk to her


b. don’t see her and keep confidentiality.
c. ask the girl if she has a boyfriend
d. prepare appointment to whole family
e. prepare appointment with girl alone

4. mother with HIV just delivered a new born doctor decided HIV prophylaxis
she came with her partner (biological father) who is not aware about of
her HIV. and the husband not aware?

a. inform the father the condition baby


b. discuss with mother to tell the husband
c. inform child care for baby protection as he is at risk
d. inform the GP about the condition of baby & its risk
e. follow for HIV team management.

Feb 2018
1. 13-year-old girl wants to have HPV vaccine and her mother refused?
a. She can have it if she is Gillick competent
b. Wait until she is 18
c. Follow mother’s wish

P a g e | 186
Ethics

2. 10-year-old came to the ED with her mother’s boyfriend, she is due to


appendicectomy after 4-hours,her mother is outside the country and can’t
get back ,who will consent?
a. Mother by the phone
b. The doctor
c. The boyfriend
d. The Child

Oct 2017
1.14-year-old girl presented with smelly vaginal discharge, she told you that
she had two-times unprotected sex with a boy, and she knew that he has
been sleeping with other classmate after her, she told you not to tell her
parents
what to do?
a. contact social worker
b. transfer to sexual clinic
c. encourage her to tell her parents and you will keep her confidentiality
d. encourage her to tell her parents and you can’t keep her confidentiality

2. 12year old boy with crohn’s disease came for regular follow
up and told that he have some lesion around the perianal area,
and told not to tell his parents what to do?
a. encourage him to tell his parents and keep confidentiality
b. encourage him to tell his parents but tell him that you can’t
keep confidentiality

P a g e | 187
Ethics

3. 45-year-old mother having Huntington, knowing that it is inherited she


asked for her 9 old boy to be tested and not tell him why?
a. wait until he is competent
b. do the test if he developed symptoms
c. No Need as there is gene therapy now
d. perform the test
June 2017
1. 15 years old girl with paracetamol toxicity level at 4 hours is below
therapeutic treatment she denied any problem at home
a. ask her if she is pregnant
b. do pregnancy test or discharge

2. brain dead child and his family refuse removal of live assistance
who can consent??
Seek legal advice

3. child need appendicectomy come with his grand parents who will
consent??
✓ consent by senior
‫االجابات المحتملة‬
✓ consultant
✓ inform mother by phone

4. child need pyloric stenosis surgery his mother on methadone for opioid
withdrawal program and both child and mother in foster care.
who can give the consent? The mother

Feb 2017
1. A grandmother with her 16-year-old daughter who presented with her
infant for vaccination. The grandma never vaccinated the 16yr old
mother. What to do?
Vaccinate both the mother and the baby

P a g e | 188
Ethics

Oct 2016
1. Your consultant prescribed methotrexate as a treatment for RA patient.
The parents read on net that this drug should prescribed only for malignant
conditions. Your action:

a. Tell them as it prescribed by a consultant you should take it even if it is


not licensed. ??
b. Refer them to rheumatologist.
c. Convince them to take it.
d. Ask them NOT to take it.
e. Refer them to social worker for education.

Oct 2013
1. regarding MMR vaccination:
a. Grandmother can consent
b. Dad who is separated but divorced
c. A stranger......!
d. A sister (can't remember the age

June 2011
1. A 14-years-old girl discloses to her nurse that she missed her last cycle
and her hcg was positive, she begs not to tell her parents and asks for
abortion, what will u do?

a. respect her wish and refer for abortion


b. straight tell her parents
c. send her for abortion and then tell her mother
d. encourage her to tell her mom

P a g e | 189
Patient
safety

P a g e | 190
Patient safety

Oct 2020
1. Q about yellow card for what you use it?

a. Record Penicillin allergy


b. Weight of child
c. Vaccination
d. Side effect of steroids ttt

Feb 2020
1. Baby 3 months old with sever reflux refractory for 1st line treatment
Consultant ordered Omeprazole, mother is Pharmacist and saw the leaflet
and ask you about the drug

a. Unlicensed drug safe in Adults and not studied in this age group
b. Ask the consultant to write licensed drug
c. till them that this drug is not experimentally approved but with safe
effect
d. till them you do not know and the consultant will change it

2. 4years Child has Epilepsy newly diagnosed What to do?

a.The parents should agree with the plan of treatment


b. Educate the student about the epilepsy in childhood
c.teach the hall school staff about basic life support
d. Exclude form school Activity
e. the class assent should be with her all the time

P a g e | 191
Patient safety

Oct 2019
1. spironolactone as unlicensed drug, the junior doctor looked it up on the
internet and found its not used for this age, what’s the most important
action?
a. Inform the parents you will use unlicensed drug
b. Write on the patient notes that you will use unlicensed drug
c. till the parents about the side effects of drugs.
d. Monitor electrolytes

2. patient had a wrong blood transfusion what can you do to prevent that:
a. check the pt. wrist id
b. check the bedside identification and needs for transfusion

3. The nurse missed a dose for vitamin k for a new born and parents is
waiting outside what is the initial steps:

a. inform the parents and consultant


b. inform the consultant and make an incident report

June 2019
1. Gentamicin over dose by nurse—next?
a. Inform parents and tell them the investigation going on
b. Inform parents and the nurse is responsible for
c. goverment
d. dont inform

P a g e | 192
Patient safety

Feb 2019

1. patient on prophylaxis penicillin then develop dental caries?


a. start another antibiotic
b. laise with dentist
c. sugar free penicillin
d. no treatment
e. check if the penicillin contain sugar

2. Yellow card
Penicillin allergy

June 2018
1. A 4-month baby the nurse forgot to give vitamin K IM at birth then he was
admitted with (i think bleeding). you admitted him for 2 days and ask the
nurse to give vitamin K now. the father is waiting and asking if anything
happen ?

a. complete incident report and inform consultant


b. Tell parent what exact happened and apologies to them
c. medication error due inform incident form
d. Tel parent nothing happens and the baby taking his routine supplement.
e. inform parents and the consultant

P a g e | 193
Patient safety

2. consultant prescribe methotrexate for management of JRA, the parent


read methotrexate is used of treatment of cancer .. what to do?

a. ask rheumatology consultant help


b. start methotrexate and monitor CBC
c. stop methotrexate
d. some drug can be used off license
e. inform parents the use of methotrexate after fail of other management

Feb 2018
1. A nurse prepared double the dose of gentamicin for a patient and it was
given by mistake?
a. Tell the parents what happened and that the investigation is going on.
b. Tell the parents what happened and who is responsible
c. Start invest and then tell the parents
d. Don’t tell the parents.

2. A doctor gave a child caffeine which is unlicensed, what should he do


about this?
a. Write in the file why you used it
b. Tell the parents that you gave their child an unlicensed drug
c. Tell the parents the expected SE.
d. Tell the parents that you’ve used all the therapies with no response.

P a g e | 194
Safeguarding

P a g e | 195
Safeguarding

Oct 2020
1. 3-years old girl came to A&E with minimal vulval bleeding, she was
playing in the garden with her elder brother who is 15 years old and
witnessed her falling off the swing on her bicycle.
What would you do?

a. Examine her under sedation


b. Call senior paediatrician for assessment
c. Refer to gynaecologist
d. Skeletal Survey
e. Call child protection (not sure)

2. Boy came on Friday night with suspected NAI


Admit

3. 15-years girl took 20 tablets of paracetamol when admitted to hospital


she escaped, you called the family and they said she did not come to
home, what is your action?

a. Tell the hospital security to find her


b. Call the social service
c. Call the police.

4. Infant (I think was CP also) with gastrostomy tube feeding came with
decreased level of consciousness with high Na = 190 with normal K and
urea and creatinine ask about the cause

a. Fabricated or induced illness


b. Nephrogenic DI
c. Hyperaldosteronism
d. SIADH.

P a g e | 196
Safeguarding

5. 8-years girl from Sudan come to you for ttt of UTI, she refused to be
examined then she told you that her aunt was cut her from down
(genital mutilation) ask about your action?

a. Call police
b. Contact social service
c. Do nothing.

Feb 2020
1. Parents with learning difficulty, the child is not taking or walking, is talking
3 words and neurological exam is free.
a. Neglect
b. CP
c. Sexual abuse

2. 12 Months old infant came with his mother 17 years baby with GCS 6 and
with bruises on the Abdomen and blood form mouth

a. CT head
b. Skeletal survey
c. Coagulation profile

3. Well Baby delivered his mother has bruises his brother 3 years in child
Protection
a. Mid wife of safe guarding
b. Keep baby with the grandmother
c. Call Police
d. Admit and do investigation

P a g e | 197
Safeguarding

4. Mother has lymphoma and she don’t like any one to know she has conflict
with the father and child has FTT and there is meeting for child Condition.
He is the family doctor. What is the GP role in the meeting?
a. read growth chart
b. give information about the family
d. Advocate for the parents
e. give report about family accommodation

Oct 2019
1.you are junior doctor and young parents brought their baby with spiral
fracture of Hummers what to do:
a. call senior registrar
b. skeletal survey
c. Tell parents you are suspecting child Abuse
d. Call senior Orthopedics

2.one sign for urgent CT head:


a. bruises around the eyes
b. amensia for more than 5 minutes
c. loss of cons. For more than 5 minutes

3. 2-month swith bruises came on Friday night and you suspect child abuse
what to do:
a. Brain CT
b. admition and inform social worker

P a g e | 198
Safeguarding

June 2019
1. 9-year no constipation soiling and leukocyte strase ++ in urine, recent
mother partner with his son 15years, stay always in room
a. sexual abuse
b. UTI

2. To whom skeletal survey should be done:


a. Spiral fracture of humorous in 4-month infant
b. Spiral fracture in tibia 2 years child
c. Spiral fracture of femur in 23-month child

3. Girl paracetamol toxicity 10 tablets below level, Dysuria refuse eating in


morning
a. Ask if pregnant
b. Do HCG
c. Refer outpatient CAMHS
d. Allow to go home

4. Scald ,Which indicates abuse:


a. Regular margins and equal depth lesion
b. Circumferential
c. abcent of splach mark

P a g e | 199
Safeguarding

Feb 2019
1. Girl present with paracetamol over does not toxic dose and multiple
scratch marks on her hand, what to do?
a. observation for 4 hours then discharge
b. discharge as it's not toxic dose
c. CAMHS
d. admission

2. girl was circumcised in Sudan


a. Police
b. FMG Social worker

3. Sure sign of abuse 4 month?


a. ear bruise

4. baby with bruises in the cheek the mother has difficulty with breast
feeding they came on Friday night what to do?
a. discharge and come back on Monday
b. admits
c. social service
d. bleeding profile

5. Feeding jejunostomy low GCS,, NA* 190


a. Fabricated
b. GE
c. Salt poising

P a g e | 200
Safeguarding

Oct 2018
1. 17 yours old mother just delivered, mother has bruised around one
eye, baby doing good. Elder child was placed at foster home for a brief
period at 1 your old age. What to do?

a. Send mother n baby to grandmother's place


b. Inform local midwife to keep on watching
c. Don't discharge call the police
d. Arrange coagulation study for baby
e. Contact midwife regarding safeguarding

2. Same scenario of TAS sample paper, but need diagnosis not


investigation. Girl with hypoglycemia and admitted became ok all 24h
of admission mama is a nurse father is a lawyer
a. FABRICATED ILLNESS ??
b. KETOTIC HYPOGLYCEMIA

June 2018
1. 13-year child brought by ambulance took 20 tablet acetaminophen while
waiting the result of drug level taken at 4 hr child escape . parent was
called and they answered she is not at home
a. call police
b. social worker will ask emergency protection
c. parents when come back
d. security to find her
e. doctor responsible for child protection.

P a g e | 201
Safeguarding

2. 11-year female just arrived form Egypt, mother show her pants with
obvious blood her breast and axilla tanner stage 1 she refuse for local
examination
a. female genital mutilation
b. precocious puberty
c. vaginal infection
d. menarche

3. 13-year-old girl with living with mother who known a sex worker was
bringing her at night presented with enuresis, constipation, vaginal
greenish discharge, the child has change behavior like to stay alone
a. non-consensual contact with adult
b. non-consensual contact with child
c. consensual sex

4. neonatal 12 hour brought by parents with abdominal bruises, vomiting of


blood. what is the best to do?
a. CT bran
b. skeletal survey
c. coagulation profile
d. Vitamin K IV
e. IV antibiotic

5. the most risk factor for SIDS?


a. Bed sharing
b. separate room from parent

P a g e | 202
Safeguarding

Feb 2018
1. Indication of skeletal survey from the following?
a. 2-year-old with spiral femur
b. 21 month with spiral tibial
c. Sibling with a non-accidental injury
d. 4 month with spiral humorous

2. 8-yrs girl developed 2ndry enuresis and fecal soiling after her mom
brought her partner with his 15 yrs old son to the house, she stays more
hours in her bedroom, diagnosis?
a. Constipation
b. Child abuse

3. A girl came with vulval bleeding ,she was playing with her teenage brother
who said she got the trauma while she was ridding a bicycle
What to do first?
a. Call senior
b. Call Gyne
c. Suite the wound

Oct 2017
1. scenario with NAI (non-accidental injury) What is most indicative?
a. Spiral fracture
b. asymmetrical nappy rash
c. parietal hematoma

P a g e | 203
Safeguarding

2. 9-months old with GCS of 6. Mother say she rolled off sofa while
3-years old was watching him. Most appropriate step after stabilisation?
a. Ct brain
b. US brain
c. Skeletal survey
d. Clotting profile

(indication in less than1 year with low GCS)

3. 2 month , rolled from sofa, diagnosis- can’t remember clearly


NAI

4. Scenario with a mother having difficult with breastfeeding,


presented at Friday night, child has bruise on his left cheek, mother
is a teacher, father is accountant, given oral vitamin K
What to do?
a. Clotting and review result on Monday
b. phone the social worker on-call
c. admit for evaluation
d. discharge and come back on Monday

5. scenario with scald Sign suggest of abuse?


a. absence of splash marks
b. blisters
c. sparing flexures
d. clear margin and uniform burn depth
e. circumferential burn

P a g e | 204
Safeguarding

June 2017
1. newborn his mother has bruise around her eyes and older sibling in
social service records what to do?
Contact Safeguard nurse OR midwife assigned for the baby
2. CP child with gastrostomy NA 190 normal K?
Fabricated

Feb 2017
1. Baby 2 months with spiral humerus fracture. What to do ?
a. Skeletal survey

Oct 2016
1. 10 years old girl brought by her mother, which is a sex worker, with green
discharge appeared on her pant. She refused examination and just ask for
a medication, you suspect:
a. Inconsequential sex contact.
b. Consequential sex contact with adult
c. Sex contact with a girl. (The same ?)

2. 15 Years old present after 20 paracetamol tablet intoxication, you told


her to wait for 4 hours to do Paracetamol level but she escaped. You
called her home and parents state they don’t know where is she.
Your action:
a. Ask hospital security to find her.
b. Call child protection registrar.
c. Call police to look for her.
d. Tell parents to ask her to come to hospital when she returned home

P a g e | 205
Safeguarding

June 2016
1. child has be well until early no fever bruising around the mouth / bleeding
from mouth, low GCS, bruises in abdomen
NAI
2. what is the indication of non accidental injury
retinal haemorrhage

June 2015
1. Child came with spiral fracture after fall from the bed Action?
a. Ask senior
b. Tell parents possibility of NAI
c. Make skeletal survey
d. child physical abuse

2. girl came with vulva bleeding after falling from bicycle What’s ur action?
a. Suture
b. Send to obs and gynea
c. Call child protection
d. Call your senior paediatrician

3. Girl her father abused her sexually came with (funny spills) What’s best
action?
a. MRI
b. CT
c. EEG
d. Refer to physiatrist
e. PTSD

P a g e | 206
Safeguarding

Oct 2013
1. What about that boy 3 years old presented to ER with burn 4 cm on his
chest his mother said that he was running to his grandmother and the tea
is thrown up on his chest now he received opiate and is good the boil now
is 4 cm what to do?
a. put ice
b. give antibiotic
c. IV fluids
d. refer to dermatologist
e. refer to social services

2. A 5-month-old African boy brought in by mother that he is not moving his


left leg, he has viral Flu like illness last week, 2 days back he also fell from
sofa, she suspected her elder son a 3yr old pulled him On exam he has
runny nose, and not moving his left leg and afebrile, what's the cause?
a. Osteomyelitis
b. Non Accidental injury
c. Transient synovitis
d. Sickle cell disease

3. A 3-year-old girl, whose elder sis is subjected to sexual abuse, complains


of perineal soreness which sign on exam confirm sexual abuse

a. Vaginal discharge
b. Torn hymen
c. Perianal warts

P a g e | 207
Safeguarding

4. A 2 years old girl with an injury to the vulvar area and mild bleeding,
while cycling in the park with her brother. What is your next step?

a. Call your senior pediatrician


b. Refer to a gynecologist
c. Suturing the vulvar wound

Feb 2011
1. A new born baby on neonatal examination has no abnormal
Finding, mother is insisting to take the baby home, on discharge
his mom is noted to have a bruise around her eye, he is the
second sibling of 2yrs old who is found to b on the child
protection register, What will be ur next step?

a. Call the police


b. Contact the named midwife for baby protection
c. Call the health visitor and discuss her case
d. Let them go without intervention

no option regarding social worker

June 2011
3. A 6 wks old baby found unconscious in cot, mother says that she left a 4 yr
old told to have an eye on the baby when he, fell of from the sofa earlier

a. Ct brain
b. Skeletal survey
c. Mri brain

2004-2005-2006

P a g e | 208
Safeguarding

1. A 17 yr old girl brings a child to AE with fever and failure to thrive.


Weight on 3rd centile, birth weight 50th centile, with car accident at
36-weeks and immediate alone with baby being ventilated and
resuscitated with cardiac compressions. On skeletal survey you observe
2nd, 3rd, 5th rib fractures healed...the fractures are due to?

a. child abuse
b. car accident
c. compressions

P a g e | 209
Neonatology

P a g e | 210
Neonatology

Oct 2020
1. preterm 32 weeks and was discharged on oral feeding after period of
intubation in NICU then developed abdominal bloating, lethargy and poor
feeding, Infant with NEC
Ask about 1 ST step in management?

a. Stop oral feeding


b. Surgical consultation
c. Change to formula milk
d. I v antibiotics

Feb 2020
1. Term delivered with thick meconium stained liquor seen in the
Oropharynx and baby was flat, white and cyanosis and HR 100/min what is
the next step?
a. suction under direct visualization
b. Inflation with bag and mask
c. Dry and stimulate
d. Endotracheal intubation

2. Baby 28 weeks was ventilated weight 0.94 kg was feeding through NGT
has abdominal distension and bloody stool and lethargy.
What is diagnosis?
a. Necrotizing enter colitis
b. Hirschsprung disease
c. Intussusception
d. meconium illus

P a g e | 211
Neonatology

Oct 2019
1. EMQ:
a. listeria
b. GBS
c. nisseria
d. staph aureus
e. staph epidermids

A. term infant after 24 hours developed infection,culture was postive cocci


GBS
B. baby wiyh meconium stained liquor c\sculture postive for gram postive
bacillus listeria
C. baby with parentrral nutriton
staph epidermids

2. 5 days old baby breast fed, jaundiced Na 151


What to give:
a.150ml/kg/day NG tube or cup
b.100ml/kg/day NG tube or cup
c. dextrose IV

P a g e | 212
Neonatology

3.neonate delivered, after inflation breath, HR less than 60,there’s chest


Movement,what to do :

a.dry and stimulate


b.inflation breath for 30 Sce
c.chest compressions
4. Baby took methadone,his O2 sat 98 ,has noisy breathing what to do:
a.secure airway
b.IM naloxone

June 2019
1. Baby 4-week dehydrated loss 12% of birth weight breast feed on demand
Na 150 next ?
a. Frequent regular feeding
b. IVF
c. Formula feed
d. IVF then formula

2. Unstable hip in 2week neonate referred to pediatric orthopedic in next


day and mother ask what will be his advice?

a. Abduction splint
b. Abduction traction
c. Double nappies
d. open reduction

P a g e | 213
Neonatology

3. 28-week loud murmur Can not wean from MV. What to give?
a. Ibuprofen
b. Prostaglandins
c. Indomethacin

4. Mother nervous worried because family history aunt of hypothyroidism


withmental retardation -TSH high 1st day neonate was done next?
a. T4 to confirm
b. regular blood tests
c. go for thyroid screening on regular date
d. need urgent iso top scan

5. Neonate vaginal bleeding bilat non painful breast enlargement what your
best advice to the mother:
a. It’s normal finding
b. It’s due to medications taken during pregnancy

Feb 2019
1. 23 weeks decision of resuscitation
a. Parental wish
b. According to the baby condition
c. Poor out com

2. Small for gestational age pulmonary stenosis absent red reflex ?


Rubella
3. Neonate has tachycardia and tachypnea Hepatosplenomegaly and anemia
TORCH

P a g e | 214
Neonatology

4. Meconium aspiration newborn prolonged labor 36 hours PROM at second


stage of labor?
a. Admit and start antibiotics
b. suction observation
c. antibiotics and observe
5. Neonate came after six hours with milky stool
Serum bilirubin
6. Neonate lethargy--- Hypoglycemia

Oct 2018
1. EMQ mother pregnant ---
a. if cardiac ds fetus-rubella
b. Baby with hydropes- parvovirus.
c. Baby with skin scarring -varicella

2. Newborn with irregular breathing n normal heart rate. Mother attends


methadone addiction rehabilitation group. What to do?
a. CPR & IV naloxone
b. UVC catheterization & naloxone
c. CPR & IM naloxone
d. 4 ventilation breaths

3. Neonate discharged at 6th day and readmission on day 8 by poor


feeding and sepsis
E coli

P a g e | 215
Neonatology

June 2018
1. NEONATE WITH SUDDEN DESATURATION NOT IMPROVED WITH OXYGEN
WHAT TO GIVE?
a. IBUPROFEN
b. PROSTAGLANDIN

2. baby born to mother who previous pregnancy has GBS baby, was born, the
mother afebrile during delivery ,she was taking IV antibiotic before
what u will do to the baby?
a. take CRP & swab, start iv antibiotic and observe for 24hr
b. choose observation for 48 hours
c. reassure and come back to hospital if something happen

Feb 2018
1. EMQ
a. Dry and wrap in a warm towel
b. Naloxone
c. Endotracheal tube under direct visualization
d. Inflation breath
e. Bag and mask ventilation
f. put in neutral position

1. Term, meconium stained liquor agar score 9 at 5 min cried immediately


pink and well
Dry and wrap in a warm towel
2. Term, bradycardic, mother took pethidin 2 hours before labour, dried and
stimulated but no response
inflation breath
3. Term, floppy, mecoinum seen in nasopharynx
Endotarcheal tube under direct visualization

P a g e | 216
Neonatology

2. 3 days old with breast enlargement and blood in the nappies?


a. Normal finding
b. Drug during pregnancy

3. Most common complication of infant of diabetic mother?


a. Sacral agenesis
b. Diaphragmatic hernia
c. Duodenal Atresia

4. Term baby born with C/S due to fetal distress developed tachypnoea at
4 hours, what is the cause?
a. Transient tachypnoea of the newborn
b. Sepsis

Oct 2017
1. Baby delivered at 28 weeks now 10 days old and difficult to wean
off from ventilator, Has a loud systolic murmur. What’s your Next step?
a. Dexamethasone
b. Furosemide
c. Prostaglandin E
d. Spironolactone
e. Ibuprofen

2. 6-hour newborn with sero-perulant eye discharge what is the


cause?
a. Neisseria Meningitidis
b. Chlamydia trochmatis
c. Neisseria Gonnorhe
d. Streptococcal

P a g e | 217
Neonatology

3. mother receive dimorph in deliver a baby flobby, no resp effort ,


covered with thick meconium HR 60 what initial step ?
a. give naloxone
b. 5 rescue breath
c. suction under direct laryngoscope
d. chest compression and ventilation 3:1
e. Dry and stimulate

4. scenario of baby 29 weeks ventilated started feed on day10 then


developed abdominal distinction, unwell,stool with mucous and
blood, what is your 1st action?

a. Stop oral feeding


b. call surgeon
c. x-ray

June 2017
1. neonate with hypoxia improved after O2 head pox cause?
Respiratory cause

2. neonate sizable before discharge nurse discover the is no movement in rt


arm..what you do?
clavicle X-ray

Feb 2017
1. Newborn with conjunctivitis developed about 60 hours after delivery (I
think), not responded to 3 days topical chloramphenicol Organism?
a. Chlamydia
b. GONOCOCAL

P a g e | 218
Neonatology

2. Neonatal Resuscitation. Now chest is rising but HR below 60. Best next
step?
Chest compression
3. Breast fed baby 3 weeks, lost weight after delivery, icteric, serum sodium
150, Diagnosis?
Poor feeding

4. Formula fed baby, increased weight after delivery. Now persistent


vomiting.
He was 3.7 kg and receiving 125 ml per feed every 3 hours
over feeding
5. Baby with Intermittent bilious vomiting. Pass stool normally. Soft
non-distended abdomen. Diagnosis?
a. malrotation
b. Hirschsprung

6. A 3 weeks neonate with fever 39c° , crying irritable, feverish. Best next
step ?
Full septic screen + IV broad spectrum Abs

7. A photograph of a case with extensive blue marks on the back.They say it's
only on the back not on the trunk or limbs.
Mongolian spots

P a g e | 219
Neonatology

Oct 2016
1. A suitable way to confirm NGT position:
a. Inflation and auscultation.
b. Gastric aspiration. (guideline neonatology) also ph study
c. CXR

June 2016
1.EMQ-neonatology:
1. 34-week + low PLT with PDA
ibuprofin
2. severe aortic stenosis
PGE1 or PGE2
3. 35wks features of renal impairment (close PDA) ?? may be heart failure
frusemide

June 2015
1. baby born after 6 days lethargic, after 9 days developed vesicles:
e. HSV
f. Varicella
g. staph
h. listeria
2. EMQ: neonate had vomiting:
A. day 1 vomiting bilious
Duodenal Atresia or duodenojejunal Atresia
B. day 6 bilious vomiting
Malrotation or Hirschsprung
C. maybe day 2 non-bilious vomiting
overfeeding or reflux

P a g e | 220
Neonatology

Oct 2013
1. A baby boy is born to mother blue, heart rate 40/min, irregular breathing,
mother is given morphine 1 hr before birth, what is the most immediate
step?
a. Adrenaline
b. Intubation
c. Chest compressions
d. IM naloxone

2. Newborn baby blood glucose was low (1.9 capillary and 1.6 venous), next
step is:

a. Give bolus (dextrose 10%)


b. Give oral feeds
c. Do a septic screen

3. 3-week baby with poor weight gain (his wt was 3.6kg). He is formula fed
and his grandmother is giving him 100 ml every 4 hours strictly.
He takes his feed well but he is miserable. What to do?

Give 140 ml every 4 hour

4. A 15 days old baby with an umbilical stump granuloma, 2 mm in size, and


oozing a serous fluid. What is your intervention?

a. Reassurance and review


b. Refer to surgery
c. Silver nitrate cauterization
d. Antibiotic powder

P a g e | 221
Neonatology

5. A full-term neonate, delivered by LSCS, and developed respiratory


distress 2 hours after birth. What is the possible cause of respiratory
distress?
a. Transient tachypnea of newborn
b. Respiratory distress syndrome
c. Meconium aspiration syndrome

June 2011
1. 27weeks born baby girl ventilated for 4 days has been on breast feed well
tolerated after 10 days. She presents with bloody and mucoid stools with
abdominal pain and is unwell
a.Meckel’s diverticulitis
b. Intussception
c. dNEC
d. Appendicitis
2. Hypoglycemia in neonate,how will u resuscitate
a.10%dextrose
b.v10%dextrose+0.45% NS
c. 0.9%NS

2004-2005-2006
1. A 24 wks. +1 day gestation prim gravida is fully dilated in your hospital
with only a scbu, nicu is miles away. What is the best step u would take?
a. transfer in utero the patient to nearest nicu,
b. allow delivery, ventilate the baby
c. allow delivery and stabiles and arrange for transfer

2. In new-born, anatomical structures in relation to vertebra?


a. Inferior vena cava
b. Oesophagus
c. coeliac trunk
d. Sup. mesenteric artery
e. bifurcation of aorta

P a g e | 222
Developmental
assessment
Motor, Vision,
hearing, Language

P a g e | 223
Developmental assessment

Oct 2020
1. 13 months boy come with head circumference on 98% centile with
normal development no other abnormalities. There were no previous
recording in his Redbook. What is the best action now?

a. Cranial US
b. Fundus examination
c. Brain MRI
d. Fallow up after 3 months
e. Assess parents head circumference

2. Q about assessment of cognitive development of child 3 years old?

a. Draws with crayon


b. Obeys simple instruction
c. Points at common objects
d. Brings her shoes from cupboard
e. Puts the car toy under table

Fop 2020
1. 5 months baby smiling at 8 weeks and not follow his mother now
What is the more concern for doctors?

a. absent red reflex


b. Head Circumference 0.4 Centile
c. present Moro reflex

P a g e | 224
Developmental assessment

2. Child 2.5 years the mother is concern about his development


What is concerning not doing at this age?

a. say 3 words only


b. draw line but cannot copy circle
c. cannot pick role in a play
d. Climb star step by step

3. Baby 32 weeks wt. 1.3 kg admitted to NICU was ventilated and developed
E Coli meningitis received IV Antibiotics for 3 weeks with Increased Head
Circumference from 35 cm to 37 cm.
What is the most important long-term complications?

a. Retinopathy of maturity
b. deafness
c. Hydrocephalus
d. CLD

4. Baby 2 weeks what is the most important indication to refer to Audiology


assessment?
a. When parents concern, he doesn’t startle to noise
b. If he missed neonatal screening
c. If there is per auricular skin tag

P a g e | 225
Developmental assessment

Oct 2019
1.6week old baby what to do for hearing:
a. distraction test
b. ABR
c. pure tone

2. Infant born 2.2 now one year on the 2nd centile what to do:
a.Previous measures

3. 12 week old baby not following his mum what to do:


a. Refer to Ophthalmology

4. 2year old no words, not interacting with anyone what to do:


a. refer to child development team
b. refer to speech therapy

June 2019
1. 19 months Girl bottom shuffler scenario, not waking, mature pincer grip,
building 3 cub need
a. MRI
b. REASSURANCE
c. Ck

2. 4m squint:
refer to ophthalmologist

P a g e | 226
Developmental assessment

3. baby born at 32wk , now 5 month old infant with history of antibiotic in
NICU and nicu pass screening hearing what test to do now?
a. ABR
b. Distraction test
c. Visual reinforcement
d. Auto ocoustic emission

Feb 2019
1. 4 years, full time in nursery youngest of 6 brothers, teacher concern from
Uncharacteristic behavioral disruption, no concern at home

a. clinical psychology assessment


b. audiology assessment
c. visual assessment
d. developmental
e. psychometric test

2. Case about plagiocephaly


Reassurance

Oct 2018
1. 3-yr old child brought for assessment. Which milestone is appropriate?
a. Drawing with crayons
b. Brings shoes when asked for
c. Puts toy car under table
d. Points to known objects

P a g e | 227
Developmental assessment

2. 5 months old not following his mother what is most important in


Examinations?
a. absent red reflex

June 2018
1. a male with macrocephaly (i can’t remember more detail) what is initial to
do
a. review after 3 months
b. measure parent head circumference
c. see previous measurement

2. baby birth weight 2.2 faltering thrive at 2 year 8.4 kg what to do ?


a. past record previous measurement
b. discharge with check up by health care
c. review after 3 month
3. squint by 6 month
a. refer to ophthalmology
b. refer to orthoptist

4. Neonate 12 week don’t follow the mother


a. referee to ophthalmic
b. review later

5. when to refer the child?


a. not sitting not support at 18 month
b. not walking by 15 month
c. convergent squint at 6 month
d. tip toe walking by 18 month

P a g e | 228
Developmental assessment

Feb 2018
1. Fixed squint at 4 months, mother has history of squint in the past ?
a. Refer to ophthalmology
b. Review at 6 months

2. A girl at 3 months not following with eyes. Reviewed after another 3


months
still not improving iris free fundus free, diagnosis?

a. Optic atrophy
b. Toxoplasma
c. Delayed maturation of visual axis
d. Birth asphyxia

3. 4 months hearing test?


a. Brain stem evoked potentials
b. Tympanometry

4. A Boy having problems with pronouncing C, K, R and tongue tie but when
he was an infant no problems with feeding. He has problems in school coz
he is not saying words right referral?
a. Pediatric surgery
b. language and speech therapy
(Ankyloglossia=tie tongue)
(in tie tongue they have PROBLEM IN PROUNONCE T D N L S Z R)

P a g e | 229
Developmental assessment

Oct 2017
1. 4 years, full time in nursery youngest of 6 brothers, teacher concern from
uncharacteristic behavioural disruption, no concern at home
a. clinical psychology assessment
b. audiology assessment
c. visual assessment
d. developmental
e. psychometric test

June 2017
1. Hand preference at 9 months with normal development her father use
the same hand
Refer to neurology

Feb 2017
1. Smiled at 8 weeks, Laughed at 3 months, normal head circumference. But
mother complains he does not follow objects with his eyes. What is the
most important test to do, The scenario was trying to make sure the girl is
developmentally normal and not a case of GDD or microcephaly.
testing red reflex

Oct 2016
1. You should refer the child for further developmental assessment if:
a. Not sit unsupported at 8 month.
b. Not babbling at 6 month.
c. Walk at toe at 18 month.
d. Convergent squint at 6 month.

P a g e | 230
Developmental assessment

June 2015
1. Build 3-cubes, palmar grasp, know his name, what is developmental age
for the child?
18 months to 2 yrs

2. Stammering child the cause may be


a. Autistic spectrum disorders
b. Bilingual parents
c. Tie tongue

Oct 2013

1. A 3 years old child with a small membranous tongue tie. He has a


difficulty in speaking the letters B, R, T. his grandfather refused to do
surgery. To which doctor you should refer the patient?
a. Occupational therapist
b. Speech and language therapist
c. Surgery
2. A 5-month-old infant presented with developmental delay. He was born
vaginally, with a birthweight of 4.5 kg and APGAR score of one at
1 minute and 8 at 5 minutes. What is the possible cause of
developmental delay?
a. Inflicted brain injury
b. Congenital brain malformation
c. Hypoglycemia ??
d. Hypoxic ischemic encephalopathy

3. An IUGR baby with periventricular calcifications on the CT scan.


What is the immediate complication?
a. Hearing loss
b. Visual impairment
c. Neurodevelopmental sequelae

P a g e | 231
Developmental assessment

June 2011
1. Cataract is associated with
a. Myotonic dystrophy
b. Galactosemia
c. Marfan syndrome
except ‫الثالثة صح غالبا بيسال‬
2. A 3-year old child of a polish mother and italian father who run their own
restaurant 8 months of age lost follow up, now went to school and
presented with speech problem…what can be the cause of his speech
delay?
a. Inadequate language exposure
b. multi lingual parents
c. chronic otitis media

Feb 2004-2005-2006
1. Cochlear implant prioritization
a. child not improved after hearing aid
b. child with learning disability
c. child going to start school

2. Stammering:
a. can be caused by tongue-tie
b. refer to speech therapy in 4 yrs. Old
c. improved spontonerously
d. caused by psychological

3. Bangladeshi boy arrived to UK 1-yr ago, has squint Best next step as
community paediatrican

P a g e | 232
Neurodevelopment
Disability

P a g e | 233
Neurodevelopment

Oct 2020

1. Scenario about child with difficulty in writing clumsy and difficulty in


walking

a. Dyslexia
b. Developmental coordination disorder (dyspraxia)

Feb 2020
1. 5 years boy with symmetric crawling ,normal tone walk at 15 months, his
uncle 23 years on weal chair

a. Duchene Muscular Dystrophy


b. Backer dystrophy
c. Dyslexia
d. DDH
e.CP

2. Child with speech problem and improved now has problem in playing and
unbooting. He has problem in writing and copying but good in reading and
math

a. Dyspraxia
b. Autism
c. DMD

P a g e | 234
Neurodevelopment

3. Girl with abnormal repetitive hand movement and regressed learning, she
is withdrawn, What is the diagnosis?
a. Rett’s Syndrome
b. Autistic Spectrum disorders
c. Global developmental delay

Oct 2019
1. EMQ:
a. educational therapy
b. occuptional therapy
c. orthotist
d. speech therapy
e. Refer to dietitian
f. Physiotherapy

1. 9-monyh old with development delay has problem with solids and take
long time to feed speech therapy

2. 4-year old has tip toe orthotist

3.child with hemiplegia ,leg discrepancy has hip,knee pain and x-ray was
normal Physiotherapy

2.child with muscle myopathy,postive gower sign what’s the diagnosis:


a. Becker
b. myotonic dystrophy
c. Duchenne

P a g e | 235
Neurodevelopment

3.2 year old girl ,history of ventilation as nenoate ,cruise around furniture
,tip toe walking:
a. DDH
b. spastic diplegia
c. Duchenne

June 2019
1. ADHD with methylphenidate difficult writing association and in wearing
his clothes
a. Dyspraxia
b. Dyslaxia
c. Side effects of drugs
2. 11yrs boy go school with his sister she noticed that he had difficulty
getting into school pus he had viral illness 1month age he had also
complain of tiredness since last year weight diagnosis?
a. chronic fatigue syndrome
b. Becker muscular dystrophy
c. duechen muscular dystrophy

3. 3yrs case of male child with delayed walking , as symmetrical muscles


what initial investigation :
a. CK
b. DNA analysis
c. dystrophin

Feb 2019
1.One child in school having difficulty in sports and he cannot hold pen in
good grip otherwise learning is okay where will you refer.
a. Physiotherapist
b. occupational
c. neurologist

P a g e | 236
Neurodevelopment

2. When to refer baby?


a. 9-months old with hand preference
b. When to refer child to hospital 40C
c. Reduce skin turgor
hand preference should not be less than 1yrs

Oct 2018
1. HE can walk but can’t jump,,-12 y ,,can walk but clumsy and deteriorate in
sports and falls dawn during sports
a. DMD
b. Fredrechs ataxia
c. Becker
2. Baby 3 months hypotonic and smile weight below 0.2 centile
SMA

June 2018
1.EMQ
a. Physiotherapy
b. occupation therapy
c. Dietician
d. play therapy
e. orthotist
f. orthoptist

a. child with CP has chocking while eating .. FTT


speech and language therapist
b. 4 year persistent tip toe — orthotist
c. child with limping hemiplegia discrepancy in the length of his leg, he has
hip pain and back pain his examination and X-ray was normal
physiotherapy

P a g e | 237
Neurodevelopment

2. chest physiotherapy is best for:


a. bronchiolitis
b. impact foreign body
c. bronchiectasis
d. SMA

Feb 2018
1. ADHD scenario, child is having difficulty in writing and refused by school
teams, messy eater, can’t tie his shoes he is on methylphenidate, what is
he having?
a. Dyspraxia
b. Methylphenidate side effect
c. Dyslexia

2. A boy with normal fine motor development sat unsupported at 7 months


Rolled at 5, he is now 18 months not walking, hypotonia ,normal reflexes,
diagnosis,,(cruel around furniture, history of bottom shufflers in baby
father)
a. DDH
b. DMD
c. Bottom shuffler
d. Becker’s muscular dystrophy

Oct 2017
1. 5-yr-old with difficulty in writing and using scissors. Can’t kick
a ball , Normal neurological examination apart from generalized
mild hypotonia. Whom to refer?
a. Physiotherapy
b. occupational therapy
c. neurology
d. education

P a g e | 238
Neurodevelopment

2. scenario with difficulty in writing, clumsy, difficult with walking


a. Dyslexia
b. developmental coordination disorder
(other name is dyspraxia)

3. 2 yr old unable to walk. Symmetrical crawling, Cruises along furniture.


Tone normal. Pull to Stands on tip toe What is Diagnosis?

a. DDH
b. Spina bifida occulta
c. Duchene muscular dystrophy
d. Becker muscular dystrophy
e. spastic diplegia

Feb 2017
1. case with signs of Duchene
2. Boy with difficulty in writing and some motor activities -Dysprxia
3. When to refer a child for developmental delay?
a. frequent falling by 18 months

June 2015
1. Child start walking at 15months then develop motor regression
proceeded by upper respiratory infection
a. Duchene muscular dystrophy
b. Becker muscular dystrophy
c. Chronic fatigue syndrome

P a g e | 239
Neurodevelopment

Feb 2011
5. EMQ
Theme: Development regression
A- Hypothyroidism
B- Batten disease
C- Aminoaciduria disorder
D- Human immunodeficiency virus (HIV) encephalopathy
E- Peroxisomal disorders
F- Lead encephalopathy
G- Subacute sclerosing panencephalopathy
H- Spieler–Mayer–Schrögen syndrome
I- Leigh’s encephalopathy
J -Hydrocephalus secondary to a medulloblastoma
Three children present to the child development centre for
assessment of developmental regression. The diagnoses listed
above are possible causes for developmental regression. Match
the three clinical scenarios to one diagnosis from the diagnostic
suggestions above.
1 . A 3-year-old boy and his parents review a hospice. He
presented previously with myoclonic jerks, optic atrophy and
progressive dementia. His parents have found it increasingly
difficult to control his myoclonic jerks and he is becoming
increasingly agitated, requiring sedation. A bone marrow sample
found lipofuscin in his marrow cells. What is his diagnosis?

Batten disease

P a g e | 240
Neurodevelopment

2. A 14-year-old Iranian girl presents with a 6-month


history of developmental regression. She has regressed in all
categories of her development and now wears nappies both day
and night. She has ten words in her own language and appears
unable to communicate. There is no previous medical history to
note. She is unvaccinated. She has had all the usual childhood
rashes. On clinical examination, she is well. Her Griffiths mental
scale scoring portrays her subquotient results as follows: gross
motor 8.5months, social skills 12months, language and hearing
14months, and hand and eye coordination 18months; she was
unable to carry out the performance tasks. What is the most
likely cause of this girl’s developmental regression?

Subacute sclerosing panencephalopathy

3. An 8-year-old girl presents with a 12-month history


of early morning headaches with associated nausea and vomiting
before breakfast. She is otherwise fit and well. Her friends have
noticed that she does not participate in lunchtime meals as she
used to. On clinical examination, she is found to have increased
reflexes and clonus on the right side. On ophthalmic examination,
there is papilloedema of the left eye.What is her diagnosis?
medulloblastoma

P a g e | 241
Neurodevelopment

June 2011
1. A child with ADHD has problem that he is slowest reading for school, his
friends don’t want him play with him and even if he tries he can’t write
,what associated problem he has ?

a. Autistic spectrum disorder


b. dypraxia
c. dyslexia
d. asperger

2. EMQ
THEME: REFRAL
Occupational therapist
Speech tharipist

A. A child with achondroplasia had problem with using desk and chair at
school
Occupational therapist

P a g e | 242
Emergency

P a g e | 243
Emergency

Oct 2020
1. Child with facial swelling, pallor and stridor 1 hour after eating nuts butter
what to give him?
a. Oral antihistamine
b. IM adrenaline
c. No Rx
d. Hydrocortisone.
e. IV adrenaline

2. Child with Hx of RTA (hit by car on 50 km / hr velocity) came with shock


and there were bruises over lt upper abdomen. 2 IV cannulas were
inserted and she received NS 0.9% 40ml /kg still CRT = 4 second, poor
peripheral perfusion, tachycardia. What to do next?

a. Ringer lactate or albumin.


b. 10 ml / kg packed RBCs
c. 20 ml /kg NS 0.9 % bolus.
d. Epinephrine infusion or inotropes
e. Fresh frozen plasma.

3. 8-weeks infant came with excessive crying, irritability, refusal of feeding,


bulging ant fontanel. Mother said he rolled from sofa and fell on carpet
1day ago. BP = 140/80, HR= 80, RR= 25. What to do?

a. IV saline
b. Call anaesthesia
c. IV acyclovir
d. IV broad-spectrum antibiotics

4. Which warrant you for urgent referral to hospital?


a. fever 40
b. Reduced skin turgor
c. Reduced urine output
d. Dry mucous membrane

P a g e | 244
Emergency

5. A child with dehydration, he mentioned also that the girl was in clinic 1
or 2 weeks ago,
What is the best way to assess the degree of dehydration?

a. skin turgor
b. Dryness of the mouth
c. Compare with previous weight
d. Hypotension

Feb 2020:
1. 5 years old Girl developed skin rash after eating Ice-cream ,no
difficulty breathing with history of asthma and eczema, other
examination were normal
a. Oral Antihistaminic
b.IM adrenalin
c. Oral Steroid
d. IV Adrenalin

Oct 2019:
1. Appendectomy patient, refuse to eat, tachycardia, CRT 3 what to do:
a. Saline bolus
b. glucose bolus

2. patient developed wheeze and rash after eating what to do:


a. oral antihistamines
b. im adrenaline
c.im phenypherine

P a g e | 245
Emergency

June 2019
1. Accident GCS 8 and abdominal bruised at upper abdominal start
deteriorate
conscious, Tachycardia 170/MIN, Next ?
a. Us abdomen
b. Ct brain
c. Cannula
d. Call anathesiest

2. anaphylaxis on a party eating cake. What to give?


a. Adrenalin 150 micro IM
b. Iv hydrocortisone
c. Antihistamines

3. 7-year old car accident no loss of consciousness observed for 6 hours and
discharged what's the most serious complication?

a. Post traumatic stress


b. Extra dural hge
c Necrotizing fasciitis

4. case of CNS infection , shocked (tachycardia + CRT 4 seconds ) what to give


:
a. Iv antibiotic
b. Iv saline

P a g e | 246
Emergency

Feb 2019
1. bicycle handle hit the upper abdomen,10hrs later vomiting and
tachycardia started?

a. pancreatic injury
b. subdural hematoma

Oct 2018
1. child was still convulsing for 2 min and is feverish=39 and nurse was
upon his head with oxygen what your next action?

a. BuccaL midazolam
b. OBSERVE
c. rectal paracetamol
d. rectal IBUBROFEN

June 2018
1. baby boy haemorrhage from circumcision i guess with tachycardia HR ~
180
and capillary refill time > 3 sec cold extremities .. what is the initial step?

a. blood transfusion
b. FFP
c. IV NS

P a g e | 247
Emergency

Feb 2018
1. Scenario of a case mostly nephrotic syndrome and came with vomiting,
cold peripheries, CRT 3 Sec , ttt?
a. Saline bolus
b. Oral prednisolone
c. Maintenance IV Fluids

2. A boy in a restaurant developed sudden shortness of breath, presented


with drooling and noisy breathing, he has eczema and asthma, diagnosis?

a. Anaphylaxis
b. Foreign body inhalation
c. Epiglottis

3. Boy was in a restaurant, developed rash and swollen face after he ate an
ice cream, no difficulty in breathing nor stridor, ttt?

a. Oral antihistamine
b. IM adrenaline
c. IV antihistamine

P a g e | 248
Emergency

Oct 2017
1.EMQ
a. Blood glucose
b. Blood pressure
c. Serum urea and electrolytes
d. Urine albumin and creatinine
e. Ct Brain
f. Abdomen US

1. scenario of a baby with convulsion. large tongue


Blood glucose
2. 14-years-girl was well , history of VUR when was infant now came
with confusion from many weeks and hyper-reflexia
Blood pressure
3. hx of fall, unilateral pupils
CT brain

2. severe wheeze, urticarial rash, what you will do initial ?


a. IM epinephrine
b. IM chloramphinarmine

June 2017
1. child after appendectomy increase heart rate and prolonged capillary
refill...what is next step?
Give 20 ml/kg NaCl 0.9% bolus

2. 4-month presented with fever and vomiting weight loss, what sign if
there make urgent refer is needed?
Drowzy

P a g e | 249
Emergency

Feb 2017
1. A case of anaphylaxis ... best next step?
IM adrenaline

June 2016
1. which of the following should measured in anaphylaxis (insect bite):
a. histamine
b. mast cell tryptase
c. IgE

June 2011
1. EMQ
a. DKA
b. Drug abuse
c. Alcohol intoxication
d. Insulin over dose

A. a 15 yrs old boy with newly diagnosed diabetes mellitus, and poorly
controlled,with HBA1C 6.5% is expelled from school for his behavior,
stays out with his friends late night,came home and was
irritable,had abnormal smell from the mouth, and was drowsy, and
uttering inappropriate words
Alcohol intoxication

B. A16 yrs old girl with good control of diabetes Mellitus came home from a
party and was sweaty and drowsy, with abnormal breath from her mouth
and was vomiting her blood glucose was 18mmol
DKA

P a g e | 250
Emergency

C. A 14yr old girl with poorly controlled diabetes mellitus, came home from
party quarrelled with her mother, shut the door behind her, mother heard
her vomiting in the room, she was sweaty and had abnormal smell from
the mouth, her diabetes is poorly controlled with HBA1c 10%
Insulin overdose
2. EMQ:
a. I/v epinephrine
b. Inhaled epinephrine
c. I/V hydrocortisone
d. I/V antihistamin
e. Oral prednisolone
f. Oral antihistamin
g. Subcutaneous epinephrine
h. Inhaled hydrocortisone

A. Baby with asthma….With swelling of face and eye, but normal breathing
and otherwise well, mom thinks after eating ice-cream
Oral antihistamine
B. A 14yrs old on asthma medication on fluticasone n inhaled LABA came
with redness of tongue face and hands
Oral antihistamine
C. A 4yrs old after eating peanut came with swelling of face and tongue
IM adrinaline

2004-2005-2006
1. A 3year girl has come to A&E with status epilepticus and your registrar is
going to give IV lorazepam, what do u want to do just before injection is
given.
a. Arterial blood gas
b. blood glucose
c. LP
d. Blood culture
e. CRP

P a g e | 251
Pharma
Accidents
poisoning

P a g e | 252
Pharma-accidents-poisoning

Oct 2020
1. 9-years-old boy with cystic fibrosis on ciprofloxacin syrup for treatment
of chest infection, came on Saturday. He refuses taking the syrup because
of its taste what to do?
a. Stop treatment.
b. Give IV antibiotic.
c. Give ciprofloxacin tablet.
d. Give amoxicillin

2. A case of congenital hypothyroidism was prescribed levothyroxine in


wrong dose (higher dose). Parents are concerned about side effects.
What is the most important sign of overdose?
a. Tachycardia
b. Weight loss
c. Diarrhea
d. Hyperthermia
e. Sleep disturbance ??

3. Child with epilepsy on carbamazepine come with community acquired


pneumonia, he had allergy to penicillin, what to give to him?
a. Cephalexin
b. Ciprofloxacin
c. Erythromycin
d. Co amoxiclav
e. Amoxicillin
Feb 2020
1. What is the most risk factor for SIDS
a. Parent bed sharing
b. Room sharing
c. Dummy
d. Smoking

P a g e | 253
Pharma-accidents-poisoning

2. Girl her patents quarrel with her she disappears for 5 hours and was found
with vomiting and she takes 18 paracetamol tablets
What to do?
a. iv acetylcysteine infusion
b. Charcoal
c. Oral acetylcysteine
d. check the level now and after 4 hours
e. checks the level now and after 2 hours

Oct 2019
1. sudden infant death most important risk factor:
a. parent smoking
b.co sleeping
c. bottle feeding

2.patient taking carbamazepine and was sick took erythromycin then came
with unsteadiness what’s the cause
a. non conclusive epilepsy
b. carbamazepine toxicity

3. Cystic fibrosis patient, first time to have pseudomonas aeruginosa, doesn’t


want to take the oral cipro hates the taste and he was asymptomatic
what to do?
a. Admit for IV antibodies
b. Nebulized to tobramycin
c. oral tablet
d. stop cipro and wait for the C.F team on Sunday

P a g e | 254
Pharma-accidents-poisoning

June 2019
1.Emq:
A. Girl take Methadone over dose
naloxone
B. Girl take paracetamols but refuse iv line
oral methionine
C. Take a lot of drink in party deny drug came after went home because of
confused and sweaty, pale
glucose v/s glucagon

2.EMQ:
3 scenarios about investigation unresponsive child
A. 12-year boy found unresponsive in his room with unilateral dilated
pupil
CT
B. 14yrs Girl un responsive with bilateral dilated pupil reactive &
tachycardia
toxicology screen
C. patient was having shallow irregular resp
aspirin
3. IV salbutamol side effects?
a. lactic acidosis
b. Hypo NA
c. Hypo ca
d. hypoglycemia
e. metabolic acidosis

P a g e | 255
Pharma-accidents-poisoning

Oct 2018
1. Child spilled coffee over his chest in wet wrap brought to hospital and
distressed what to give
a. pain relief
b. measure temp
c. undressing

June 2018
1. What most important sign to do CT brain:
a. vomiting 3 times
b. Fall of 3 meter
c. Amnesia
d. loss of consciousness > 5 min
e. bruises around both eye
2. the most risk factor for SIDS?
a. Bed sharing
b. separate room from parent

3. A girl with epilepsy on carbamazepine and allergic to penicillin came with


community acquired pneumonia what you will give?
a. erythromycin
b. doxycycline
c. amoxicillin
d. cephalexin
e. ciprofloxacin

(carbamazepine enzyme inducer, erythromycin inhibitor)

P a g e | 256
Pharma-accidents-poisoning

Oct 2017
1. EMQ
a. Acetylcysteine iv
b. Diazepam
c. Glucose
d. Naloxone
e. Flumazinel
f. methionine oral

A. methadone
Naloxone
B. 14 yrs old boy presented after night out with friends, deny any drug
only drinking, Pale and sweaty, confused
Glucose
B. Girl took 25 tablets of paracetamol after 2-hour irritable and refusing
Cannulation
Methionine oral

2. scenario of a teenager and when doing urine test next day they
found opiate, what’s your explanation
(3 seblings came with same symptom)

a. crack cocaine
b. ecstasy use
c. due to lumber puncture
d. anti-cough medication containing codeine ??

P a g e | 257
Pharma-accidents-poisoning

June 2017
1. correct prescription of trimethoprim in:
mg or ml with different conc
8mg/kg/day divided /12h (40mg/5ml conc)

2. Child on penicillin prophylaxis from 4 months come with sever tooth


decay the dentist said this because of penicillin what should u do??
a. stop drug
b. change to sugar free penicillin
c. ignore dentist opinion

3. child with hypothyroidism on thyroxin replacement and get high dose by


mistake what is the first clinical sign of toxicity?
Same qs in oct 2020

Feb 2017
1. Patient with abnormal up rolling and movement of eyes. Received
Metoclopramide since days.
Oculogyric crisis of metoclopramide

2. A girl took 18 tablets of paracetamol un-witnessed last of which was 5


hours
ago as her parents sow her. She is not cooperative and refuse to speak.
a. oral NAC
b. measure her level now
c. wait 4 hours then measure
d. Intravenous N-acetylCysteine now

3. A case with tonsillitis. A history of penicillin allergy. Best alternative?


Clarithromycin
- Options included different cephalosporins

P a g e | 258
Pharma-accidents-poisoning

4. A case with constricted pupil and bradycardia, bradypnea. -


Methadone toxicity

June 2016
1. allergic to penicillin + taking carbamazepine:
a. Rifampicin
b. Ciprofloxacin
c. cefalexin

June 2015
1. Sudden infant death syndrome?
a. Risk decrease if mother smoke outdoor
b. You need metabolic screening for the next baby
c. It decreases by nursing in prone position to avoid respiratory
obstruction
d. Use of apnoea alarm decrease its incidence
e. Mother need to train about CPR

Feb 2015
3.EMQ:
Ceftriaxone
Cefotaxime
Amoxicillin
Penicillin
Vancomycin
gentamycin

A. treatment of salmonella ceftriaxone


B. Rx of listeria amoxicillin plus gentamicin in n. guidelines
C. Rx of necrotizing fasciitis swab growth of MRSA
Vancomycine

P a g e | 259
Pharma-accidents-poisoning

Oct 2013
1. EMQ:
Best antibiotic for:
1. Newborn with listeria monocytogenes
ampicillin
2. Salmonella infection
cephalosporin
3. A cystic fibrosis patient with acute pseudomonas infection
ceftazidine and tobramycine

2. An asthmatic patient received salbutamol and Ipratropium bromide


nebulizer. The patient improved but developed nausea and headache with
unequal pupils. The left one is 5 mm and the right one is 3 mm and
unresponsive to light but there is spontaneous eye movements.
What is the best test to diagnose the underlying cause:

a. Application of pilocarpine eye drops


b. Application of fluorescence eye drops
c. CT brain
3. What is the most important investigation before starting treatment with
sodium valproate?

a. Liver function tests


b. Serum ammonia level

P a g e | 260
Pharma-accidents-poisoning

June 2011
1. A 15-yrs-old girl came with recurrent abdominal pain. on investigation she
had deranged liver function tests, and HEP C At the age of 6 yrs she had
Kawasaki disease and was give immunoglobulins.
And 2 yrs back she had an accident with splenic rupture and was
transfused with .two units of blood Her parents say she has a regular
boyfriend and stays out till morning with her friends .she dose not share
her social life with her parents.
What is the cause of Hep C?

a. Blood transfusion
b. Immunoglobulins
c. I/v drug abuse
d. Sexually transmitted

2004-2005-2006
1. Side effects of long-term steroids for chronic lung disease:
a. rupture of stomach
b. rupture of duodenum
c. NEC
d. Hypotension
e. pit-adrenal axis dysfunction

2. Side effects of desferrioxamine

a. diabetes mellitus
b. yersenia infection
c. Retinopathy
d. hearing loss

P a g e | 261
Psychiatry

P a g e | 262
Psychiatry

Oct 2020
1. 15-years-old boy with 3rd attempt to suicide now come with cutting
injury in wrist, the boy wants to go home and reluctant to admission but
was calmed down by a nurse he knows from previous admission.
What the best action for the doctor to do now?

a. Telephone social service.


b. Urgent assessment by psychiatric child and adolescent service.
c. Permit him to return home
d. Discharge him and ask GP doctor to fallow him
e. Admit in child protection unit.

2. 13-years-old girl with HR 55, she loss 5 kg weight she isolates herself
from family and friends not eating with family she goes every day to
school when doctor ask her she said she is all right and she was quite
during assessment ask about Dx?

a. Eating disorder??
b. Depression
c. Hyperthyroidism

3. 3-years old child with temper tantrum, stole chocolate from


supermarket, the re was a younger sister of 10 months age what to do?

a. Praise him for good behaviour


b. Mother tell him that everybody loves him
c. Punish him for bad behaviourr
d. Refer him to psychiatric

P a g e | 263
Psychiatry

4. 20-months-old girl her mother said she found her arching her back,
profuse sweating, sea- saw respiration for 2-3 min. then she fell sleep not
repeated in the day, if interrupted she got angry. She had normal
development. Ask about Dx?

a. Reflex anoxic seizer


b. Sandifer
c. Self-gratification
d. Temper tantrum

5. 3-years boy came with his mother complaining abnormal behavior


(sameness, hand flapping), speech & language delay.
Mother is worried about school and there is no facility for behavioural
assessment in you unit, what should you do?

a. Prescribe drugs
b. Refer to educational psychologist for preschool
c. Speech and language therapy referral
d. Refer to health visitor for behavioural assessment.
e. Refer to community paediatrician for assessment

Feb 2020
1. 14 years old Girl lack of interest, not refusing investigation or toxicology
Screening and screening was normal, she was refusing eating with weight
loss
a. Depression
b. Autism
c. ADHD
d. Anorexia nervosa

P a g e | 264
Psychiatry

2. 10 years old boy, he is awake at Night and screaming and go back to sleep,
in The morning he cannot recall. What is the management?
a. Reassurance
b. Sleep Hygiene
c. Melatonin
d. keep light opened

3. Boy 14 years old. He has parotitis 7 days and is athletes since he was 11
years his weight on 0.4 centile drop from 50% height on 25 % by
examination he has prepubertal testicle
What is diagnosis?
a. Testicular atrophy
b. Anorexia nervosa
c. Pituitary tumor
d. Cushing
e. Hypopituitarism

Oct 2019
1.child recently moved to new house and school, he is out of control at home
that his mother had to call the police, he gets upset for missing with his
tidy toys, at school he is quite, good at his math
a. autism spectrum
b. ADHD
2.parent with child when he cries, he becames blue and fall down what
advice to give them:
a. distract him from the trigger
b. give him O2
c. give him bag to breath in

P a g e | 265
Psychiatry

3. patient was clumsy, enjoys playground, no one wants to invite him to their
house.
a. ADHD
b.autism

4. patient transferred from small primary school to big secondary school,


Became aggressive and developed learning difficulties:
a. educational therapy
b. home tution teaching
c. family therapy

June 2019
1. child frightened at night ,crying , recurrent attacks couldn’t remember
What
happens next day :
a. Night terrors
b. Night mares
c. Seizure

Feb 2019
1. Anorexia nervosa with social withdrawal eating disorder heart rate low
a. depression disorder
b. eating disorder
2. 2-year Girl with abnormal posture occur with stress or cry ,symptoms
“see-saw” at the beginning of sleep? hypnagogia ?
Self-gratification

P a g e | 266
Psychiatry

Oct 2018
1. Parents with learning difficulties and their son 20 MONTHS OLD can’t
talk at all. Didn’t want to cruise FTT ?
a. emotional DEPRIVIATION
b. CP

2. 12 years old thin built very active athlete with weight at 0.2 centile
height 75 centile
a. hyperthyroidism
b. anorexia nervosa

3. 14-year-boy not sleeping at night until 2 am autism at 2yrs, will not


come to breakfast , with social and language delay.,,with hyperactivity
a. ADHD
b. GLUE EAR
c. AUTISM
d. depression

June 2018
1. Athlete girl BMI 18 training every day has secondly amenorrhea
a. athlete amenorrhea
b. anorexia nervosa

2. child aggressive behavior at home after they transfer to a new home. He


academically good at school especially in math and science subject ,not
interest in group play.
a. conduct disorder
b. ADHD
c. Autism spectrum disease

P a g e | 267
Psychiatry

3. child 6 year easy distractible, fail to make friend delayed reading 2 year
different to compare with his classmate. not concentrating in his academy
in the school classmates can enjoy play with him but no body like to invite
him to his party at home, calling the teacher
a. dyspraxia
b. ADHD
c. dyslexia
d. autism
e. deafness

4. 3-year temper tantrum toddler still chocolate from market change


behaviorafter the new born sister arrived, living together with
grandmother
a. praise him for good behavior
b. mom to tell him she love her
c. assure him every body love her

Feb 2018
1. ADHD scenario child is not sleeping at night, he watches TV till late night
and then falls asleep but he is tired next morning, management?
a. Sleep hygiene
b. CAMHS
c. Melatonin

2. ADHD scenario, symptoms reported during the last year both at school
and home, mother thinks he need treatment, what is the most
appropriate action?
a. Ask for report from school
b. Assessment by educational psychologist
c. Prescribe methylphenidate

P a g e | 268
Psychiatry

3. Girl with peculiar feeding habits, lost weight, mother thinks she induces
vomiting what is the clinical finding most suggestive of this diagnosis?
a. Teeth erosion
b. Secondary amenorrhea

Oct 2017
1. 20-month girl sort of arching of the back, prefuse sweating seasaw
respiration then she fall a sleep 2-3 minutes, not repeat in the same day, if
interrupted she will get angry, normal development
a. self gratification
b. temper tantrum
c. sandifier syndrome

Feb 2017
1. 10 years boy with night terrors. What to do?
Reassurance
2. A Boy with episodes of loss of concentration frequently at school (not
witnessed at home) but parents say he usually forget what they ask him to
do?
Day dreaming
3. ttt of a case of suspected Autism and learning disorders..
Educational therapy (?)
4. ttt case with poor performance after family stress. He was a always a good
achiever with high marks.
Family therapy (?)
5. ttt of 13- year female with bullying activity ... she has been always normal
before psychotherapy (?)

P a g e | 269
Psychiatry

6. A case girl stays at her room all day. Not meet friends. Not eat with family.
Lost weight. Didn't refuse to take the urine toxicology test which was
negative. Diagnosis ?
Depression
7. A 14 yr Boy. Doing much exercises since age of 11. Now lost weight and on
25 centile.
mostly anorexia nervosa??

June 2016
1. abnormal eating habit
a. anorexia
b. bulimia
2. girl not eating, loss of wt. withdrawal from friends
a. Depression
b. bipolar

3. scenario about Asperger

June 2015
1. 12-yrs old took 6 tabs of paracetamol, bulling at school, level under
treatment, plan to discharge, how you reassure the parent he is ready to
go home??

a. child not on child protection list


b. he didn't try to suicidal before.
c. refer to psychiatry before discharge

P a g e | 270
Psychiatry

2004-2005-2006
1. A girl 15-yr-old stays in her room and doesn't come out Has reduced in
weight and eats less & reduced stool frequency. Parents think she
takes drugs and ask for drug screen she agrees and is negative

a. Agoraphobia
b. Eating disorder
c. Depression
d. Avoidant personality disorder

P a g e | 271
Palliative
pain
management

P a g e | 272
Palliative

Oct 2020
1.EMQ:
a. Ketamine
b. IV morphine
c. intranasal morphine
d. fentanyl patch
e. no drug ttt
f. Femoral nerve block
g. Diclofenac
h. Ibuprofen

A. Girl had RTA come with large hematoma over temporal area with thigh
deformity (fracture) and hypotensive and GCS 13. IV cannula inserted and
CT arranged, orthopaedic want to do splint and child sever thigh pain.
Femoral nerve block

B. Scenario about child with burn by hot coffee on his Lt arm and anterior
chest they apply tap water for 20 min and put cling film but the child still
distressed and crying and have pain.
intranasal morphine

C. 13 years old girl was complaining banding headache and was took regular
paracetamol and ibuprofen in maximum dose for weeks but no effect on
pain now.
No treatment

P a g e | 273
Palliative

Feb 2020
1. EMQ:
a.oral morphine
b.Patient Control analgesia(PCA)
c. intranasal morphine
d.Fentanyl Patch
e. IM morphine
f. rectal paracetamol
g. Oral paracetamol

A. 9 years post-surgery perforated appendicitis and developed peritonitis


PCA

B. 3years chubby accident with fracture Clavicle, screaming


intra nasal morphine

C. 14 years old girl with Chronic regional pain took paracetamol and
ibuprofen,
still in pain
Fentanyl Patch

2. Child came with end stage adrenodystrophy to another hospital with


Respiratory symptoms then his condition become more worse
What to do?
a. ask parent if they have end life plan
b. Call consultant
c. Intubation with ventilation

P a g e | 274
Palliative

Oct 2019
1. malignancy patient with bone pain, she is taking paracetamol 4 times and
Oral morphine prn what to do:
a. rectal diclofenac
b. slow release morphine with morphine breakthrough
c. IV morphine

2. patient was diagnosed with a life limiting condition, his parents ask if
they should refer him to hospice, he needed NG tube what to do:

a. refer them to initiate advanced care plan


b. Refer them to hospice
c. must provide him with full care
d. he is too ill
e. to refer him to holistic palliative care

💠 June 2019
1. EMQ:
a. decrease morphine
b. Senna
c. continuous SC pump
d. hyoscine patch
e. movicol
f. metoclopramide
g. diclofenac Na
h. domperidone

P a g e | 275
Palliative

(all taking oral slow acting morphine with breakthroughs)


A. uncontrolled pain in end stage cancer,,need to increase dose to more than
8 time ……
continuous SC pump
B. controlled pain with difficult swallowing …
hyoscine patch
C. controlled pain + constipation for 3day
Movicol N.B no fentanyle patch in choices
2.Fracture tibia 5yr on pain despite regular paracetamol and brufen next?
a. Oral morphine
b. Codiene
c. Diclofenac

Feb 2019
1.EMQ:
a. Diclofenac
b. Iv morphine
c. Intranasal diamorphine
d. Femoral nerve block
e. Ketamine
f. Nothing (as for the overuse headache)

A. A child with burns, parents washed with cold water for 15 min, now child
had hypotension with cool peripheries
What is the drug of choice for pain management?
Iv morphine

P a g e | 276
Palliative

B. 14-year-old boy met with RTA had head injury followed by swelling
gradually progressive and child also got right femur fracture. Now ortho-
pediatrician
want to manipulation and put cast, so what next management for pain
Femoral nerve block
2. A child with ALL in remission on maintenance chemotherapy medication.
child went to 350 mile trip and parents were forgotten to bring medication.
You spoke with tertiary care treating hospital nurse over the phone about
details of the medication. What is the next plan?
a. Ask the consultant
b. Request urgent previous plan
c. Start the treatment

Oct 2018
1. EMQ
a. IM morphine
b. intranasal dimorphine
c. syrup morphine
d. PCA
e. PUMP
f. FENTANYL patches
g. rectal diclofenac

A. 15 years old girl in chronic regional pain syndrome not releaved by


oral morphine or electric nerve stimulation.
fentanyl patches

P a g e | 277
Palliative

B. 3-years old clavicle fracture in acute pain


IM or intranasal morphine
C. 9 Y-Post peritonitis post surgical pain
PCA pump

June 2018
1. child with cancer Increasing bone pain .. On paracetamol regular and
morphine PRN, has kidney problem, platelet low
a. Morphine pump
b. morphine SC
c. oral morphine slow release twice daily
d. ibuprofen

Feb 2018
1. 12/Child has adrenoleukodystrophy, came to a new hospital in which he
has no register, he developed respiratory failure, what is the best action?
a. Intubate and ventilate
b. Ask the parents if they have end of life plan
c. Call his consultant in his original hospital

Oct 2017
1. EMQ
a. Ondansetron
b. Metoclopramide
c. increase morphine frequency
d. increase morphine dose
e. decrease morphine
f. movical
g. hyoscine patch
h. Senna
i. s/c Pump morphine

P a g e | 278
Palliative

A. scenario of patient with metastasis on slow release morphine and prn


Oral morphine and pain in the last few days reached 8 extra prn
doses max in last 48 hours.
s/c Pump morphine
B. scenario patient with neuroblastoma with metastasis and prn morphine
and pain is controlled but she has developed difficulty in
swallowing
hyoscine patch

C. scenario patient on chemotherapy and didn’t pass normal bowel


habit for 3 days.
Movicol

June 2017
1. EMQ:
A. child with burn with sever pain and screaming
Intranasal or IV morphine
B. child with head injury and fractured femur in sever pain
Nerve block

Feb 2017
1. A patient receiving ttt for malignancy with significant neutropenia, he is on
paracetamol and oramorph as PRN. What to do for his uncontrolled pain?
a. S.C morphine infusion
b. Oral slow-release morphine twice daily
c. transdermal patch

P a g e | 279
Palliative

Oct 2016
EMQ 2: End stage cancer on methotrexate???
a. Decrease the dose of morphine.
b. Decrease the dose of methotrexate.
c. Increase the frequency of morphine
d. Increase the dose of methotrexate.

1. Need morphine more frequently than his plan.


2. Difficulty of swallowing
3. ?

June 2011
1. A child under palliative care treated with Morphine infusion and morphine
patches intermittently having constipation has some abdominal pain with
sena..what will u give him to treat pain?

a. Increased fiber diet


b. Movicol
c. Fantanyl patches
d. Enema

P a g e | 280
Palliative

2004-2005-2006
1. 15-year-old spastic quadriplegia, severe global developmental delay,
admitted with fever and cough, x-ray showed left sided complete
opacification, started on antibiotics and 24 hrs later
sudden deterioration in clinical condition needing 15 liter of oxygen and
the saturation are only 87%, parents wants the child to be transferred to
PICU, what is the next step?

a. Immediately intubate, ventilate and transfer


b. CPAP
c. discuss with trust legal department/authorities
d. speak with social services
e. give morphine

P a g e | 281

You might also like